Вы находитесь на странице: 1из 190

Archive for the LABOR LAW Category

NORKIS VS. BUAT Leave a comment


NORKIS DISTRIBUTORS, INC. AND ALEX D. BUAT, Petitioners, vs DELFIN S.
DESCALLAR, Respondent.
G.R. No. 185255, March 14, 2012
FACTS:
Respondent Delfin S. Descallar was assigned at the Iligan City Branch of petitioner Norkis
Distributors, Inc., a distributor of Yamaha motorcycles. He became a regular employee and was
promoted as Branch Manager. He acted as branch administrator and had supervision and control
of all the employees. Respondent was also responsible for sales and collection
In a memorandum, petitioners required respondent to explain in writing within 48 hrs why he
should not be penalized or terminated for being absent without official leave (AWOL) or
rendering under-time service on certain dates. Respondent explained that he reported to the office
on those dates, but he either went to the bank or followed-up on prospects. As he was still within
city limits, he did not file any official leave or travel record.
Norkis conducted an investigation. Finding that respondent was not able to prove that he was
really in the branch or on official travel, petitioners suspended him for 15 days without pay.
According to petitioners, respondent admitted during the investigation that he used company
time for his personal affairs, but only for a few hours and not the whole day.
While respondent was still suspended, Norkis also found that Respondent committed some
inappropriate and irregular acts such as unexplained low performance of his branch, missing
funds, unauthorized disbursement of funds, irregular transactions.
Petitioners terminated respondents services for loss of trust and confidence and gross
inefficiency. Respondent filed a complaint for illegal suspension and illegal dismissal. LA
favored respondent. Petitioners appealed to NLRC. NLRC reversed the LAs decision and found
respondent to have been validly dismissed. The NLRC, however, upheld the LAs finding that
petitioners are liable to respondent for unpaid wages. Respondent filed MR. It was denied so he
filed with the CA a petition for certiorari. CA reinstated with modification the decision of the
LA. Respondent filed a motion for clarification as to the awards of separation pay and back
wages while petitioners filed MR. CA issued a Resolution stating that as regards respondents
motion for clarification, the separation pay and back wages shall be reckoned from the time
respondent was illegally suspended until finality of its earlier Decision. The CA likewise denied
petitioners MR. Hence, petitioners filed the present petition.
ISSUE:
Was the failure of respondent to reach his monthly sales quota a valid basis for loss of trust and
confidence?

RULING:
NO. Loss of trust and confidence as a ground for termination of an employee under Article 282
of the Labor Code requires that the breach of trust be willful, meaning it must be done
intentionally, knowingly, and purposely, without justifiable excuse. The basic premise for
dismissal on the ground of loss of confidence is that the employee concerned holds a position of
trust and confidence. It is the breach of this trust that results in the employers loss of confidence
in the employee.
Here, there is no question that as petitioners Branch Manager in Iligan City, respondent was
holding a position of trust and confidence. He was responsible for the administration of the
branch, and exercised supervision and control over all the employees. He was also incharge of
sales and collection.
In termination cases, the burden of proof rests upon the employer to show that the dismissal is
for a just and valid cause and failure to do so would necessarily mean that the dismissal was
illegal. The quantum of proof required in determining the legality of an employees dismissal is
only substantial evidence. CA correctly held that petitioners failed to discharge this burden.
Failure to reach the monthly sales quota cannot be considered an intentional and unjustified act
of respondent amounting to a willful breach of trust on his part that would call for his
termination based on loss of confidence. This is not the willful breach of trust and confidence
contemplated in Article 282(c) of the Labor Code. Low sales performance could be attributed to
several factors which are beyond respondents control. To be a valid ground for an employees
dismissal, loss of trust and confidence must be based on a willful breach. To repeat, a breach is
willful if it is done intentionally, knowingly and purposely, without justifiable excuse.
Petitioners having failed to establish by substantial evidence any valid ground for terminating
respondents services, we uphold the finding of the Labor Arbiter and the CA that respondent
was illegally dismissed.
An illegally dismissed employee is entitled to two reliefs: back wages and reinstatement. The
two reliefs provided are separate and distinct. In instances where reinstatement is no longer
feasible because of strained relations between the employee and the employer, separation pay is
granted. The normal consequences of respondents illegal dismissal, then, are reinstatement
without loss of seniority rights, and payment of back wages computed from the time
compensation was withheld from him up to the date of actual reinstatement. Where reinstatement
is no longer viable as an option, separation pay equivalent to one month salary for every year of
service should be awarded as an alternative. The payment of separation pay is in addition to
payment of back wages.
The CA merely clarified the period of payment of back wages and separation pay up to the
finality of its decision modifying the LAs decision. In view of the modification of monetary
awards in the Labor Arbiters decision, the time frame for the payment of back wages and
separation pay is accordingly modified to the finality of the CA decision.

WHEREFORE, the petition for review on certiorari is DENIED.


from Atty. Daan^^
Posted November 17, 2013 by vbdiaz in LABOR LAW

PRUDENTIAL VS. MAURICIO Leave a comment


PRUDENTIAL BANK (now Bank of the Philippine Islands), Petitioner, vs. ANTONIO S.A.
MAURICIO substituted by his legal heirs, MARIA FE, VOLTAIRE, ANTONIO, JR.,
ANTONILO, EARL JOHN, and FRANCISCO ROBERTO all surnamed MAURICIO,
Respondent. G.R. No. 183350; January 18, 2012
FACTS:
Respondent Mauricio was the Branch Manager of Prudential Banks Magallanes Branch in
Makati City when he was dismissed from employment.
Spouses Marcelo and Corazon Cruz (Spouses Cruz) opened a dollar savings account (FXSD No.
221-6) with an initial cash deposit of US$500.00, in the Banks Magallanes Branch. At that time,
Mauricio was already its Branch Manager. Spouses Cruz also executed Deeds of Real Estate
Mortgage over their properties in San Juan in favor of the bank.
An audit investigation was conducted in the Magallanes Branch. The reports of the audit team
showed that from March 1991 to August 1991, credits to FXSD No. 221-6 consisted mostly of
dollar check deposits composed of U.S. Treasury Warrants (USTWs), U.S. Postal Money Orders,
Travellers Express and Amexco Money Orders. Despite the fact that Spouses Cruz were not the
payees of said instruments and neither of them endorsed the same, Mauricio allowed immediate
withdrawals against them. Most of the proceeds of the encashments were then deposited to a
peso savings account, S/A No. 3396, also in the name of the Spouses Cruz.
The dollar checks were eventually returned by their drawee banks for having forged
endorsements, alterations to the stated amounts, or being drawn against insufficient funds, among
other reasons. Allegedly, upon receipt of the returned checks at the Magallanes Branch, Mauricio
debited FXSD No. 221-6, but such debits were made against the uncollected deposits of the
Spouses Cruz. Some of the returned checks and USTWs were lodged to accounts receivable
because the balance of FXSD No. 221-6 was not sufficient to cover the returned checks.
Simultaneously, cash withdrawals were allowed even if S/A No. 3396 did not have sufficient
balance to cover the withdrawals at the time they were made.
Mauricio was directed to report for work at the Head Office immediately. The Prudential Bank
President issued a Memorandum to Mauricio furnishing him with a copy of the audit teams
report and directing him to report in writing within 72 hours from receipt of the memorandum
why the bank should not institute an action against him. The report showed that the bank was
exposed to losses amounting to $774,561.58.

While the investigation against Mauricio was ongoing, as conducted by a Hearing Committee,
the property subject of the Deeds of Real Estate Mortgage executed by the Spouses Cruz was
extrajudicially foreclosed by the Bank for. Spouses Cruz, however, sought the annulment and/or
declaration of nullity of foreclosure in a complaint or civil case filed with RTC- Makati.
The Bank claimed that it sent the proper demand letters to the Spouses but to no avail. Thus, it
was constrained to foreclose the mortgaged property extrajudicially for the settlement of the
obligations of the Spouses Cruz including the returned USTWs, checks and drafts. Later, while
the investigation against Mauricio was still ongoing, the Bank filed an Amended Answer to
implead Mauricio in its counterclaim in the case filed by the Spouses against the former,
contending that he conspired and confederated with the Spouses Cruz to commit the fraud.
The Hearing Committee of the Bank found that there was sufficient evidence to hold Mauricio
guilty of the charges against him. The Board of Directors issued Resolution considering the
recommendation of the Hearing Committee and the Board found Antonio S.A. Mauricio to have
violated Bank policies and regulations and committed imprudent acts prejudicial to the interests
of the Bank, resulting in monetary loss to the Bank and giving rise to loss of trust and
confidence. The services of Mr. Mauricio was terminated and that his retirement benefits was
forfeited.
Mauricio filed with the NLRC a complaint for illegal dismissal with prayer for back wages,
retirement and provident benefits, vacation and sick leave credits, and actual, moral and
exemplary damages, plus attorneys fees. While the illegal dismissal complaint was pending, the
Makati RTC rendered a Decision in favor of the Spouses Cruz and Mauricio. It was affirmed by
the CA and Supreme Court.
On the other hand, LA rendered a Decision holding that the Bank was justified in terminating
Mauricios employment. The LA ruled that even if Mauricio, as branch manager, was clothed
with discretion, he gravely abused it to the detriment and prejudice of the Bank and that he was
afforded procedural due process before he was dismissed. However, LA ordered the bank to pay
Mauricio his 13th month pay and sick leaves earned and reimburse him his actual contributions
to the provident fund, all with legal interest at 12% per annum from date of the decision until
actual payment and/or finality of the decision.
Mauricio filed a partial appeal of the LAs decision with the NLRC, which, however, affirmed
the LAs decision. On appeal, CA set aside the NLRC decision and ruled in favor of Mauricio.
Bank filed the instant petition.
ISSUE:
Whether the acts of Mauricio with respect to the accounts of Spouses Cruz can be considered as
grounds for his termination due to loss of trust and confidence.
RULING:

Civil and labor cases require different quanta of proof the former requiring preponderance of
evidence while the latter only calls for substantial evidence. Despite the dissimilarity, this does
not spell closing our eyes to facts conclusively determined in one proceeding when the
determination of the very same facts are crucial in resolving the issues in another proceeding
pursuant to the doctrine of res judicata.
The present labor case is closely related to the civil case that was decided with finality. In the
civil case, the Banks counterclaim for actual and exemplary damages against Mauricio was
grounded on his alleged violations of office policies when he allowed the encashment and/or
withdrawal prior to clearing of numerous USTWs and dollar checks and allegedly tried
concealing from the Bank the fact that said instruments were returned.
The RTC in the civil case ruled:
Further, this court finds that PRUDENTIALs branch manager MAURICIOs act of allowing
SPOUSES CRUZ to immediately withdraw the instruments is well within his functions as a
branch manager. A person occupying such position exercises a certain degree of discretion with
respect to the accommodations extended to certain valued clients such as herein SPOUSES
CRUZ. Having been recommended by the legal counsel himself of PRUDENTIAL and in view
of the fact that they have substantial deposit with the same bank, it cannot be doubted that
SPOUSES CRUZ were valued clients.
The court also holds that MAURICIO was not in anyway prompted by any malicious motive in
approving the encashment and/or withdrawal.
The acts and omissions alleged by the Bank in the civil case as basis of its counterclaim against
Mauricio, are the very same acts and omissions which were used as grounds to terminate his
employment. Mauricio cannot be held to have abused the discretion he was clothed with absent
some semblance of parameters. In the absence of such guidelines, the validity of Mauricios acts
can be tested by determining whether they were justified under the circumstances. In exercising
his discretion to allow the questioned withdrawals, Mauricio took into consideration the fact that
the Spouses Cruz have substantial deposit and security, and enjoyed a favorable credit standing
with the Bank. No malice can be inferred from Mauricios acts who tried to collect from the
Spouses Cruz and reported all the transactions to the head office; in fact, the Bank never called
his attention to any irregularity in the transactions but even continued to credit the account of the
spouses for the value of the returned checks. Under the circumstances, Mauricio indeed fully
considered the interest of his employer before approving the questioned transactions.
For a dismissal based on loss of trust and confidence to be valid, the breach of trust must be
willful, meaning it must be done intentionally, knowingly, and purposely, without justifiable
excuse. Loss of trust and confidence stems from a breach of trust founded on dishonest, deceitful
or fraudulent act. This is obviously not the case here.
Office Order No. 1596, one of the office orders allegedly violated by Mauricio, provides:
Approving officers shall exercise extreme caution in allowing deposit of, encashment or
withdrawals against foreign and out-of-town checks. Refund to the bank of the amount involved

shall be the personal responsibility and accountability of the officer who authorized the deposit
or encashment over the counter when the check should be returned by the drawee bank for any
reason whatsoever.
The above company directive is an explicit admission that Mauricio was clothed with such
discretion to enter into the questioned transactions as well as a forewarning that in case the
foreign and out-of-town checks were returned for whatever reason, the approving officer, in this
case, Mauricio, shall be personally responsible and accountable. personal responsibility and
accountability could only mean the reimbursement of the value of any dishonored check but
does not mean termination of the approving officers employment for breaching the banks trust
and confidence.
WHEREFORE, the petition for review on certiorari is DENIED.
From Atty. Daan^^
Posted November 17, 2013 by vbdiaz in LABOR LAW

COLEGIO VS. VILLAS Leave a comment


G.R. No. 137795 March 26, 2003
COLEGIO DE SAN JUAN DE LETRAN CALAMBA, petitioner,
vs.
BELEN P. VILLAS, respondent.
FACTS: respondent Belen Villas was employed by the petitioner School as high school teacher
in September 1985. On May 15, 1995, she applied for a study leave for six months, from June to
December 31, 1995. In a letter dated June 2, 1995, Mrs. Angelina Quiatchon, principal of the
high school department, told Villas that her request for study leave was granted for one school
year subject to the following conditions:
1. The requested study leave takes effect on June 5, 1995 and ends on March 31, 1996;
2. The requested study leave involves no remuneration on the part of the School;
3. The documents that justify the requested study leave should be submitted upon return on April
1, 1996;
4. Faculty Manual Section 40 Special Provisions on the Granting of Leave of Absence should
be observed:
a. Once proven beyond reasonable doubt during the period of the approved leave of absence that
the faculty member shall engage himself in employment outside the institution, the
administration shall regard the faculty member on leave as resigned;
b. The maximum length of leave of absence that may be applied for by the faculty member and
granted by administration is twelve (12) months. If, at the lapse of the period, the faculty
member fails to return for work, the administration shall regard the faculty member as resigned.

RESPONDENT ALLEGED: that she intended to utilize the first semester of her study leave to
finish her masteral degree at the Philippine Womens University (PWU). Unfortunately, it did not
push through so she took up an Old Testament course in a school of religion and at the same time
utilized her free hours selling insurance and cookware to augment her familys income. However,
during the second semester of her study leave, she studied and passed 12 units of education
subjects at the Golden Gate Colleges in Batangas City. In response to the letters sent her by
petitioner to justify her study leave, she submitted a certification from Golden Gate Colleges and
a letter explaining why she took up an Old Testament course instead of enrolling in her masteral
class during the first semester.
President and Rector of the School, Fr. Ramonclaro G. Mendez, O. P., wrote her, stating that her
failure to enroll during the first semester was a violation of the conditions of the study leave and
that the reasons she advanced for failure to enroll during the first semester were not acceptable
and thus:
In the first place, prudence dictates that you should have ascertained first that you are still
eligible to study at PWU to finish your masteral degree before applying and securing the
approval of your leave by the School. In the second place, you should have informed the School
at once that you could not enroll in the first semester so that your leave could have been adjusted
for only one-half (1/2) year. Thirdly, your engaging in some part-time business instead of
studying in the first semester of your leave is sufficient justification for the School to consider
you as resigned under the Faculty Manual. And lastly, your failure to study in the first semester
of your study leave without informing the School beforehand constitutes deception, to say the
least, which is not a good example to the other teachers.
Voluntary Arbitrator Mayuga who found that respondent was illegally dismissed. MR denied. CA
affirmed, Hence, this petition.
ISSUE: whether or not respondents alleged violation of the conditions of the study grant
constituted serious misconduct which justified her termination from petitioner School.
HELD: NO
Under the Labor Code, there are twin requirements to justify a valid dismissal from employment:
(a) the dismissal must be for any of the causes provided in Article 282 of the Labor Code
(substantive aspect) and (b) the employee must be given an opportunity to be heard and to defend
himself (procedural aspect).7 The procedural aspect requires that the employee be given two
written notices before she is terminated consisting of a notice which apprises the employee of the
particular acts/omissions for which the dismissal is sought and the subsequent notice which
informs the employee of the employers decision to dismiss him.
In the case at bar, the requirements for both substantive and procedural aspects were not
satisfied.
petitioner School argues that the conduct of respondent breached not only the provisions of the
study grant (which was a contractual obligation) but also the Faculty Manual. Respondent was
thus guilty of serious misconduct which was a ground for termination.

Misconduct is improper or wrongful conduct. It is the transgression of some established and


definite rule of action, a forbidden act, a dereliction of duty, willful in character, and implies
wrongful intent and not mere error of judgment.9 Under Article 282 of the Labor Code, the
misconduct, to be a just cause for termination, must be serious. This implies that it must be of
such grave and aggravated character and not merely trivial or unimportant.
The alleged infractions of the respondent could hardly be considered serious misconduct:
1. Her alleged failure to report for work EXACTLY on April 1, 1996 (respondent reported on
May 15, 1996) is not equivalent to failure to return for work, a sanctionable offense under the
Faculty Manual. Although we give credence to petitioners argument that a private high school
teacher still has work at the end of the schoolyear to assist in the graduation preparations and
in the beginning of the school year to assist in the enrollment such tasks cannot be considered
a teachers main duties, the failure to perform which would be tantamount to dereliction of duty
or abandonment.
2. With regard to her alleged failure to enroll during the first semester, although we agree with
the President and Rector, Fr. Mendez, that respondent should have first ascertained whether she
was still eligible to study at the PWU before applying for a study leave,17 such lapse was more
of an error in judgment rather than an act of serious misconduct. If respondent intended to use
her study leave for other unauthorized purposes, as petitioner would like us to believe, she would
not have enrolled at the Golden Gate Colleges during the second semester. Yet she did, as borne
out by the certification18 prepared by the Registrar of Golden Gate Colleges.
3. Respondent did not violate the prohibition on engaging in employment outside the school as
specified in her study leave grant and as provided in the Faculty Manual. Section 40 (a) of the
Manual. The prohibition against outside employment was enacted to prevent the teacher from
using the study leave period for unsanctioned purposes since the School pays the teacher while
pursuing further studies. That rationale was not violated by respondent for the reason that her
part-time activity of selling insurance and cookware could not have prevented her in any way
from studying and, more importantly, she was not being paid by the School while on leave. How
did the school expect her and her family to survive without any income for one whole year?
Petitioner also failed to comply with the procedural requirements for a valid dismissal. Petitioner
failed to give respondent the first notice which should have informed the latter of the formers
intention to dismiss her. Petitioner argues that it complied with this requirement as there were
several exchanges of communication between the School and respondent regarding the cause of
her termination. However, we find that these letters did not apprise respondent that her dismissal
was being sought by petitioner School as said letters only required respondent to submit proof of
enrollment.
PETITION DENIED.
________________________
NOTES:

Examples of serious misconduct justifying termination, as held in some of our decisions,


include: sexual harassment (the managers act of fondling the hands, massaging the shoulder and
caressing the nape of a secretary);11 fighting within company premises;12 uttering obscene,
insulting or offensive words against a superior;13 misrepresenting that a student is his nephew
and pressuring and intimidating a co-teacher to change that students failing grade to passing.
respondent is not entitled to the six-month study leave and vacation pay, the same was
expressly waived by complainant when she signed conforme to the letter dated June 2, 1995
approving her study leave which states among others, to wit: 2. The requested study leave
involves no remuneration on the part of the school
from Atty. Bayani^^
Posted November 17, 2013 by vbdiaz in LABOR LAW

VICENTE VS. CA Leave a comment


G.R. No. 175988 August 24, 2007
MA. FININA E. VICENTE, Petitioner,
vs.
THE HON. COURT OF APPEALS, Former Seventeenth Division and CINDERELLA
MARKETING CORPORATION, Respondents.
FACTS: Petitioner Finina E. Vicente was employed by respondent Cinderella Marketing
Corporation (Cinderella) as Management Coordinator in January 1990. Prior to her resignation in
February 2000, she held the position of Consignment Operations Manager with a salary of
P27,000.00 a month.5 She was tasked with the oversight, supervision and management of the
Consignment Department dealing directly with Cinderellas consignors.
Petitioner alleged that it has been a practice among the employees of Cinderella to obtain cash
advances by charging the amount from the net sales of Cinderellas suppliers/consignors.
Request for cash advances are approved by Mr. TECSON (AVP-Finance).
After some time, one of Cinderellas suppliers complained about the unauthorized deductions
from the net sales due them. Accordingly, an investigation was conducted and upon initial review
of respondents business records, it appears that petitioner was among those involved in the
irregular and fraudulent preparation and encashment of respondents corporate checks amounting
to at least P500,000.00.
Petitioner alleged that Mr. Tecson demanded her resignation on several occasions. On February
15, 2000, Mr. Tecson allegedly told her MAG-RESIGN KANA AGAD KASI MAIIPIT
KAMI, in the presence of Lizz Villafuerte, the Accounting Manager.9 As a result of this alleged
force and intimidation, petitioner tendered her resignation letter.
Three (3) years after her resignation, petitioner filed a complaint against Cinderella alleging that
her severance from employment was involuntary amounting to constructive dismissal. Cinderella
denied the charge of constructive dismissal.

LA ruled in favour of petitioner; NLRC affirmed. MR denied;


CA reversed on stating that, totality of evidence on record showed that petitioner voluntarily
resigned from her employment; that the subsequent acts of petitioner belie the claim of
constructive dismissal; that after the alleged forced resignation, petitioner attended the meetings
concerning her involvement in the anomalous transactions and even arranged for the settlement
of her consequent liabilities as may be determined during the investigation; that the belated filing
of the complaint militates against petitioner because it is hardly expected from an aggrieved
employee to wait three years before instituting the case. MR denied.
Hence, this petition for review on certiorari.
ISSUE: WON petitioner was constructively dismissed by Cinderella (or was there voluntary
resignation on the part of petitioner?)
HELD: NO, respondent voluntarily resigned.
In termination cases, burden of proof rests upon the employer to show that the dismissal is for a
just and valid cause and failure to do so would necessarily mean that the dismissal was illegal.19
In Mobile Protective & Detective Agency v. Ompad, the Court ruled that should an employer
interpose the defense of resignation, as in the present case, it is still incumbent upon respondent
company to prove that the employee voluntarily resigned.
From the totality of evidence on record, it was clearly demonstrated that respondent Cinderella
has sufficiently discharged its burden to prove that petitioners resignation was voluntary. In
voluntary resignation, the employee is compelled by personal reason(s) to disassociate himself
from employment. It is done with the intention of relinquishing an office, accompanied by the act
of abandonment.21 To determine whether the employee indeed intended to relinquish such
employment, the act of the employee before and after the alleged resignation must be considered.
Petitioner relinquished her position when she submitted the letters of resignation.The resignation
letter submitted on February 15, 2000 confirmed the earlier resignation letter she submitted on
February 7, 2000. The resignation letter contained words of gratitude which can hardly come
from an employee forced to resign.
A careful scrutiny of the said letter shows that it bears the signature of petitioner (contrary to
what the LA stated). More importantly, petitioner admitted having submitted the said letter,
albeit, due to an alleged intimidation.
Subsequently, petitioner stopped reporting for work although she met with the officers of the
corporation to settle her accountabilities but never raised the alleged intimidation employed on
her. Also, though the complaint was filed within the 4-year prescriptive period, its belated filing
supports the contention of respondent that it was a mere afterthought.24 Taken together, these
circumstances are substantial proof that petitioners resignation was voluntary.
Having submitted a resignation letter, it is then incumbent upon her to prove that the resignation
was not voluntary but was actually a case of constructive dismissal with clear, positive, and
convincing evidence.26 Petitioner failed to substantiate her claim of constructive dismissal.

Bare allegations of constructive dismissal, when uncorroborated by the evidence on record,


cannot be given credence.
In St. Michael Academy v. National Labor Relations Commission,28 we ruled that mere
allegations of threat or force do not constitute substantial evidence to support a finding of forced
resignation. We enumerated the requisites for intimidation to vitiate consent as follows:
(1) that the intimidation caused the consent to be given; (2) that the threatened act be unjust or
unlawful; (3) that the threat be real or serious, there being evident disproportion between the evil
and the resistance which all men can offer, leading to the choice of doing the act which is forced
on the person to do as the lesser evil; and (4) that it produces a well-grounded fear from the fact
that the person from whom it comes has the necessary means or ability to inflict the threatened
injury to his person or property. x x x
None of the above requisites was established by petitioner. Neither can we consider the conduct
of audits and other internal investigations as a form of harassment against petitioner. Said
investigation was legitimate and justified
Moreover, we note that petitioner is holding a managerial position with a salary of P27,000.00 a
month. Hence, she is not an ordinary employee with limited understanding such that she would
be easily maneuvered or coerced to resign against her will.
PETITION DENIED.
__________
NOTES:
In administrative proceedings, the quantum of proof required is substantial evidence, which is
more than a mere scintilla of evidence, but such amount of relevant evidence which a reasonable
mind might accept as adequate to justify a conclusion.
from Atty. Bayani^^
Posted November 17, 2013 by vbdiaz in LABOR LAW

BARCENAS VS. NLRC Leave a comment


G.R. No. 87210 July 16, 1990
FILOMENA BARCENAS, petitioner,
vs.
THE NATIONAL LABOR RELATIONS COMMISSION (NLRC), Rev. SIM DEE the
present Head Monk of the Manila Buddha Temple, MANUEL CHUA, in his capacity as the
President and Chairman of the Board of Directors of the Poh Toh Buddhist Association of
the Philippines, Inc., and in his private capacity,respondents.
FACTS: The Buddhist Temple has hired petitioner who speaks the Chinese language as secretary
and interpreter. The head monk, Chua Se Su, had sexual relations with petitioner, which resulted
to the latter giving birth to a child. In May, 1982, of five months before giving birth to the

alleged son of Su on October 12, 1982, petitioner was sent home to Bicol. Upon the death of Su
in July, 1983, complainant remained and continued in her job. In 1985, respondent Manuel Chua
(Chua, for short) was elected President and Chairman of the Board of the Poh Toh Buddhist
Association of the Philippines, Inc. and Rev. Sim Dee for short) was elected Head Buddhist
Priest. Thereafter, Chua and Dee discontinued payment of her monthly allowance and the
additional P500.00 allowance effective 1983. Petitioner and her son were evicted forcibly from
their quarters in the temple by six police officers. She was brought first to the Police precinct in
Tondo and then brought to Aloha Hotel where she was compelled to sign a written undertaking
not to return to the Buddhist temple in consideration of the sum of P10,000.00. Petitioner refused
and Chua shouted threats against her and her son. Her personal belongings including assorted
jewelries were never returned by respondent Chua.
Chua alleges that she was never an employee of the temple, but only attended to the personal
needs of the former head monk, hence was co-terminus with such.
LA ruled in favour of the petitioner. NLRC reversed.
ISSUE: WON petitioner is an employee of the temple
HELD:
Petitioner is an employee of the temple as secretary and interpreter.
Moreover, the work that petitioner performed in the temple could not be categorized as mere
domestic work. We find that petitioner, being proficient in the Chinese language, attended to the
visitors, mostly Chinese, who came to pray or seek advice before Buddha for personal or
business problems; arranged meetings between these visitors and Su and supervised the
preparation of the food for the temple visitors; acted as tourist guide of foreign visitors; acted as
liaison with some goverment offices; and made the payment for the temples Meralco, MWSS
and PLDT bills. Indeed, these tasks may not be deemed activities of a household helper. They
were essential and important to the operation and religious functions of the temple.
In spite of this finding, her status as a regular employee ended upon her return to Bicol in May,
1982 to await the birth of her love-child allegedly by Su. The records do not show that petitioner
filed any leave from work or that a leave was granted her. Neither did she return to work after the
birth of her child on October 12, 1982, whom she named Robert Chua alias Chua Sim Tiong. The
NLRC found that it was only in July, 1983 after Su died that she went back to the Manila
Buddhist Temple. Petitioners pleadings failed to rebut this finding. Clearly, her return could not
be deemed as a resumption of her old position which she had already abandoned.
Thus, her return to the temple was no longer as an employee but rather as Sus mistress who is
bent on protecting the proprietary and hereditary rights of her son and nephew. Finally, while
petitioner contends that she continued to work in the temple after Su died, there is, however, no
proof that she was re-hired by the new Head Monk.
from Atty. Renes^^

Posted November 17, 2013 by vbdiaz in LABOR LAW

BACSIN VS. WAHIMAN Leave a comment


G.R. No. 146053, April 30, 2008
DIOSCORO F. BACSIN, petitioner,
vs.
EDUARDO O. WAHIMAN, respondent.
FACTS: Petitioner is a public school teacher of Pandan Elementary School. Respondent Eduardo
O. Wahiman is the father of AAA, an elementary school student of the petitioner.
AAA claimed that on August 16, 1995, petitioner asked her to be at his office to do an errand.
Once inside, she saw him get a folder from one of the cartons on the floor near his table, and
place it on his table. He then asked her to come closer, and when she did, held her hand, then
touched and fondled her breast. She stated that he fondled her breast five times, and that she felt
afraid. A classmate of hers, one Vincent B. Sorrabas, claiming to have witnessed the incident,
testified that the fondling incident did happen just as AAA related it.
In his defense, petitioner claimed that the touching incident happened by accident, just as he was
handing AAA a lesson book.6 He further stated that the incident happened in about two or three
seconds, and that the girl left his office without any complaint.
CSC found petitioner guilty of Grave Misconduct (Acts of Sexual Harassment), and dismissed
him from the service. Specifically, the CSC found the petitioner to have committed an act
constituting sexual harassment, as defined in Sec. 3 of Republic Act No. (RA) 7877, the AntiSexual Harassment Act of 1995.
CA determined that the issue revolved around petitioners right to due process, and based on its
finding that petitioner had the opportunity to be heard, found that there was no violation of that
right. The CA ruled that, even if petitioner was formally charged with disgraceful and immoral
conduct and misconduct, the CSC found that the allegations and evidence sufficiently proved
petitioners guilt of grave misconduct, punishable by dismissal from the service.
Petitioner argues that the CSC cannot validly adjudge him guilty of an offense, such as Grave
Misconduct (Acts of Sexual Harassment), different from that specified in the formal charge
which was Misconduct. He further argues that the offense of Misconduct does not include
the graver offense of Grave Misconduct.
ISSUE: WON petitioner is guilty of Sexual Harassment
HELD: The formal charge, while not specifically mentioning RA 7877, The Anti-Sexual
Harassment Act of 1995, imputes on the petitioner acts covered and penalized by said law.
Contrary to the argument of petitioner, the demand of a sexual favor need not be explicit or
stated. In Domingo v. Rayala, it was held, It is true that this provision calls for a demand,

request or requirement of a sexual favor. But it is not necessary that the demand, request, or
requirement of a sexual favor be articulated in a categorical oral or written statement. It may be
discerned, with equal certitude, from the acts of the offender.
The CSC found, as did the CA, that even without an explicit demand from petitioner his act of
mashing the breast of AAA was sufficient to constitute sexual harassment. Moreover, under
Section 3 (b) (4) of RA 7877, sexual harassment in an education or training environment is
committed (w)hen the sexual advances result in an intimidating, hostile or offensive
environment for the student, trainee or apprentice. AAA even testified that she felt fear at the
time petitioner touched her.
In grave misconduct, the elements of corruption, clear intent to violate the law, or flagrant
disregard of established rule must be manifest.14 The act of petitioner of fondling one of his
students is against a law, RA 7877, and is doubtless inexcusable. The particular act of petitioner
cannot in any way be construed as a case of simple misconduct.
He is dismissed from service
Petitioner was not denied due process of law, contrary to his claims. The essence of due process
is simply an opportunity to be heard, or, as applied to administrative proceedings, an opportunity
to explain ones side or an opportunity to seek for a reconsideration of the action or ruling
complained of. It is clear that petitioner was sufficiently informed of the basis of the charge
against him, which was his act of improperly touching one of his students. Thus informed, he
defended himself from such charge. The failure to designate the offense specifically and with
precision is of no moment in this administrative case.
from Atty. Renes^^
Posted November 17, 2013 by vbdiaz in LABOR LAW

YRASUEGUI VS. PAL Leave a comment


G.R. No. 168081, October 17, 2008
ARMANDO G. YRASUEGUI, petitioners,
vs.
PHILIPPINE AIRLINES, INC., respondents.
FACTS: THIS case portrays the peculiar story of an international flight steward who was
dismissed because of his failure to adhere to the weight standards of the airline company.
The proper weight for a man of his height and body structure is from 147 to 166 pounds, the
ideal weight being 166 pounds, as mandated by the Cabin and Crew Administration Manual of
PAL.
In 1984, the weight problem started, which prompted PAL to send him to an extended vacation
until November 1985. He was allowed to return to work once he lost all the excess weight. But

the problem recurred. He again went on leave without pay from October 17, 1988 to February
1989.
Despite the lapse of a ninety-day period given him to reach his ideal weight, petitioner remained
overweight. On January 3, 1990, he was informed of the PAL decision for him to remain
grounded until such time that he satisfactorily complies with the weight standards. Again, he was
directed to report every two weeks for weight checks, which he failed to comply with.
On April 17, 1990, petitioner was formally warned that a repeated refusal to report for weight
check would be dealt with accordingly. He was given another set of weight check dates, which
he did not report to.
On November 13, 1992, PAL finally served petitioner a Notice of Administrative Charge for
violation of company standards on weight requirements. Petitioner insists that he is being
discriminated as those similarly situated were not treated the same.
On June 15, 1993, petitioner was formally informed by PAL that due to his inability to attain his
ideal weight, and considering the utmost leniency extended to him which spanned a period
covering a total of almost five (5) years, his services were considered terminated effective
immediately.
LABOR ARBITER: held that the weight standards of PAL are reasonable in view of the nature
of the job of petitioner. However, the weight standards need not be complied with under pain of
dismissal since his weight did not hamper the performance of his duties.
NLRC affirmed.
CA: the weight standards of PAL are reasonable. Thus, petitioner was legally dismissed because
he repeatedly failed to meet the prescribed weight standards. It is obvious that the issue of
discrimination was only invoked by petitioner for purposes of escaping the result of his dismissal
for being overweight.
ISSUE: WON he was validly dismissed.
HELD: YES
A reading of the weight standards of PAL would lead to no other conclusion than that they
constitute a continuing qualification of an employee in order to keep the job. The dismissal of the
employee would thus fall under Article 282(e) of the Labor Code.
In the case at bar, the evidence on record militates against petitioners claims that obesity is a
disease. That he was able to reduce his weight from 1984 to 1992 clearly shows that it is possible
for him to lose weight given the proper attitude, determination, and self-discipline. Indeed,
during the clarificatory hearing on December 8, 1992, petitioner himself claimed that [t]he issue
is could I bring my weight down to ideal weight which is 172, then the answer is yes. I can do it
now.

Petitioner has only himself to blame. He could have easily availed the assistance of the company
physician, per the advice of PAL.
In fine, We hold that the obesity of petitioner, when placed in the context of his work as flight
attendant, becomes an analogous cause under Article 282(e) of the Labor Code that justifies his
dismissal from the service. His obesity may not be unintended, but is nonetheless voluntary. As
the CA correctly puts it, [v]oluntariness basically means that the just cause is solely attributable
to the employee without any external force influencing or controlling his actions. This element
runs through all just causes under Article 282, whether they be in the nature of a wrongful action
or omission. Gross and habitual neglect, a recognized just cause, is considered voluntary
although it lacks the element of intent found in Article 282(a), (c), and (d).
NOTES:
The dismissal of petitioner can be predicated on the bona fide occupational qualification defense.
Employment in particular jobs may not be limited to persons of a particular sex, religion, or
national origin unless the employer can show that sex, religion, or national origin is an actual
qualification for performing the job. The qualification is called a bona fide occupational
qualification (BFOQ). In short, the test of reasonableness of the company policy is used because
it is parallel to BFOQ. BFOQ is valid provided it reflects an inherent quality reasonably
necessary for satisfactory job performance.
The business of PAL is air transportation. As such, it has committed itself to safely transport its
passengers. In order to achieve this, it must necessarily rely on its employees, most particularly
the cabin flight deck crew who are on board the aircraft. The weight standards of PAL should be
viewed as imposing strict norms of discipline upon its employees.
The primary objective of PAL in the imposition of the weight standards for cabin crew is flight
safety.
Separation pay, however, should be awarded in favor of the employee as an act of social justice
or based on equity. This is so because his dismissal is not for serious misconduct. Neither is it
reflective of his moral character.
from Atty. Renes^^
Posted November 17, 2013 by vbdiaz in LABOR LAW

SABEROLA VS. SUAREZ Leave a comment


G.R. No. 151227 July 14, 2008
GREGORIO S. SABEROLA, Petitioner,
vs.
RONALD SUAREZ and RAYMUNDO LIRASAN, JR., Respondents.
FACTS: Case is for illegal dismissal with money claims filed by respondents against petitioner.
Latter is the owner and manager of G.S. Saberola Electrical Services, a firm engaged in the

construction business specializing in installing electrical devices in subdivision homes and in


commercial and non-commercial buildings. Respondents were employed by petitioner as
electricians. They worked from Monday to Saturday and, occasionally, on Sundays, with a daily
wage of P110.00.
Petitioner averred that respondents were part-time project employees and were employed only
when there were electrical jobs to be done in a particular housing unit contracted by petitioner.
He maintained that the services of respondents as project employees were coterminous with each
project. As project employees, the time of rendition of their services was not fixed. Thus, there
was no practical way of determining the appropriate compensation of the value of respondents
accomplishment, as their work assignment varied depending on the needs of a specific project.
LABOR ARBITER: they are project employees, not entitled to benefits
NLRC: affirmed, but said they were illegally dismissed
CA: affirmed
ISSUE: What is their status? And were they illegally dismissed?
HELD: Project employees (BUT were illegally dismissed)
Petitioner, as an electrical contractor, depends for his business on the contracts that he is able to
obtain from real estate developers and builders of buildings. Thus, the work provided by
petitioner depends on the availability of such contracts or projects. The duration of the
employment of his work force is not permanent but coterminous with the projects to which the
workers are assigned. Viewed in this context, the respondents are considered as project
employees of petitioner.
A project employee is one whose employment has been fixed for a specific project or
undertaking, the completion or termination of which has been determined at the time of the
engagement of the employee or where the work or service to be performed is seasonal in nature
and the employment is for the duration of the season.
However, respondents, even if working as project employees, enjoy security of tenure.
Nonetheless, when a project employee is dismissed, such dismissal must still comply with the
substantive and procedural requirements of due process. Termination of his employment must be
for a lawful cause and must be done in a manner which affords him the proper notice and
hearing.
A project employee must be furnished a written notice of his impending dismissal and must be
given the opportunity to dispute the legality of his removal. In termination cases, the burden of
proof rests on the employer to show that the dismissal was for a just or authorized cause.
Employers who hire project employees are mandated to state and prove the actual basis for the
employees dismissal once its veracity is challenged.

Petitioner failed to present any evidence to disprove the claim of illegal dismissal. No evidence
was presented by petitioner to show the termination of the project which would justify the
cessation of the work of respondents. Neither was there proof that petitioner complied with the
substantive and procedural requirements of due process.
from Atty. Renes^^
Posted November 17, 2013 by vbdiaz in LABOR LAW

RBC VS. BALUYOT Leave a comment


G.R. No. 172670, January 20, 2009
RBC CABLE MASTER SYSTEM AND/OR EVELYN CINENSE, Petitioners,
vs.
MARCIAL BALUYOT, Respondent.
FACTS: Herein petitioner RBC Cable Master System (petitioner RBC) is a cable firm engaged
in the business of providing home cable service. Sometime in March 1996, petitioner RBC hired
herein private respondent Marcial Baluyot as a Lineman.
In the middle part of the year 2000, private respondent learned that his outstanding loan from
cash advances accumulated to P18,000.00. The cash advances he made [were] pursuant to a long
time practice for the employees of petitioner RBC to advance amounts of money in the form of
cash vales with the condition that the same be deducted from their monthly salaries on a
staggered or periodic basis. Respondent alleged that he delivered his motorcycle as a security for
said loans. But petitioner avers that such motorcycle was actually leased, which lease only
ceased when respondent no longer owned said vehicle because of non-payment of its financing.
On February 1, 2001, when private respondent reported for work, he was informed that no blank
official receipts could be issued to him for his collection job for that day or for a month because
he is being suspended. Thus, for one month, he did not report for work and when he reported
back to duty, he was told by petitioner RBC that he is now out of job and is considered
terminated.
Petitioner RBC denied dismissing private respondent by contending that it was private
respondent who abandoned his work when, sometime in March 2001, he left without any notice
and never returned back for work. They also alleged that respondent committed several
infractions such as misappropriations and falsification of documents.
LABOR ARBITER ruled that private respondent abandoned his job and committed acts of
dishonesty such as theft of company funds and property.
NLRC ruled that private respondent did not abandon his job but was illegally dismissed.
ISSUE: WON respondent was illegally dismissed
HELD: YES

After respondent was punished with suspension by petitioners, he was admitted back to
work on the condition that he will not repeat the same violations and he will pay back the
sums he owed. This proved that petitioners had condoned the infractions previously
committed by the respondent.
To constitute abandonment, two elements must concur:
(1) the failure to report for work or absence without valid or justifiable reason, and
(2) a clear intention to sever the employer-employee relationship, with the second element
as the more determinative factor and being manifested by some overt acts. Mere absence is
not sufficient. The employer has the burden of proof to show a deliberate and unjustified
refusal of the employee to resume his employment without any intention of returning.
In the case at bar, the charge of abandonment is belied by the following circumstances:
First, the high improbability of private respondent to intentionally abandon his work
considering that he had already served a penalty of suspension for his infractions and
violations as well as the petitioners tacit condonation of the infractions he committed, by
permitting him to go back to work and by asking him to execute a promissory note. It is
incongruent to human nature, that after having ironed things out with his employer, an
employee would just not report for work for no apparent reason. Secondly, there was no
proof that petitioner sent private respondent a notice of termination on the ground of
abandonment, if indeed it is true that he really failed to go back to work. Section 2, Rule
XVI, Book V, Rules and regulations implementing the Labor Code provides that any
employer who seeks to dismiss a worker shall furnish him a written notice stating the
particular act or omission constituting the ground for his dismissal. In cases of
abandonment of work, the notice shall be served at the workers last known address. For
this reason, We are constrained to give credence to private respondents assertion that he
attempted to report back to work but he was just asked to leave as he was considered
terminated. And lastly, private respondents filing of a case for illegal dismissal with the
labor arbiter negates abandonment. As held by the Supreme Court, a charge of
abandonment is totally inconsistent with the immediate filing of a complaint for illegal
dismissal, more so when it includes a prayer for reinstatement.
Finally, an employee who is illegally dismissed is entitled to the twin reliefs of full
backwages and reinstatement. If reinstatement is not viable, separation pay is awarded to
the employee. In awarding separation pay to an illegally dismissed employee, in lieu of
reinstatement, the amount to be awarded shall be equivalent to one (1) month salary for
every year of service.
from Atty. Renes^^
Posted November 17, 2013 by vbdiaz in LABOR LAW

LEYTE GEOTHERMAL VS. PNOC Leave a comment

G.R. No. 170351, March 30, 2011


LEYTE GEOTHERMAL POWER PROGRESSIVE EMPLOYEES UNION ALU
TUCP, Petitioner,
vs.
PHILIPPINE NATIONAL OIL COMPANY ENERGY DEVELOPMENT
CORPORATION, Respondent.
FACTS: Respondent is a GOCC while petitioner is a legitimate labor organization. Among
[respondents] geothermal projects is the Leyte Geothermal Power Project located at the
Greater Tongonan Geothermal Reservation in Leyte. Thus, the [respondent] hired and
employed hundreds of employees on a contractual basis, whereby, their employment was
only good up to the completion or termination of the project and would automatically
expire upon the completion of such project.
Majority of the employees hired by [respondent] in its Leyte Geothermal Power Projects
had become members of petitioner. In view of that circumstance, the petitioner demands
from the [respondent] for recognition of it as the collective bargaining agent of said
employees and for a CBA negotiation with it. However, the [respondent] did not heed such
demands of the petitioner. Sometime in 1998 when the project was about to be completed,
the [respondent] proceeded to serve Notices of Termination of Employment upon the
employees who are members of the petitioner.
On December 28, 1998, the petitioner filed a Notice of Strike with DOLE against the
[respondent] on the ground of purported commission by the latter of unfair labor practice
for refusal to bargain collectively, union busting and mass termination. On the same day,
the petitioner declared a strike and staged such strike.
Secretary of Labor intervened and ordered all workers to return to work. However,
petitioner did not abide.
NLRC: ruled that the employees are PROJECT EMPLOYEES, and the strike as
ILLEGAL
Petitioner Union contends that its officers and members performed activities that were
usually necessary and desirable to respondents usual business.
ISSUE: WON they are project employees
HELD: They are PROJECT EMPLOYEES
Article 280 of the Labor Code contemplates four (4) kinds of employees:
(a) regular employees or those who have been engaged to perform activities which are
usually necessary or desirable in the usual business or trade of the employer;
(b) project employees or those whose employment has been fixed for a specific project or
undertaking[,] the completion or termination of which has been determined at the time of
the engagement of the employee;

(c) seasonal employees or those who work or perform services which are seasonal in nature,
and the employment is for the duration of the season; and
(d) casual employees or those who are not regular, project, or seasonal employees.
Jurisprudence has added a fifth kind a fixed-term employee.
By entering into such a contract, an employee is deemed to understand that his
employment is coterminous with the project. He may not expect to be employed
continuously beyond the completion of the project. It is of judicial notice that project
employees engaged for manual services or those for special skills like those of carpenters or
masons, are, as a rule, unschooled. However, this fact alone is not a valid reason for
bestowing special treatment on them or for invalidating a contract of employment. Project
employment contracts are not lopsided agreements in favor of only one party thereto. The
employers interest is equally important as that of the employee[s] for theirs is the interest
that propels economic activity. While it may be true that it is the employer who drafts
project employment contracts with its business interest as overriding consideration, such
contracts do not, of necessity, prejudice the employee. Neither is the employee left helpless
by a prejudicial employment contract. After all, under the law, the interest of the worker is
paramount.
Unions own admission, both parties had executed the contracts freely and voluntarily
without force, duress or acts tending to vitiate the worker[s] consent. Thus, we see no
reason not to honor and give effect to the terms and conditions stipulated therein.
The litmus test to determine whether an individual is a project employee lies in setting a
fixed period of employment involving a specific undertaking which completion or
termination has been determined at the time of the particular employees engagement.
NOTES:
WHAT IS A PROJECT? In the realm of business and industry, we note that project
could refer to one or the other of at least two (2) distinguishable types of activities. Firstly, a
project could refer to a particular job or undertaking that is within the regular or usual
business of the employer company, but which is distinct and separate, and identifiable as
such, from the other undertakings of the company. Such job or undertaking begins and
ends at determined or determinable times. The typical example of this first type of project
is a particular construction job or project of a construction company. A construction
company ordinarily carries out two or more [distinct] identifiable construction projects:
e.g., a twenty-five-storey hotel in Makati; a residential condominium building in Baguio
City; and a domestic air terminal in Iloilo City. Employees who are hired for the carrying
out of one of these separate projects, the scope and duration of which has been determined
and made known to the employees at the time of employment, are properly treated as
project employees, and their services may be lawfully terminated at completion of the
project.

The term project could also refer to, secondly, a particular job or undertaking that is not
within the regular business of the corporation. Such a job or undertaking must also be
identifiably separate and distinct from the ordinary or regular business operations of the
employer. The job or undertaking also begins and ends at determined or determinable
times.
from Atty. Renes^^
Posted November 17, 2013 by vbdiaz in LABOR LAW

DELA CRUZ VS. MAERSK Leave a comment


G.R. No. 172038 April 14, 2008
DANTE D. DE LA CRUZ, petitioner,
vs.
MAERSK FILIPINAS CREWING, INC. and ELITE SHIPPING A.S., respondents.
FACTS: Respondent Elite Shipping A.S. hired petitioner Dante D. de la Cruz as third
engineer for the vessel M/S Arktis Morning through its local agency in the Philippines, corespondent Maersk Filipinas Crewing Inc. The contract of employment was for a period of
nine months, starting April 19, 1999, with a monthly basic salary of US$1,004.00 plus other
benefits. Petitioner was deployed to Jebel Ali, United Arab Emirates and boarded M/S
Arktis Morning on May 14, 1999.
However, chief engineer Normann Per Nielsen expressed his dissatisfaction over
petitioners performance, the latter has been informed that if he does not improve his
Job/Working performance within a short time he will be signed off according to CBA
Article 1 (7) which provides that the first sixty (60) days of service is to be considered a
probationary period. On June 27, 1999, petitioner was informed of his discharge.
When petitioner got back to Manila, he filed a complaint for illegal dismissal with claims
for the monetary equivalent of the unexpired portion of his contract, damages and
attorneys fees in the National Labor Relations Commission (NLRC) on September 21,
1999.
LABOR ARBITER: ruled that petitioner was dismissed without just cause and due process
as the logbook entry (which respondents claimed to be the first notice to petitioner) was
vague. It failed to expound on or state the details of petitioners shortcomings or
infractions. As such, petitioner was deprived of a real or meaningful opportunity to explain
his side.
NLRC upheld the LAs finding of illegal dismissal but deleted the award of moral and
exemplary damages.
CA reversed.
ISSUE: whether or not petitioner was illegally dismissed by respondents.

HELD: YES
Procedural due process requires that a seaman must be given a written notice of the
charges against him and afforded a formal investigation where he can defend himself
personally or through a representative before he can be dismissed and disembarked from
the vessel. Furthermore, the notice must state with particularity the acts or omissions for
which his dismissal is being sought. This was not complied with as the logbook was vague
and did not contain the infractions being attributed to petitioner.
It was petitioners position that he was already a regular employee when his services were
terminated; respondents, on the other hand, insisted that he was then still on probationary
status. This, according to respondents, entitled them to dismiss him in accordance with the
provisions of Article 1 (7) of the CBA (which allows the master to terminate the contract of
one under probation by merely serving a written notice 14 days prior to the contemplated
discharge)
HE IS NOT a regular employees, he is considered a contractual employee whose rights and
obligations are governed primarily by the POEA Standard Employment Contract for
Filipino Seamen the Rules and Regulations Governing Overseas Employment, and, more
importantly, by Republic Act No. 8042, otherwise known as The Migrant Workers and
Overseas Filipinos Act of 1995.
While we recognize that petitioner was a registered member of the Associated Marine
Officers and Seamens Union of the Philippines which had a CBA with respondent Elite
Shipping A.S. providing for a probationary period of employment, the CBA cannot
override the provisions of the POEA Standard Employment Contract. The law is read into,
and forms part of, contracts. And provisions in a contract are valid only if they are not
contrary to law, morals, good customs, public order or public policy.
from Atty. Renes^^
Posted November 17, 2013 by vbdiaz in LABOR LAW

CASERES VS. URSUMCO Leave a comment


G.R.No. 159343 September 28, 2007
PEDY CASERES and ANDITO PAEL, Petitioners,
vs.
UNIVERSAL ROBINA SUGAR MILLING CORPORATION (URSUMCO) and/or
RESIDENT MANAGER RENE CABATE, Respondents.
FACTS: Universal Robina Sugar Milling Corporation (respondent) is a corporation
engaged in the cane sugar milling business. Petitioners were employees.
At the start of their respective employments, they were made to sign a Contract of
Employment for Specific Project or Undertaking. Petitioners contracts were renewed from

time to time, until May 1999 when they were informed that their contracts will not be
renewed anymore.
Petitioners filed a complaint for illegal dismissal, regularization, incentive leave pay, 13th
month pay, damages and attorneys fees.
LABOR ARBITER: They were not regular employees
NLRC and CA: affirmed
ISSUE: WON they are regular employees
HELD: They are NOT regular employees
ART. 280. Regular and Casual Employees. The provision of written agreement to the
contrary notwithstanding and regardless of the oral agreement of the parties, an
employment shall be deemed to be regular where the employee has been engaged to
perform activities which are usually necessary or desirable in the usual business or trade of
the employer, except where the employment has been fixed for a specific project or
undertaking the completion or termination of which has been determined at the time of the
engagement of the employee or where the work or services to be performed is seasonal in
nature and the employment is for the duration of the season. An employment shall be
deemed to be casual if it is not covered by the preceding paragraph: Provided, That, any
employee who has rendered at least one year of service, whether such service is continuous
or broken, shall be considered a regular employee with respect to the activity in which he is
employed and his employment shall continue while such actually exists.
The principal test for determining whether an employee is a project employee or a regular
employee is whether the employment has been fixed for a specific project or undertaking,
the completion or termination of which has been determined at the time of the engagement
of the employee.
A true project employee should be assigned to a project which begins and ends at
determined or determinable times, and be informed thereof at the time of hiring.
The very nature of the terms and conditions of complainants hiring reveals that they were
required to perform phases of special projects for a definite period after, their services are
available to other farm owners. This is so because the planting of sugar does not entail a
whole year operation, and utility works are comparatively small during the off-milling
season.
It must be noted that there were intervals in petitioners respective employment contracts,
and that their work depended on the availability of such contracts or projects.
Consequently, the employment of URSUMCOs work force was not permanent but coterminous with the projects to which the employees were assigned and from whose payrolls
they were paid

The fact that petitioners were constantly re-hired does not ipso facto establish that they
became regular employees. Their respective contracts with respondent show that there
were intervals in their employment. In petitioner Caseress case, while his employment
lasted from August 1989 to May 1999, the duration of his employment ranged from one day
to several months at a time, and such successive employments were not continuous. With
regard to petitioner Pael, his employment never lasted for more than a month at a time.
These support the conclusion that they were indeed project employees, and since their
work depended on the availability of such contracts or projects, necessarily the
employment of respondents work force was not permanent but co-terminous with the
projects to which they were assigned and from whose payrolls they were paid.
Moreover, even if petitioners were repeatedly and successively re-hired, still it did not
qualify them as regular employees, as length of service is not the controlling determinant of
the employment tenure of a project employee, but whether the employment has been fixed
for a specific project or undertaking, its completion has been determined at the time of the
engagement of the employee. Further, the proviso in Article 280, stating that an employee
who has rendered service for at least one (1) year shall be considered a regular employee,
pertains to casual employees and not to project employees.
from Atty. Renes^^
Posted November 17, 2013 by vbdiaz in LABOR LAW

SERRANO VS. SANTOS TRANSIT Leave a comment


Serrano vs Severino Santos Transit
GR 187698
Facts:
After 14 years of service or on July 14, 2006, petitioner applied for optional retirement
from the company whose representative advised him that he must first sign the already
prepared Quitclaim before his retirement pay could be released. As petitioners request to
first go over the computation of his retirement pay was denied, he signed the Quitclaim on
which he wrote U.P. (under protest) after his signature, indicating his protest to the
amount of P75,277.45 which he received, computed by the company at 15 days per year of
service.
Petitioner soon after filed a complaint, alleging that the company erred in its computation
since under Republic Act No. 7641, otherwise known as the Retirement Pay Law, his
retirement pay should have been computed at 22.5 days per year of service to include the
cash equivalent of the 5-day service incentive leave (SIL) and 1/12 of the 13th month pay
which the company did not.
The company maintained, however, that the Quitclaim signed by petitioner barred his
claim and, in any event, its computation was correct since petitioner was not entitled to the

5-day SIL and pro-rated 13th month pay for, as a bus conductor, he was paid on
commission basis.
Issue: WON 22.5 days retirement pay per year of service is the correct formula.
Held:
Republic Act No. 7641 which was enacted on December 9, 1992 amended Article 287 of the
Labor Code by providing for retirement pay to qualified private sector employees in the
absence of any retirement plan in the establishment. The pertinent provision of said law
reads:
Section 1. Article 287 of Presidential Decree No. 442, as amended, otherwise known as the
Labor Code of the Philippines, is hereby amended to read as follows:
In the absence of a retirement plan or agreement providing for retirement benefits of
employees in the establishment, an employee upon reaching the age of sixty (60) years or
more, but not beyond sixty-five (65) years which is hereby declared the compulsory
retirement age, who has served at least five (5) years in the said establishment, may retire
and shall be entitled to retirement pay equivalent to at least one-half (1/2) month salary for
every year of service, a fraction of at least six (6) months being considered as one whole
year.
Unless the parties provide for broader inclusions, the term one-half (1/2) month salary shall
mean fifteen (15) days plus one-twelfth (1/12) of the 13th month pay and the cash
equivalent of not more than five (5) days of service incentive leaves.
Admittedly, petitioner worked for 14 years for the bus company which did not adopt any
retirement scheme. Even if petitioner as bus conductor was paid on commission basis then,
he falls within the coverage of R.A. 7641 and its implementing rules. As thus correctly ruled
by the Labor Arbiter, petitioners retirement pay should include the cash equivalent of the
5-day SIL and 1/12 of the 13th month pay.
from Atty. Alba^^
Posted November 17, 2013 by vbdiaz in LABOR LAW

PALTENG VS. UCPB Leave a comment


Palteng vs UCPB
GR 172199
Facts:
Petitioner Elizabeth D. Palteng was the Senior Assistant Manager/Branch Operations
Officer of respondent United Coconut Planters Bank in its Banaue Branch in Quezon City.

On April 15, 1996, Area Head and Vice-President Eulallo S. Rodriguez reported to the
banks Internal Audit and Credit Review Division that bank client Clariza L. Mercado-The
Red Shop has incurred Past Due Domestic Bills Purchased (BP) of P34,260,000. After
conducting a diligence audit, the division reported to the Audit and Examination
Committee that Palteng committed several offenses under the Employee Discipline Code in
connection with Mercados Past Due Domestic BP. It also recommended that the matter be
referred to the Committee on Employee Discipline for proper disposition.
On August 14, 1996, Palteng was required to explain why no disciplinary action should be
taken against her.
In response, Palteng explained that while she admitted committing a major offense that
may cause her dismissal, she claimed that it was an honest mistake.
After hearing and investigation, the committee recommended Paltengs dismissal. On
October 25, 1996, Palteng was dismissed with forfeiture of all benefits.
Palteng filed a complaint for illegal dismissal seeking reinstatement to her former position
without loss of seniority rights with full backwages, or in the alternative, payment of
separation pay with full backwages, and recovery of her monetary claims with damages.
Issue: WON backwages is proper.
Held:
Settled is the rule that an employee who is illegally dismissed from work is entitled to
reinstatement without loss of seniority rights, and other privileges as well as to full
backwages, inclusive of allowances, and to other benefits or their monetary equivalent
computed from the time his compensation was withheld from him up to the time of his
actual reinstatement. However, in the event that reinstatement is no longer possible, the
employee may be given separation pay instead.
Notably, reinstatement and payment of backwages are distinct and separate reliefs. The
award of one does not bar the other. Backwages may be awarded without reinstatement,
and reinstatement may be ordered without awarding backwages.
In a number of cases, the Court, despite ordering reinstatement or payment of separation
pay in lieu of reinstatement, has not awarded backwages as penalty for the misconduct or
infraction committed by the employee.
In the case at bar, petitioner admitted that she granted the BP accommodation against
Mercados personal checks beyond and outside her authority. The Labor Arbiter, the
NLRC and the Court of Appeals all found her to have committed an error of judgment,
honest mistake, honest mistake vis--vis a major offense.

Since petitioner was not faultless in regard to the offenses imputed against her, we hold that
the award of separation pay only, without backwages, is proper.
CA decision affirmed.
from Atty. Alba^^
Posted November 17, 2013 by vbdiaz in LABOR LAW

HYATT VS. CATINOY Leave a comment


Hyatt Taxi Services vs Catinoy
GR 143204
Facts:
Both respondent and Saturnino are employees of petitioner. They are also officers of the
legitimate labor organization/association of the petitioner. Both of them had a fist fight. After
securing medical treatment, complainant on the same day filed a criminal complaint for physical
injuries with the fiscals Office and Saturnino was arrested by the police for investigation.
On August 25, Jaime Dublin, Chairman of the Board of the Association, issued a memorandum
to the Operation Manager of the petitioner company, Mr. H. Caraig recommending the indefinite
suspension of respondent and Saturnino. On August 26, 1995, the Asst. Vice-President of the
petitioner company Melchor Acosta, Jr. issued a memorandum preventively suspending for 30
days the services of the respondent and Saturnino pending investigation in response to the
recommendation of the Chairman of the Board of the Association.
Respondent, filed a complaint for illegal suspension, unpaid wages, and damages against both
the association-union and management on August 28, 1995 before the NLRC
After the lapse of his 30 days preventive suspension, respondent reported for work but he was
not allowed to resume his duties as a taxi driver allegedly, since he is pursuing the criminal
complainant for physical injuries against Saturnino, the associations President and the complaint
for the illegal suspension with the NLRC.
On October 12, 1995, since there was no response from Respondent company, complainant
decided to amend his complaint to include constructive dismissal as an additional cause of action
since he was not allowed to resume his employment after the lapse of his preventive suspension.
Issue: WON respondent is constructively dismissed.
Held:
Clearly, constructive dismissal had already set in when the suspension went beyond the
maximum period allowed by law. Section 4, Rule XIV, Book V of the Omnibus Rules provides

that preventive suspension cannot be more than the maximum period of 30 days. Hence, we have
ruled that after the 30-day period of suspension, the employee must be reinstated to his former
position because suspension beyond this maximum period amounts to constructive dismissal.
CA Affirmed.
from Atty. Alba^^
Posted November 17, 2013 by vbdiaz in LABOR LAW

HINOGUIN VS. ECC Leave a comment


Hinoguin vs Employees Compensation Commission
GR 84307
Facts:
Sgt. Lemick Hinoguin was a sergeant in A company, 14th Infantry Battalion, 5th Infantry
Division, Philippine Army.
On August 1, 1985, Sgt. Hinoguin, Cpl. Rogelio Clavo and Dft. Nicomedes Alibuyog sought
permission from Capt. Frankie Besas, to go on overnight pass to Aritao, Nueva Viscaya. Capt.
Besas orally granted them permission to go to Aritao and to take their issued firearms with them
considering that Aritao was regarded as a critical place.
The three soldiers went to Dft. Alibuyogs home for a meal and some drinks. At around 7:00 PM,
the soldiers headed back to the headquarters. They boarded a tricycle. When they reached the
poblacion, Alibuyog dismounted from the tricycle. Not noticing that his rifles safety lever was
on semi-automatic, he accidentally touched the trigger, firing a single shot in the process and
hitting Sgt. Hinoguin in the left lower abdomen. Sgt. Hinoguin died a few days after the incident.
In the investigation conducted by the 14th Infantry Battalion, it was found that the shooting of
Sgt. Hinoguin was purely accidental in nature and that he died in the line of duty. The Life of
Duty Board of Officers recommended that all benefits due the legal dependents of the late Sgt.
Hinoguin be given.
However, when the father of the deceased made a claim from GSIS, the same was denied on the
ground that the deceased was neither at his work place nor performing his duty as a soldier of the
Philippine Army at the time of his death. This denial was confirmed by the respondent ECC.
Issue: WON the death of Sgt. Hinoguin is compensable.
Held:
Article 167 (k) of the Labor Code as amended defines a compensable injury quite simply as
any harmful change in the human organism from any accident arising out of and in the course of

the employment. The Amended (Implementing) Rules have, however, elaborated considerably
on the simple and succinct statutory provision. Rule III, Section 1 (a) reads:
SECTION 1. Grounds. (a) For the injury and the resulting disability or death to be compensable,
the injury must be the result of an employment accident satisfying all of the following grounds:
(1) The employee must have been injured at the place work requires him to be;
(2) The employee must have been performing his official functions; and
(3) If the injury is sustained elsewhere, the employee must have been executing an order for the
employer.
The concept of a work place referred to in Ground 1, for instance, cannot always be literally
applied to a soldier on active duty status, as if he were a machine operator or a worker in an
assembly line in a factory or a clerk in a particular fixed office. Obviously, a soldier must go
where his company is stationed. In the instant case, Aritao, Nueva Viscaya was not, of course,
Carranglan, Nueva Ecija. Aritao being approximately 1-1/2 hours away from the latter by public
transportation. But Sgt. Hinoguin, Cpl. Clavo and Dft. Alibuyog had permission from their
Commanding Officer to proceed to Aritao, and it appears to us that a place which soldiers have
secured lawful permission to be at cannot be very different, legally speaking, from a place where
they are required to go by their commanding officer. They were not on vacation leave.
It may be noted in this connection that a soldier on active duty status is really on 24 hours a day
official duty status and is subject to military discipline and military law 24 hours a day. He is
subject to call and to the orders of his superior officers at all times, 7 days a week, except, of
course, when he is on vacation leave status (which Sgt. Hinoguin was not). Indeed, it appears to
us that a soldier should be presumed to be on official duty unless he is shown to have clearly and
unequivocally put aside that status or condition temporarily by, e.g., going on an approved
vacation leave.
Thus, we think that the work-connected character of Sgt. Hinoguins injury and death was not
effectively precluded by the simple circumstance that he was on an overnight pass to go to the
home of Dft. Alibuyog, a soldier under his own command. Sgt. Hinoguin did not effectively
cease performing official functions because he was granted a pass. While going to a fellow
soldiers home for a few hours for a meal and some drinks was not a specific military duty, he
was nonetheless in the course of performance of official functions.
from Atty. Alba^^
Posted November 17, 2013 by vbdiaz in LABOR LAW

RP VS. KAWASHIMA Leave a comment


G.R. No. 160352, July 23, 2008
REPUBLIC OF THE PHILIPPINES, represented by Department of Labor and

Employment (DOLE),Petitioner,
vs.
KAWASHIMA TEXTILE MFG., PHILIPPINES, INC., Respondent.
FACTS: KFWU filed with DOLE Regional Office No. IV, a Petition for Certification Election to
be conducted in the bargaining unit composed of 145 rank-and-file employees of respondent.
Respondent-company filed a Motion to Dismiss8 the petition on the ground that KFWU did not
acquire any legal personality because its membership of mixed rank-and-file and supervisory
employees violated Article 245 of the Labor Code, and its failure to submit its books of account
contravened the ruling of the Court in Progressive Development Corporation v. Secretary,
Department of Labor and Employment.
Med-Arbiter Bactin found KFWUs legal personality defective and dismissed its petition for
certification election, stating that, Since petitioners members are mixture of rank and file and
supervisory employees, petitioner union, at this point [in] time, has not attained the status of a
legitimate labor organization. Petitioner should first exclude the supervisory employees from it
membership before it can attain the status of a legitimate labor organization.
Respondent filed with DOLE Regional Office No. IV a Petition for Cancellation of
Charter/Union Registration of KFWU,13 the final outcome of which, unfortunately, cannot be
ascertained from the records.
KFWU appealed to the DOLE which granted the appeal; ordered the case be remanded to the
office of origin for the immediate conduct of certification election xxx CA reversed. MR denied.
Hence, this petition.
ISSUE:
(1) whether a mixed membership of rank-and-file and supervisory employees in a union is a
ground for the dismissal of a petition for certification election in view of the amendment brought
about by D.O. 9, series of 1997, which deleted the phraseology in the old rule that [t]he
appropriate bargaining unit of the rank-and-file employee shall not include the supervisory
employees and/or security guards; and
(2) whether the legitimacy of a duly registered labor organization can be collaterally attacked in a
petition for a certification election through a motion to dismiss filed by an employer such as
Kawashima Textile Manufacturing Phils., Inc.
HELD: The petition is imbued with merit.
The key to the closure that petitioner seeks could have been Republic Act (R.A.) No. 9481 [AN
ACT STRENGTHENING THE WORKERS' CONSTITUTIONAL RIGHT TO SELFORGANIZATION, AMENDING FOR THE PURPOSE PRESIDENTIAL DECREE NO. 442,
AS AMENDED, OTHERWISE KNOWN AS THE LABOR CODE OF THE PHILIPPINES]
Sections 8 and 9 (See NOTES)

However, R.A. No. 9481 took effect only on June 14, 2007;26 hence, it applies only to labor
representation cases filed on or after said date.27 As the petition for certification election subject
matter of the present petition was filed by KFWU on January 24, 2000,28 R.A. No. 9481 cannot
apply to it. There may have been curative labor legislations29 that were given retrospective
effect,30 but not the aforecited provisions of R.A. No. 9481, for otherwise, substantive rights and
interests already vested would be impaired in the process.
Instead, the law and rules in force at the time of the filing by KFWU of the petition for
certification election on January 24, 2000 are R.A. No. 6715,32 amending Book V of
Presidential Decree (P.D.) No. 442 (Labor Code),33as amended, and the Rules and Regulations
Implementing R.A. No. 6715,34 as amended by Department Order No. 9, series of 1997.35
One area of contention has been the composition of the membership of a labor organization,
specifically whether there is a mingling of supervisory and rank-and-file employees and how
such questioned mingling affects its legitimacy.
Effective 1989, R.A. No. 6715 restored the prohibition against the questioned mingling in one
labor organization, viz:
Sec. 18. Article 245 of the same Code, as amended, is hereby further amended to read as follows
Art. 245. Ineligibility of managerial employees to join any labor organization; right of
supervisory employees. Managerial employees are not eligible to join, assist or form any labor
organization. Supervisory employees shall not be eligible for membership in a labor organization
of the rank-and-file employees but may join, assist or form separate labor organizations of their
own. (Emphasis supplied)
Unfortunately, just like R.A. No. 875, R.A. No. 6715 omitted specifying the exact effect any
violation of the prohibition would bring about on the legitimacy of a labor organization.
Thus, when the issue of the effect of mingling was brought to the fore in Toyota,48 the Court,
citing Article 245 of the Labor Code, as amended by R.A. No. 6715, held:
Clearly, based on this provision, a labor organization composed of both rank-and-file and
supervisory employees is no labor organization at all. It cannot, for any guise or purpose, be a
legitimate labor organization. Not being one, an organization which carries a mixture of rankand-file and supervisory employees cannot possess any of the rights of a legitimate labor
organization, including the right to file a petition for certification election for the purpose of
collective bargaining. It becomes necessary, therefore, anterior to the granting of an order
allowing a certification election, to inquire into the composition of any labor organization
whenever the status of the labor organization is challenged on the basis of Article 245 of the
Labor Code xxxx
In the case at bar, as respondent unions membership list contains the names of at least twentyseven (27) supervisory employees in Level Five positions, the union could not, prior to purging
itself of its supervisory employee members, attain the status of a legitimate labor organization.

Not being one, it cannot possess the requisite personality to file a petition for certification
election.
But then, on June 21, 1997, the 1989 Amended Omnibus Rules was further amended by
Department Order No. 9, series of 1997 (1997 Amended Omnibus Rules). Specifically, the
requirement under Sec. 2(c) of the 1989 Amended Omnibus Rules that the petition for
certification election indicate that the bargaining unit of rank-and-file employees has not been
mingled with supervisory employees was removed.
Consequently, the Court reverses the ruling of the CA and reinstates that of the DOLE granting
the petition for certification election of KFWU.
II. Now to the second issue of whether an employer like respondent may collaterally attack the
legitimacy of a labor organization by filing a motion to dismiss the latters petition for
certification election.
Except when it is requested to bargain collectively,62 an employer is a mere bystander to any
petition for certification election; such proceeding is non-adversarial and merely investigative,
for the purpose thereof is to determine which organization will represent the employees in their
collective bargaining with the employer.63 The choice of their representative is the exclusive
concern of the employees; the employer cannot have any partisan interest therein; it cannot
interfere with, much less oppose, the process by filing a motion to dismiss or an appeal from
it;64 not even a mere allegation that some employees participating in a petition for certification
election are actually managerial employees will lend an employer legal personality to block the
certification election.65 The employers only right in the proceeding is to be notified or informed
thereof.66
The amendments to the Labor Code and its implementing rules have buttressed that policy even
more.
Petition is GRANTED.
___________
NOTES:
Section 8. Article 245 of the Labor Code is hereby amended to read as follows:
Art. 245. Ineligibility of Managerial Employees to Join any Labor Organization; Right of
Supervisory Employees. Managerial employees are not eligible to join, assist or form any labor
organization. Supervisory employees shall not be eligible for membership in the collective
bargaining unit of the rank-and-file employees but may join, assist or form separate collective
bargaining units and/or legitimate labor organizations of their own. The rank and file union and
the supervisors union operating within the same establishment may join the same federation or
national union.
Section 9. A new provision, Article 245-A is inserted into the Labor Code to read as follows:
Art. 245-A. Effect of Inclusion as Members of Employees Outside the Bargaining Unit. The
inclusion as union members of employees outside the bargaining unit shall not be a ground for

the cancellation of the registration of the union. Said employees are automatically deemed
removed from the list of membership of said union. (Emphasis supplied)
Moreover, under Section 4, a pending petition for cancellation of registration will not hinder a
legitimate labor organization from initiating a certification election, viz:
Sec. 4. A new provision is hereby inserted into the Labor Code as Article 238-A to read as
follows:
Art. 238-A. Effect of a Petition for Cancellation of Registration. A petition for cancellation of
union registration shall not suspend the proceedings for certification election nor shall it prevent
the filing of a petition for certification election.
In case of cancellation, nothing herein shall restrict the right of the union to seek just and
equitable remedies in the appropriate courts. (Emphasis supplied)
Furthermore, under Section 12 of R.A. No. 9481, employers have no personality to interfere with
or thwart a petition for certification election filed by a legitimate labor organization, to wit:
Sec. 12. A new provision, Article 258-A is hereby inserted into the Labor Code to read as
follows:
Art. 258-A. Employer as Bystander. In all cases, whether the petition for certification election
is filed by an employer or a legitimate labor organization, the employer shall not be considered a
party thereto with a concomitant right to oppose a petition for certification election. The
employers participation in such proceedings shall be limited to: (1) being notified or informed
of petitions of such nature; and (2) submitting the list of employees during the pre-election
conference should the Med-Arbiter act favorably on the petition. (Emphasis supplied)
from Atty. Bayani^^
Posted November 17, 2013 by vbdiaz in LABOR LAW

UNITED PEPSI-COLA VS. LAGUESMA Leave a


comment
G.R. No. 122226 March 25, 1998
UNITED PEPSI-COLA SUPERVISORY UNION (UPSU), petitioner,
vs.
HON. BIENVENIDO E. LAGUESMA and PEPSI-COLA PRODUCTS, PHILIPPINES,
INC. respondents.
FACTS: Petitioner is a union of supervisory employees. It appears that on March 20, 1995 the
union filed a petition for certification election on behalf of the route managers at Pepsi-Cola
Products Philippines, Inc. However, its petition was denied by the med-arbiter and, on appeal, by
the Secretary of Labor and Employment, on the ground that the route managers are managerial

employees and, therefore, ineligible for union membership under the first sentence of Art. 245 of
the Labor Code, which provides:
Ineligibility of managerial employees to join any labor organization; right of supervisory
employees. Managerial employees are not eligible to join, assist or form any labor
organization. Supervisory employees shall not be eligible for membership in a labor organization
of the rank-and-file employees but may join, assist or form separate labor organizations of their
own.
Petitioner brought this suit challenging the validity of the order, dismissed.
Hence, this petition. Pressing for resolution its contention that the first sentence of Art. 245 of the
Labor Code, so far as it declares managerial employees to be ineligible to form, assist or join
unions, contravenes Art. III, 8 of the Constitution which provides:
The right of the people, including those employed in the public and private sectors, to form
unions, associations, or societies for purposes not contrary to law shall not be abridged.
ISSUES:
(1) whether the route managers at Pepsi-Cola Products Philippines, Inc. are managerial
employees and
(2) whether Art. 245, insofar as it prohibits managerial employees from forming, joining or
assisting labor unions, violates Art. III, 8 of the Constitution.
HELD: YES and NO
As a class, managers constitute three levels of a pyramid: (1) Top management; (2) Middle
Management; and (3) First-line Management [also called supervisors].
FIRST-LINE MANAGERS The lowest level in an organization at which individuals are
responsible for the work of others is called first-line or first-level management. First-line
managers direct operating employees only; they do not supervise other managers. Examples of
first-line managers are the foreman or production supervisor in a manufacturing plant, the
technical supervisor in a research department, and the clerical supervisor in a large office. Firstlevel managers are often called supervisors.
MIDDLE MANAGERS The term middle management can refer to more than one level in an
organization. Middle managers direct the activities of other managers and sometimes also those
of operating employees. Middle managers principal responsibilities are to direct the activities
that implement their organizations policies and to balance the demands of their superiors with
the capacities of their subordinates. A plant manager in an electronics firm is an example of a
middle manager.

TOP MANAGERS Composed of a comparatively small group of executives, top management


is responsible for the overall management of the organization. It establishes operating policies
and guides the organizations interactions with its environment. Typical titles of top managers are
chief executive officer, president, and senior vice-president. Actual titles vary from one
organization to another and are not always a reliable guide to membership in the highest
management classification.
A distinction exists between those who have the authority to devise, implement and control
strategic and operational policies (top and middle managers) and those whose task is simply to
ensure that such policies are carried out by the rank-and-file employees of an organization (firstlevel managers/supervisors). What distinguishes them from the rank-and-file employees is that
they act in the interest of the employer in supervising such rank-and-file employees.
Managerial employees may therefore be said to fall into two distinct categories: the
managers per se, who compose the former group described above, and the supervisors who
form the latter group.
#1: It appears that this question was the subject of two previous determinations by the Secretary
of Labor and Employment, in accordance with which this case was decided by the med-arbiter.
To qualify as managerial employee, there must be a clear showing of the exercise of managerial
attributes under paragraph (m), Article 212 of the Labor Code as amended. Designations or titles
of positions are not controlling. As to the route managers and accounting manager, we are
convinced that they are managerial employees. Their job descriptions clearly reveal so (Workers
Alliance Trade Union (WATU) v. Pepsi-Cola Products Philippines, Inc., Nov. 13, 1991)
This finding was reiterated in Case No. OS-A-3-71-92. entitled In Re: Petition for Direct
Certification and/or Certification Election-Route Managers/Supervisory Employees of PepsiCola Products Phils.Inc.
* doctrine of res judicata certainly applies to adversary administrative proceedings
Thus, we have in this case an experts view that the employees concerned are managerial
employees within the purview of Art. 212.
At the very least, the principle of finality of administrative determination compels respect for the
finding of the Secretary of Labor that route managers are managerial employees as defined by
law in the absence of anything to show that such determination is without substantial evidence to
support it.
The Court now finds that the job evaluation made by the Secretary of Labor is indeed supported
by substantial evidence. The nature of the job of route managers is given in a four-page
pamphlet, prepared by the company, called Route Manager Position Description, the pertinent
parts of which read:
A. BASIC PURPOSE
A Manager achieves objectives through others.
As a Route Manager, your purpose is to meet the sales plan; and you achieve this objective
through the skillful MANAGEMENT OF YOUR JOB AND THE MANAGEMENT OF YOUR

PEOPLE.
These then are your functions as Pepsi-Cola Route Manager. Within these functions managing
your job and managing your people you are accountable to your District Manager for the
execution and completion of various tasks and activities which will make it possible for you to
achieve your sales objectives.
Xxxx
Distinction is evident in the work of the route managers which sets them apart from supervisors
in general. Unlike supervisors who basically merely direct operating employees in line with set
tasks assigned to them, route managers are responsible for the success of the companys main
line of business through management of their respective sales teams. Such management
necessarily involves the planning, direction, operation and evaluation of their individual teams
and areas which the work of supervisors does not entail.
The route managers cannot thus possibly be classified as mere supervisors because their work
does not only involve, but goes far beyond, the simple direction or supervision of operating
employees to accomplish objectives set by those above them.
While route managers do not appear to have the power to hire and fire people (the evidence
shows that they only recommended or endorsed the taking of disciplinary action against
certain employees), this is because thisis a function of the Human Resources or Personnel
Department of the company.
# 2: Constitutionality of Art. 245
Art.245 is the result of the amendment of the Labor Code in 1989 by R.A. No. 6715, otherwise
known as the Herrera-Veloso Law. Unlike the Industrial Peace Act or the provisions of the Labor
Code which it superseded, R.A. No. 6715 provides separate definitions of the terms
managerial and supervisory employees, as follows:
Art. 212. Definitions. . . .
(m) managerial employee is one who is vested with powers or prerogatives to lay down and
execute management policies and/or to hire transfer, suspend, lay off, recall, discharge, assign or
discipline employees. Supervisory employees are those who, in the interest of the employer,
effectively recommend such managerial actions if the exercise of such authority is not merely
routinary or clerical in nature but requires the use of independent judgment. All employees not
falling within any of the above definitions are considered rank-and-file employees for purposes
of this Book.
The distinction between top and middle managers, who set management policy, and front-line
supervisors, who are merely responsible for ensuring that such policies are carried out by the
rank and file, is articulated in the present definition. 30 When read in relation to this definition in
Art. 212(m), it will be seen that Art. 245 faithfully carries out the intent of the Constitutional
Commission in framing Art. III, 8 of the fundamental law.
*Framers Intent: MR. LERUM. My amendment is on Section 7, page 2, line 19, which is to
insert between the words people and to the following: WHETHER EMPLOYED BY THE
STATE OR PRIVATE ESTABLISHMENTS. In other words, the section will now read as
follows: The right of the people WHETHER EMPLOYED BY THE STATE OR PRIVATE

ESTABLISHMENTS to form associations, unions, or societies for purposes not contrary to law
shall not be abridged.
Nor is the guarantee of organizational right in Art. III, 8 infringed by a ban against managerial
employees forming a union. The right guaranteed in Art. III, 8 is subject to the condition that its
exercise should be for purposes not contrary to law. In the case of Art. 245, there is a rational
basis for prohibiting managerial employees from forming or joining labor organizations.
PETITION is DISMISSED.
From Atty. Bayani^^
Posted November 17, 2013 by vbdiaz in LABOR LAW

VICTORIANO VS. ELIZALDE UNION Leave a comment


BENJAMIN VICTORIANO, plaintiff-appellee, vs. ELIZALDE ROPE WORKERS
UNION and ELIZALDE ROPE FACTORY, INC., defendants, ELIZALDE ROPE
WORKERS UNION, defendant-appellant.
GRN L-25246 September 12, 1974
FACTS:
Benjamin Victoriano (Appellee), a member of the religious sect known as the Iglesia ni Cristo,
had been in the employ of the Elizalde Rope Factory, Inc. (Company) since 1958. He was a
member of the Elizalde Rope Workers Union (Union) which had with the Company a CBA
containing a closed shop provision which reads as follows: Membership in the Union shall be
required as a condition of employment for all permanent employees workers covered by this
Agreement.
Under Sec 4(a), par 4, of RA 975, prior to its amendment by RA 3350, the employer was not
precluded from making an agreement with a labor organization to require as a condition of
employment membership therein, if such labor organization is the representative of the
employees. On June 18, 1961, however, RA 3350 was enacted, introducing an amendment to
par 4 subsection (a) of sec 4 of RA 875, as follows: xxx but such agreement shall not cover
members of any religious sects which prohibit affiliation of their members in any such labor
organization.
Being a member of a religious sect that prohibits the affiliation of its members with any labor
organization, Appellee presented his resignation to appellant Union. The Union wrote a formal
letter to the Company asking the latter to separate Appellee from the service because he was
resigning from the Union as a member. The Company in turn notified Appellee and his counsel
that unless the Appellee could achieve a satisfactory arrangement with the Union, the Company
would be constrained to dismiss him from the service.

Appellee filed an action for injunction to enjoin the Company and the Union from dismissing
Appellee. The Union invoked the union security clause of the CBA and assailed the
constitutionality of RA 3350 and contends it discriminatorily favors those religious sects which
ban their members from joining labor unions.
ISSUE:
Whether Appellee has the freedom of choice in joining the union or not.
RULING:
YES. The Constitution and RA 875 recognize freedom of association. Sec 1 (6) of Art III of the
Constitution of 1935, as well as Sec 7 of Art IV of the Constitution of 1973, provide that the
right to form associations or societies for purposes not contrary to law shall not be abridged.
Section 3 of RA 875 provides that employees shall have the right to self-organization and to
form, join of assist labor organizations of their own choosing for the purpose of collective
bargaining and to engage in concerted activities for the purpose of collective bargaining and
other mutual aid or protection. What the Constitution and the Industrial Peace Act recognize and
guarantee is the right to form or join associations. A right comprehends at least two broad
notions, namely: first, liberty or freedom, i.e., the absence of legal restraint, whereby an
employee may act for himself without being prevented by law; and second, power, whereby an
employee may, as he pleases, join or refrain from joining an association. It is, therefore, the
employee who should decide for himself whether he should join or not an association; and
should he choose to join, he himself makes up his mind as to which association he would join;
and even after he has joined, he still retains the liberty and the power to leave and cancel his
membership with said organization at any time. The right to join a union includes the right to
abstain from joining any union. The law does not enjoin an employee to sign up with any
association.
The right to refrain from joining labor organizations recognized by Section 3 of the Industrial
Peace Act is, however, limited. The legal protection granted to such right to refrain from joining
is withdrawn by operation of law, where a labor union and an employer have agreed on a closed
shop, by virtue of which the employer may employ only members of the collective bargaining
union, and the employees must continue to be members of the union for the duration of the
contract in order to keep their jobs. By virtue of a closed shop agreement, before the enactment
of RA 3350, if any person, regardless of his religious beliefs, wishes to be employed or to keep
his employment he must become a member of the collective bargaining union. Hence, the right
of said employee not to join the labor union is curtailed and withdrawn.
To that all-embracing coverage of the closed shop arrangement, RA No.3350 introduced an
exception, when it added to Section 4 (a) (4) of the Industrial Peace Act the following proviso:
but such agreement shall not cover members of any religious sects which prohibit affiliation of
their members in any such labor organization. Republic Act No. 3350 merely excludes ipso jure
from the application and coverage of the closed shop agreement the employees belonging to any
religious sects which prohibit affiliation of their members with any labor organization. What the
exception provides is that members of said religious sects cannot be compelled or coerced to join
labor unions even when said unions have closed shop agreements with the employers; that in

spite of any closed shop agreement, members of said religious sects cannot be refused
employment or dismissed from their jobs on the sole ground that they are not members of the
collective bargaining union. It does not prohibit the members of said religious sects from
affiliating with labor unions. It still leaves to said members the liberty and the power to affiliate,
or not to affiliate, with labor unions. If, notwithstanding their religious beliefs, the members of
said religious wets prefer to sign up with the labor union, they can do so. If in deference and
fealty to their religious faith, they refuse to sign up, they can do so; the law does not coerce them
to join; neither does the law prohibit them from joining, and neither may the employer or labor
union compel them to join.
The Company was partly absolved by law from the contractual obligation it had with the Union
of employing only Union members in permanent positions. It cannot be denied, therefore, that
there was indeed an impairment of said union security clause.
The prohibition to impair the obligation of contracts is not absolute and unqualified. The
prohibition is general. The prohibition is not to be read with literal exactness, for it prohibits
unreasonable impairment only. In spite of the constitutional prohibition, the State continues to
possess authority to safeguard the vital interests of its people. Legislation appropriate to
safeguarding said interests may modify or abrogate contracts already in effect. For not only are
existing laws read into contracts in order to fix the obligations as between the parties, but the
reservation of essential attributes of sovereign power is also read into contracts as a postulate of
the legal order. The contract clause of the Constitution. must be not only in harmony with, but
also in subordination to, in appropriate instances, the reserved power of the state to safeguard the
vital interests of the people. This has special application to contracts regulating relations between
capital and labor which are not merely contractual, and said labor contracts, for being impressed
with public interest, must yield to the common good.
The purpose to be achieved by RA 3350 is to insure freedom of belief and religion, and to
promote the general welfare by preventing discrimination against those members of religious
sects which prohibit their members from joining labor unions, confirming thereby their natural,
statutory and constitutional right to work, the fruits of which work are usually the only means
whereby they can maintain their own life and the life of their dependents.
The individual employee, at various times in his working life, is confronted by two aggregates of
power collective labor, directed by a union, and collective capital, directed by management. The
union, an institution developed to organize labor into a collective force and thus protect the
individual employee from the power of collective capital, is, paradoxically, both the champion of
employee rights, and a new source of their frustration. Moreover, when the Union interacts with
management, it produces yet a third aggregate of group strength from which the individual also
needs protection the collective bargaining relationship.
The free exercise of religious profession or belief is superior to contract rights. In case of
conflict, the latter must yield to the former.
The purpose of RA 3350 is to serve the secular purpose of advancing the constitutional right to
the free exercise of religion, by averting that certain persons be refused work, or be dismissed

from work, or be dispossessed of their right to work and of being impeded to pursue a modest
means of livelihood, by reason of union security agreements. To help its citizens to find gainful
employment whereby they can make a living to support themselves and their families is a valid
objective of the state. The Constitution even mandated that the State shall afford protection to
labor, promote full employment and equality in employment, ensure equal work opportunities
regardless of sex, race or creed and regulate the relation between workers and employers.
The primary effects of the exemption from closed shop agreements in favor of members of
religious sects that prohibit their members from affiliating with a labor organization, is the
protection of said employees against the aggregate force of the collective bargaining agreement,
and relieving certain citizens of a burden on their religious beliefs; and by eliminating to a
certain extent economic insecurity due to unemployment, which is a serious menace to the
health, morals, and welfare of the people of the State, the Act also promotes the well-being of
society. It is our view that the exemption from the effects of closed shop agreement does not
directly advance, or diminish, the interests of any particular religion. Although the exemption
may benefit those who are members of religious sects that prohibit their members from joining
labor unions, the benefit upon the religious sects is merely incidental and indirect.
The purpose of RA 3350 was not to grant rights to labor unions. The rights of labor unions are
amply provided for in Republic Act No. 875 and the new Labor Code.
The Act does not require as a qualification, or condition, for joining any lawful association
membership in any particular religion or in any religious sect; neither does the Act require
affiliation with a religious sect that prohibits its members from joining a labor union as a
condition or qualification for withdrawing from a labor union. Joining or withdrawing from a
labor union requires a positive act Republic Act No. 3350 only exempts members with such
religious affiliation from the coverage of closed shop agreements. So, under this Act, a religious
objector is not required to do a positive act-to exercise the right to join or to resign from the
union. He is exempted ipso jure without need of any positive act on his part.
WHEREFORE, the instant appeal is dismissed.
from Atty. Daan^^
Posted November 17, 2013 by vbdiaz in LABOR LAW

LI YAO VS. CIR Leave a comment


WILLIAM LI YAO, petitioner, vs. COLLECTOR OF INTERNAL REVENUE, respondent.
GRN L-11875 December 23, 1963
FACTS:
Petitioner is a naturalized Filipino of Chinese parents, the eldest son of a prosperous local
businessman by the name of Li Chay Too, who died sometime in 1948. In 1945 petitioner
organized the Li Yao & Company and made himself managing partner; from 1948, to February

1955 he was president of, and owned shares in, the Li Chay Too & Sons, Inc.; and in 1950 he
organized a corporation known as the Far East Realty & Investment Co. of which he was also
stockholder and president. Petitioner filed his income tax returns for the years 1945 to 1951.
However, a deficiency income tax in the amount of P5,470.98 was assessed against him, which
he paid.
In 1952 the Collector of Internal Revenue (CIR) believing that petitioner had not reported his
true incomes for the previous years, appointed a team to examine his books, an additional
assessment of P899,794.02 was made against. A second team of investigators was appointed and
this team recommended a deficiency income tax assessment of P2,722,030.33. This team
employed what is known as the net worth or inventory method. A third team was appointed, this
team recommended an assessment of P1,505,768.54 against petitioner; the inventory method was
also used in making this assessment. Demand was made for the collection of said assessment so
petitioner herein presented a petition with the CTA for the review of the said assessment. CTA
found that the amount of the income tax deficiency due from petitioner is P424,536.77.
Petitioner Li Yao sought to reconsider the decision and the assessment, alleging that the sum of
P5,470.98 paid by him as additional tax for the years 1945 to 1947 should be credited against his
deficiency income taxes, so that instead of P424,536.77 the -sum due should be only
P411,294.12. The court approved this petition for recoupment and reduced the assessment to
P411,293.80.
Both petitioner and respondent appealed from the decision of CTA. Petitioner Li Yao raised the
questions on the validity of the net worth method of inventory used against him, and assails the
CTAs refusal to grant petitioners request that the deficiency income assessed be distributed
evenly over the taxable years
ISSUE:
Whether the contention of petitioner should be allowed.
RULING:
NO. The taxpayer has the means of proving the existence of the obligation and it is he that must
produce such proof. The procedure followed by the CTA is that laid down by the rules on
evidence; that is, that the taxpayer who alleges that an obligation still exists must prove the
existence thereof by preponderance of evidence. The obligor or taxpayer has the means of
proving that the obligation does not exist or has been paid; the Government collecting the tax
cannot be expected to find the evidence itself, because it is natural that the taxpayer would try to
suppress such evidence as may prove that the obligation still exists.
After reading the arguments presented by petitioner and considering that the witnesses for
petitioner herein are his father-in-law and his wife and their testimonies failed to convince the
judges of the court below, the Court finds no potent reason why the findings of the court below
that heard the evidence should be disturbed.

The use of the inventory method is authorized under Section 15 of the National Internal Revenue
Code, as amended, which authorizes the CIR to assess taxes due a taxpayer from any other
available fact or evidence. If a taxpayer commits a violation of the law, hiding his income to
evade payment of taxes, the Government must be permitted to resort to all evidence or sources
available to determine his said income, so that the tax may be collected for public purposes.
There is and there should be a presumption of regularity accorded this action of the CIR in
assessing the tax on the best evidence obtainable, otherwise it, would be impossible to assess
taxes due from a dishonest taxpayer.
In the case at bar the existence of assets or properties appearing in the name of the taxpayer or in
the name of his dummies or friends, without the taxpayer being able to give a definite reasonable
explanation for their existence justifies the CTA and this Court to resort to the inventory method
of assessment, such being necessary and at the same time just and equitable.
The last legal question raised is petitioners claim that the unreported incomes which appeared
during the last years of the period of assessment should not be considered as having been earned
during the years in which said incomes appeared, but should be spread throughout the whole
period covered by the assessment, that is, from 1945 to 1951.
Petitioner does not claim that the amounts appearing in the last period of the assessment were
acquired through savings or accumulated savings or any slow and continuous process, such that
the income cannot be distributed to any particular year of the period of assessment.
Section 39 of the National Internal Revenue Code requires the taxpayer to report yearly to the
CIR the income that he gets during the year from whatever source and include the same in the
taxable year in which the income was received by him. It is to be presumed that the income was
earned at the time that it appeared in the possession or control of the taxpayer, in accordance with
the rule that the law has been followed.
If Petitioners contention is to be followed, a taxpayer would be encouraged to hide his income
because in any case, if his unreported income would be discovered afterwards the said income,
although appearing in one year, would be distributed over a period of years. In other words, we
will have a rule, as advocated by petitioners counsel, that would not discourage the hiding of
taxable income because any discovery of any unreported income could always be allowed to be
distributed over a period of years. In the case at bar, the distribution over a period of years
demanded by petitioner would bring about a reduction of the tax assessed by the Court of Tax
Appeals from P424,536,77 to P232,416.59, or about one-half of the assessment made by the
CTA. We are not prepared to permit such unauthorized reduction in public taxes favorable to a
dishonest taxpayer and prejudicial to the interests of the State.
Wherefore, the decision appealed from is affirmed.
from Atty. Daan^^
Posted November 17, 2013 by vbdiaz in LABOR LAW

HEIRS OF CRUZ VS. CIR Leave a comment


HEIRS OF TEODOLO M. CRUZ, (represented by ARSENIA, FREDESWINDA,
TEODOLO, JR., ERLINDA, EDGARDO and MYRNA, all surnamed CRUZ). MARY
CONCEPCION and EDGARDO CRUZ, petitioners, vs. COURT OF INDUSTRIAL
RELALATIONS, SANTIAGO RICE MILL and KING HONG AND COMPANy,
respondents.
GRN L-23331-32 December 27, 1969
FACTS:
June 21, 1952, the Santiago Labor Union, composed of workers of the Santiago Rice Mill, a
business enterprises engaged in the, buying. and milling of palay at Santiago, Isabela, and owned
and operated by King Hong Co., Inc., filed before the respondent Court of Industrial Relations
(CIR) cases for petition for overtime pay, premium pay for night, Sunday and holiday work, and
for reinstatement of workers illegally laid off. CIR favored the union by a split decision of 3-2
vote. The case reached the Supreme Court but the SC still favored the union. The SC remanded
the records for enforcement by respondent CIR. In examination of books, said CIRs Chief
Examiner filed his Partial Report regarding the computation of the benefits rendered in the case
in favor of the Union.
Petitioners claim that in this computation of the Examiner did not, include the claims of 70 other
laborers whose total claims (for back wages), at the rate of P6,300.00 each and would be
P441,000.00. Therefore, the correct. grand total amount due the laborers would be 864,756.74.
The trial judge took no action on the latest Urgent Motion of the union, wherein it emphasized
that respondent, with Courts action rejecting its appeal, no longer had any excuse for refusing to
comply with the deposit Order. Instead, an unscheduled conference was called and held on
October 31, 1963 in the chambers of the trial judge, and attended by representatives of
respondent firm, including their counsels of record and the President of the union and 8 directors
of the union. Four of these nine union representatives, including the union president himself, had
no claims or awards whatever under the judgment. Said union officials were not assisted by
counsel, as petitioner Mary Concepcion, counsel of record of the union, was not present, not
having been notified of the conference.
In this conference respondent firm made again the same offer to settle and quitclaim the
judgment in favor of the union members for the same amount of P110,000.00, which offer had
already been rejected by the union at the earlier conference held on June 25, 1963. But this time,
respondent and the directors of the union decided to settle the case amicably with the payment by
the firm of the same amount of P110,000.00 which was deposited with the Courts disbursing
officer immediately upon the signing of the settlement which will be prepared by the respondent
firm through its counsel.
One of the union director together with 49 of its members questioned the amicable settlement
that took place. They claim that the Board of Directors did not have any express authority of the
members of the Santiago Labor Union to enter into any compromise for the sum of P110,000.00,
that it was tainted by apparent bad faith on the part of the President of the Union, that the amount

of P110,000.00 is unconscionable, considering, that the total claims of the members of the union
is more than P400,000.00.
ISSUE:
Whether the amicable settlement is valid.
RULING:
NO. Petitioners were not accorded due process of law. The union was deprived of the assistance
of its counsel. The lack or due deliberation and caution in the trial judges instant approval of the
settlement is seen from the stipulations therein that the union thereby waived and quitclaimed
any and all claims which it may have against the respondent, as well as the claim of each and
every one of the members of the union against respondent, when precisely the authority of the
union board members to enter into any such compromise or settlement was under express
challenge by petitioner Magalpo, a board member herself which the trial judge completely
disregarded.
Petitioners were deprived of the formal conference on the and of their right to be assisted by the
union counsel as expressly requested, so that a fair hearing could be accorded petitioners and an
opportunity afforded them to air their serious charges of bad faith and lack authority against the
Union leadership. Certainly, all these serious questions and charges made by petitioners could
have been threshed out and verified, if the formal conference had been held with the presence of
union counsel.
The transcript of the conference is deficient and does not reflect the actual discussions and
proceedings. This is to be deplored, for in a matter of such great importance, especially where the
union officials were unassisted by counsel in an unscheduled conference, care should be taken by
the trial judge that the proceedings are faithfully recorded.
We find the forcing through of the settlement arbitrary, unfair and unconscionable.
Another reason for striking down the settlement is the lack of any express or specific authority of
the president and majority. of the union board of directors to execute the same and scale down
the estimated P423,756.74 judgment liability of respondent firm in favor of the individual union
members to P110,000.00. On the contrary, petitioner board member Magalpo timely challenged
the authority or the union board to execute any such settlement, expressly informing the trial
judge that the union had specifically appointed an entity in Manila, the CREAM, Inc., as its
attorney in-fact and exclusive authorized representative for the evaluation, adjustment and
-liquidation or its claim against respondent. These union members have repudiated the former
union president, Maylem and his board of directors, for having betrayed the union members, and
the new union leadership.
The authority of the union, to execute a settlement of the judgment award in favor of the
individual union members, cannot be presumed but must be expressly granted.
Just as this Court has stricken down unjust exploitation of laborers by oppressive employers, so
will it strike down their unfair treatment by their own unworthy leaders. The Constitution enjoins

the State to afford protection to labor. Fair dealing is equally demanded of unions as well as of
employers in their dealings with employees. The union has been evolved as an organization of
collective strength for the protection of labor against the unjust exactions of capital, but equally
important is the requirement of fair dealing between the union and its members, which is
fiduciary in nature, and arises out of two factors: one is the degree of dependence of the
individual employee on the union organization; the other, a corollary of the first, is the
comprehensive power vested in the union with respect to the individual. The union may, be
considered but the agent of its members for the purpose of securing for them fair and just wages
and good working conditions and is subject to the obligation of giving the members as its
principals all information relevant to union and labor matters entrusted to it.
The union leadership in the case at bar was recreant in its duty towards the union members in
apparently having failed to disclose to the union members the full situation of their judgment
credit against respondent, to wit, that they were in the advantageous position of being able to
require enforcement of the respondent courts P200,000.00-deposit order, and in presuming that
it had authority to waive and quitclaim the estimated P423,756.74-judgment credit of the union
members for the unconscionable amount of P110,000.00, which had already been previously
rejected by the workers. Respondent firm could not claim that it dealt in good faith with the
union officials, for it hastily executed the purported settlement notwithstanding the serious
charges of bad faith against the union leadership, and the non-holding of the scheduled
conference where the union leaders, at their express request, could be duly assisted by union
counsel.
The interests of the individual worker can be better protected on the whole by a strong union
aware of its moral and legal obligations to represent the rank and file faithfully and secure for
them the best wages and working terms and conditions in the process of collective bargaining. As
has been aptly pointed out, the will of the majority must prevail over that of the minority in the
process, for under the philosophy of collective responsibility, an employer who bargains in good
faith should be entitled to rely upon the promises and agreements of the union representatives
with whom he must deal. under the compulsion of, law and contract. The collective bargaining
process should be carried on between parties who can mutually respect and rely upon the
authority of each other. Where, however, collective bargaining process is not involved, and what
is at stake are back wages already earned by the individual workers by way of overtime,
premium and differential pay, and final judgment has been rendered in their favor, as in the
present case, the real parties in interest with direct material interest, as against the union which
has only served as a vehicle for collective action to enforce their just claims, are the individual
workers themselves. Authority of the union to waive or quitclaim all or part of the judgment
award in favor of the individual workers cannot be lightly presumed but must be expressly
granted, and the employer, as judgment debtor, must deal in all good faith with the union as the
agent of the individual workers. The Court in turn should certainly verify and assure itself of the
fact and extent of the authority of the union leadership to execute any compromise or settlement
of the judgment on behalf of the individual workers who are the real judgment creditors.
The settlement in the case at bar was precipitately approved without verification of the union
boards authority to execute the compromise settlement and that there was no such authority.

WHEREFORE, the respondent Courts Orders are hereby declared null and void and set aside.
from Atty. Daan^^
Posted November 17, 2013 by vbdiaz in LABOR LAW

SSC VS. ALBA Leave a comment


SOCIAL SECURITY COMMISSION AND APOLONIO LAMBOSO, PETITIONERS, VS.
FAR S. ALBA, RESPONDENT. G.R. No. 165482, July 23, 2008
FACTS:
Petitioner (Apolonio Lamboso) worked in Hda. La Roca (owned by Far Alba) from 1960 to 1973
as cabo, in Hda. Alibasao from 1973 to 1979 as overseer and in Hda. Kamandag from 1979 to
1984. The latter 2 haciendas are both owned by Ramon S. Benedicto. When he filed a claim for
retirement pension benefit with the SSS, however, the same was denied on the ground that he
had 39 monthly contributions to his credit and so he could not qualify for monthly pension under
R.A. No. 1161 ( the Social Security Act of 1954). Lamboso appealed the denial of his claim by
filing a petition before the Commission wherein he alleged that he should be entitled to monthly
retirement pension. He prayed for the adjustment of the date of his Social Security (SS) coverage
and for the remittance of his delinquent monthly contribution.
Private respondent Ramon S. Benedicto alleged that he was only a lessee of Hdas. Albasao and
Kamandag. When he took over as lessee thereof, there was no available records to support the
petitioners claim of employment. He also avers that the petitioner was employed by him from
1973 to 1984 (1973 to 1979 in Hda. Alibasao and from 1979 to 1984 in Hda. Kamandag) and all
of his employment records were already destroyed and damaged by termites. He prays that the
petition be dismissed for lack of cause of action. Respondent Far Alba (Hda. La Roca) was motu
propio declared in default for failure to file his answer.
The Commission ordered Far Alba to pay to the SSS the delinquent monthly contributions of
Apolonio Lamboso from June 18, 1960 to April; and Respondent Ramon Benedicto to pay to
SSS the delinquent monthly contributions due the petitioner for the period May 1973 to 1984.
The SSS, on the other hand, is ordered to pay Apolonio Lamboso his retirement benefit upon the
filing of the claim therefore.
Far Alba moved for reconsideration of the Commissions resolution, citing lack of notice and
denial of due process. The Commission denied the motion. Alba filed a Petition for Review
before the CA. CA reversed and set aside both the resolution and the order of the Commission. It
held that Far Alba cannot be considered as an employer of Lamboso prior to 1970 because as
administrator of the family-owned hacienda, he is not an employer under Section 8 (c) of the
Social Security Act of 1954; and that that since it was Arturo Alba, Sr., Far Albas father, who
had failed to remit the SS contributions prior to 1970, Lamboso should have asserted his claim
before the estate proceedings of his deceased employer
ISSUE:

Whether an administrator could be considered an employer within the scope of the Social
Security Act of 1954.
RULING:
YES. Section 8 (c), Social Security Act of 1954 (amended by P.D. No. 1202 and P.D. No. 1636)
defines an employer as any person, natural or juridical, domestic or foreign, who carries on in
the Philippines any trade or business, industry, undertaking, or activity of any kind and uses the
services of another person who is under his orders as regards the employment, except the
Government and any of its political subdivisions, branches or instrumentalities, including
corporations owned or controlled by the Government. Section 8 (d) defines an employee as
any person who performs services for an employer in which either or both mental and physical
efforts are used and who receives compensation for such services where there is an employeremployee relationship.
Based on the testimony of Lamboso and his witness it is evidently that Far Alba had indeed
served as Lambosos employer from 1965 to 1970 or, at the very least, he had served as the
haciendas administrator before 1970.
Far Alba was no ordinary administrator. He was no less than the son of the haciendas owner and
as such he was an owner-in-waiting prior to his fathers death. He was a member of the owners
family assigned to actively manage the operations of the hacienda. As he stood to benefit from
the haciendas successful operation, he ineluctably took his job and his fathers wishes to heart.
Far Alba and the owners interests in the business where plainly and inextricably linked by filial
bond. He more than just acted in the interests of his father as employer, and could himself pass
off as the employer, the one carrying on the undertaking.
Applying the control test which is used to determine the existence of employer-employee
relationship for purposes of compulsory coverage under the SSS law, Far Alba is technically
Lambosos employer. The essential elements of an employer-employee relationship are: (a) the
selection and engagement of the employee; (b) the payment of wages; (c) the power of dismissal;
and (d) the power of control with regard to the means and methods by which the work is to be
accomplished, with the power of control being the most determinative factor.
Lamboso testified that he was selected and his services were engaged by Far Alba himself. Far
Alba held the prerogative of terminating Lambosos employment. Lamboso also testified in a
direct manner that he had been paid his wages by Far Alba. This testimony was seconded by
Lambosos co-worker, Rodolfo Sales. It is not essential for the employer to actually supervise the
performance of duties of the employee, it is sufficient that the former has a right to wield the
power (Power of Control).
Article 167(f) of the Labor Code which deals with employees compensation and state insurance
fund defines and employer as any person, natural or juridical, employing the services of the
employee. It also defines a person as any individual, partnership, firm, association, trust,
corporation or legal representative thereof. Plainly, Far Alba, as the hacienda administrator, acts
as the legal representative of the employer and is thus an employer within the meaning of the law
liable to pay the SS contributions.

Section 8 (c) of the Social Security Act of 1954 is broad enough to include those persons acting
directly or indirectly in the interest of the employer. If under Article 212 (e), persons acting in the
interest of the employer, directly or indirectly, are obliged to follow the government labor
relations policy, it could be reasonably concluded that such persons may likewise be held liable
for the remittance of SS contributions which is an obligation created by law and an is employees
right protected by law.
Far Albas is accountable to the SSS for Lambosos unremitted contributions form 1960 to 1970.
In any event, the Court sustains the jurisdiction of the Commission over disputes under the
Social Security Act with respect to coverage, benefits, contributions and penalties thereon or
any other matter related thereto. Moreover, the Court agrees with the Commissions assertion
that an action for remittance of SS monthly contributions is not a type of money claim which
needs to be filed against the estate proceedings. Employers are required to remit the
contributions to the SSS by mandate of law. Actions of this type should be treated in much the
same way as taxesthat they are not required to be filed against the estate and that they be
claimed against the heirs of the errant decedent.
WHEREFORE, the petition is GRANTED.
From Atty. Daan^^
Posted November 13, 2013 by vbdiaz in LABOR LAW

TANCINCO vs. PURA FERER-CALLEJA Leave a


comment
TANCINCO vs. PURA FERER-CALLEJA
FACTS:
This special civil action for certiorari seeks to annul the Resolution and the Decision of the
Bureau of Labor Relations setting aside the order which decreed the inclusion and counting of
the 56 segregated votes for the determination of the results of the election of officers of Imperial
Textile Mills Inc. Monthly Employees Association (ITM-MEA).
Private respondents are the prime organizers of ITM-MEA. While said respondents were
preparing to file a petition for direct certification of the Union as the sole and exclusive
bargaining agent of ITMs bargaining unit, the unions Vice-President, Carlos Dalmacio was
promoted to the position of Department Head, thereby disqualifying him for union membership.
Said incident, among others led to a strike spearheaded by Lacanilao group, respondents herein.
Another group however, led by herein petitioners staged a strike inside the company premises.
After four (4) days the strike was settled. An agreement was entered into by the representatives
of the management, Lacanilao group and the Tancinco group the relevant terms of which are as
follows:

1. That all monthly-paid employees shall be United under one union, the ITM Monthly
Employees Association (ITM-MEA), to be affiliated with ANGLO;
2. That the management of ITM recognizes ANGLO as the sole and exclusive bargaining agent
of all the monthly-paid employees;
3. That an election of union officers shall be held on 26 May l986, from 8:00 a.m. to 5:00 p.m.;
4. That the last day of filing of candidacy shall be on l9 May l986 at 4:00 p.m.;
5. That a final pre-election conference to finalize the list of qualified voters shall be held on 19
May 1986, at 5:00 p.m.;
A pre-election conference was held, but the parties failed to agree on the list of voters. During
the May 21, 1986 pre-election conference attended by MOLE officers, ANGLO through its
National Secretary, a certain Mr. Cornelio A. Sy made a unilateral ruling excluding some 56
employees consisting of the Manila office employees, members of Iglesia ni Kristo, non-time
card employees, drivers of Mrs. Salazar and the cooperative employees of Mrs. Salazar. Prior to
the holding of the election of union officers petitioners, 2 through a letter addressed to the
Election Supervisor, MOLE San Fernando Pampanga, protested said ruling but no action was
taken. On May 26, 1986, the election of officers was conducted under the supervision of MOLE
wherein the 56 employees in question participated but whose votes were segregated without
being counted. Lacanilaos group won. Lacanilao garnered 119 votes with a margin of three (3)
votes over Tancinco prompting petitioners to make a protest. Thereafter, petitioners filed a
formal protest with the Ministry of Labor Regional Office in San Fernando, Pampanga 3
claiming that the determination of the qualification of the 56 votes is beyond the competence of
ANGLO. Private respondents maintain the contrary on the premise that definition of unions
membership is solely within their jurisdiction.
On the basis of the position papers submitted by the parties MOLEs Med Arbiter 4 issued an
order dated July 25, 1986 directing the opening and counting of the segregated votes. 5 From the
said order private respondents appealed to the Bureau of Labor Relations (BLR) justifying the
disenfranchisement of the 56 votes. Private respondents categorized the challenged voters into
four groups namely, the Manila Employees, that they are personal employees of Mr. Lee; the
Iglesia ni Kristo, that allowing them to vote will be anomalous since it is their policy not to
participate in any form of union activities; the non-time card employees, that they are managerial
employees; and the employees of the cooperative as non-ITM employees. 6 On December 10,
1986, BLR rendered a decision 7 holding the exclusion of the 56 employees as arbitrary,
whimsical, and wanting in legal basis 8 but set aside the challenged order of July 26, 1986 on the
ground that 51 ** of 56 challenged voters were not yet union members at the time of the election
per April 24, 1986 list submitted before the Bureau. 9 The decision directed among others the
proclamation of Lacanilaos group as the duly elected officers and for ITM-MEA to absorb in the
bargaining unit the challenged voters unless proven to be managerial employees. 10 Petitioners
motion for reconsideration was likewise denied.
Dissatisfied with the turn of events narrated above petitioners elevated the case to this Court by
way of the instant petition for certiorari under Rule 65 of the Rules of Court.

Petitioners allege that public respondent director of Labor Relations committed grave abuse of
discretion in ordering the Med-Arbiter to disregard the 56 segregated votes and proclaim private
respondents as the duly elected officers of ITM-MEA whereas said respondent ruled that the
grounds relied upon by ANGLO for the exclusion of voters are arbitrary, whimsical and without
legal basis.
ISSUE:
Whether the order of segregation (exclusion) of 56 votes of the petitioners union by the
respondent is tantamount to grave abuse of discretion.
HELD:
The petition is impressed with merit. The record of the case shows that public respondent
categorically declared as arbitrary, whimsical and without legal basis the grounds 11 relied upon
by ANGLO in disenfranchising the 56 voters in question. However, despite said finding public
respondent ruled to set aside the Resolution of July 25, 1986 of the Med-Arbiter based on its own
findings 12 that 51 of the 56 disenfranchised voters were not yet union members at the time of
the election of union officers on May 26, 1986 on the ground that their names do not appear in
the records of the Union submitted to the Labor Organization Division of the Bureau of Labor on
April 24, 1986.
The finding does not have a leg to stand on. Submission of the employees names with the BLR
as qualified members of the union is not a condition sine qua non to enable said members to vote
in the election of unions officers. It finds no support in fact and in law. Per public respondents
findings, the April 24, 1986 list consists of 158 union members only 13 wherein 51 of the 56
challenged voters names do not appear. Adopting however a rough estimate of a total number of
union members who cast their votes of some 333 14 and excluding therefrom the 56 challenged
votes, if the list is to be the basis as to who the union members are then public respondent should
have also disqualified some 175 of the 333 voters. It is true that under article 242(c) of the Labor
Code, as amended, only members of the union can participate in the election of union officers.
The question however of eligibility to vote may be determined through the use of the applicable
payroll period and employees status during the applicable payroll period. The payroll of the
month next preceding the labor dispute in case of regular employees 15 and the payroll period at
or near the peak of operations in case of employees in seasonal industries.
In the case before Us, considering that none of the parties insisted on the use of the payroll
period-list as voting list and considering further that the 51 remaining employees were correctly
ruled to be qualified for membership, their act of joining the election by casting their votes on
May 26, 1986 after the May 10, 1986 agreement is a clear manifestation of their intention to join
the union. They must therefore be considered ipso facto members thereof Said employees having
exercised their right to unionism by joining ITM-MEA their decision is paramount. Their names
could not have been included in the list of employee submitted on April 24, 1986 to the Bureau
of Labor for the agreement to join the union was entered into only on May 10, 1986. Indeed the
election was supervised by the Department of Labor where said 56 members were allowed to
vote. Private respondents never challenged their right to vote then.

The Solicitor General in his manifestation agreed with petitioners that public respondent
committed a grave abuse of discretion in deciding the issue on the basis of the records of
membership of the union as of April 24, 1986 when this issue was not put forward in the appeal.
WHEREFORE, premises considered, the petition for certiorari is GRANTED. The temporary
restraining order issued by this Court is hereby made permanent. The questioned Resolution and
the Decision are hereby set aside for being null and void and the Order of the Mediator Arbiter is
hereby declared immediately executory.
Cost against private respondents.
From Atty. Mindalano^^
Posted November 13, 2013 by vbdiaz in LABOR LAW

SUGBUANON VS. LAGUESMA Leave a comment


G.R. No. 116194 February 2, 2000
SUGBUANON RURAL BANK, INC. v. HON. UNDERSECRETARY BIENVENIDO E.
LAGUESMA, DEPARTMENT OF LABOR AND EMPLOYMENT, MED-ARBITER
ACHILLES MANIT, DEPARTMENT OF LABOR AND EMPLOYMENT, REGIONAL
OFFICE NO. 7, CEBU CITY, AND SUGBUANON RURAL BANK, INC.
ASSOCIATION OF PROFESSIONAL, SUPERVISORY, OFFICE, AND TECHNICAL
EMPLOYEES UNION-TRADE UNIONS CONGRESS OF THE PHILIPPINES
FACTS: Petitioner Sugbuanon Rural Bank, Inc., (SRBI, for brevity) is a duly-registered banking
institution with principal office in Cebu City and a branch in Mandaue City. Private respondent
SRBI Association of Professional, Supervisory, Office, and Technical Employees Union
(APSOTEU) is a legitimate labor organization affiliated with the Trade Unions Congress of the
Philippines (TUCP).1wphi1.nt
On October 8, 1993, the DOLE Regional Office in Cebu City granted Certificate of Registration
No. R0700-9310-UR-0064 to APSOTEU-TUCP, hereafter referred to as the union.
On October 26, 1993, the union filed a petition for certification election of the supervisory
employees of SRBI. It alleged, among others, that: (1) APSOTEU-TUCP was a labor
organization duly-registered with the Labor Department; (2) SRBI employed 5 or more
supervisory employees; (3) a majority of these employees supported the petition: (4) there was
no existing collective bargaining agreement (CBA) between any union and SRBI; and (5) no
certification election had been held in SRBI during the past 12 months prior to the petition.
On October 28, 1993, the Med-Arbiter gave due course to the petition. The pre-certification
election conference between SRBI and APSOTEU-TUCP was set for November 15, 1993.
On November 12, 1993, SRBI filed a motion to dismiss the unions petition. It sought to prevent
the holding of a certification election on two grounds. First, that the members of APSOTEUTUCP were in fact managerial or confidential employees.

ISSUES:
(1) Whether or not the members of the respondent union are managerial employees and/or
highly-placed confidential employees, hence prohibited by law from joining labor organizations
and engaging in union activities.
(2) Whether or not the Med-Arbiter may validly order the holding of a certification election upon
the filing of a petition for certification election by a registered union, despite the petitioners
appeal pending before the DOLE Secretary against the issuance of the unions registration.
RULING:
(1) Petitioners explanation does not state who among the employees has access to information
specifically relating to its labor to relations policies. Even Cashier Patricia Maluya, who serves
as the secretary of the banks Board of Directors may not be so classified.
Confidential employees are those who (1) assist or act in a confidential capacity, in regard (2) to
persons who formulate, determine, and effectuate management policies [specifically in the field
of labor relations].9 The two criteria are cumulative, and both must be met if an employee is to
be considered a confidential employee that is, the confidential relationship must exist between
the employee and his superior officer; and that officer must handle the prescribed responsibilities
relating to labor relations.
Art. 245 of the Labor Code does not directly prohibit confidential employees from engaging in
union activities. However, under the doctrine of necessary implication, the disqualification of
managerial employees equally applies to confidential employees. The confidential-employee rule
justifies exclusion of confidential employees because in the normal course of their duties they
become aware of management policies relating to labor relations. It must be stressed, however,
that when the employee does not have access to confidential labor relations information, there is
no legal prohibition against confidential employees from forming, assisting, or joining a union.
(2) One of the rights of a legitimate labor organization under Article 242(b) of the Labor Code is
the right to be certified as the exclusive representative of all employees in an appropriate
bargaining unit for purposes of collective bargaining. Having complied with the requirements of
Art. 234, it is our view that respondent union is a legitimate labor union. Article 257 of the Labor
Code mandates that a certification election shall automatically be conducted by the Med-Arbiter
upon the filing of a petition by a legitimate labor organization.16 Nothing is said therein that
prohibits such automatic conduct of the certification election if the management appeals on the
issue of the validity of the unions registration. On this score, petitioners appeal was correctly
dismissed.
From Atty. Soriano^^
Posted November 13, 2013 by vbdiaz in LABOR LAW

SAN MIGUEL UNION VS. LAGUESMA Leave a comment

G.R. No. 110399 August 15, 1997


SAN MIGUEL CORPORATION SUPERVISORS AND EXEMPT UNION AND
ERNESTO L. PONCE, President V. HONORABLE BIENVENIDO E. LAGUESMA IN
HIS CAPACITY AS UNDERSECRETARY OF LABOR AND EMPLOYMENT,
HONORABLE DANILO L. REYNANTE IN HIS CAPACITY AS MED-ARBITER AND
SAN MIGUEL CORPORATION
FACTS: Petitioner union filed before DOLE a Petition for Direct Certification or Certification
Election among the supervisors and exempt employees of the SMC Magnolia Poultry Products
Plants of Cabuyao, San Fernando and Otis.
Med-Arbiter Danilo L. Reynante issued an Order ordering the conduct of certification election
among the abovementioned employees of the different plants as one bargaining unit.
San Miguel Corporation filed a Notice of Appeal with Memorandum on Appeal, pointing out,
among others, the Med-Arbiters error in grouping together all three (3) separate plants, into one
bargaining unit, and in including supervisory levels 3 and above whose positions are confidential
in nature.
The public respondent, Undersecretary Laguesma, granted respondent companys Appeal and
ordered the remand of the case to the Med-Arbiter of origin for determination of the true
classification of each of the employees sought to be included in the appropriate bargaining unit.
Upon petitioner-unions motion, Undersecretary Laguesma granted the reconsideration prayed
for and directed the conduct of separate certification elections among the supervisors ranked as
supervisory levels 1 to 4 (S1 to S4) and the exempt employees in each of the three plants at
Cabuyao, San Fernando and Otis.
ISSUE:
1. Whether Supervisory employees 3 and 4 and the exempt employees of the company are
considered confidential employees, hence ineligible from joining a union.
2. If they are not confidential employees, do the employees of the three plants constitute an
appropriate single bargaining unit.
RULING:
(1) On the first issue, this Court rules that said employees do not fall within the term
confidential employees who may be prohibited from joining a union.
They are not qualified to be classified as managerial employees who, under Article 245 of the
Labor Code, are not eligible to join, assist or form any labor organization. In the very same
provision, they are not allowed membership in a labor organization of the rank-and-file
employees but may join, assist or form separate labor organizations of their own.

Confidential employees are those who (1) assist or act in a confidential capacity, (2) to persons
who formulate, determine, and effectuate management policies in the field of labor relations. The
two criteria are cumulative, and both must be met if an employee is to be considered a
confidential employee that is, the confidential relationship must exist between the employee
and his supervisor, and the supervisor must handle the prescribed responsibilities relating to
labor relations.
The exclusion from bargaining units of employees who, in the normal course of their duties,
become aware of management policies relating to labor relations is a principal objective sought
to be accomplished by the confidential employee rule. The broad rationale behind this rule is
that employees should not be placed in a position involving a potential conflict of interests.
Management should not be required to handle labor relations matters through employees who
are represented by the union with which the company is required to deal and who in the normal
performance of their duties may obtain advance information of the companys position with
regard to contract negotiations, the disposition of grievances, or other labor relations matters.
The Court held that if these managerial employees would belong to or be affiliated with a
Union, the latter might not be assured of their loyalty to the Union in view of evident conflict of
interest. The Union can also become company-dominated with the presence of managerial
employees in Union membership.
An important element of the confidential employee rule is the employees need to use labor
relations information. Thus, in determining the confidentiality of certain employees, a key
question frequently considered is the employees necessary access to confidential labor relations
information.
(2) The fact that the three plants are located in three different places, namely, in Cabuyao,
Laguna, in Otis, Pandacan, Metro Manila, and in San Fernando, Pampanga is immaterial.
Geographical location can be completely disregarded if the communal or mutual interests of the
employees are not sacrificed.
An appropriate bargaining unit may be defined as a group of employees of a given employer,
comprised of all or less than all of the entire body of employees, which the collective interest of
all the employees, consistent with equity to the employer, indicate to be best suited to serve the
reciprocal rights and duties of the parties under the collective bargaining provisions of the law.
A unit to be appropriate must effect a grouping of employees who have substantial, mutual
interests in wages, hours, working conditions and other subjects of collective bargaining.
From Atty. Soriano^^
Posted November 13, 2013 by vbdiaz in LABOR LAW

SS VENTURES VS. SS VENTURES UNION Leave a


comment

S.S. VENTURES INTERNATIONAL, INC., PETITIONER, VS. S.S. VENTURES LABOR


UNION (SSVLU) AND DIR. HANS LEO CACDAC, IN HIS CAPACITY AS DIRECTOR
OF THE BUREAU OF LABOR RELATIONS (BLR), RESPONDENTS.
G.R. No. 161690, July 23, 2008
FACTS:
Petitioner S.S. Ventures International, Inc. (Ventures), a PEZA- registered export firm with
principal place of business at Phase I-PEZA- Bataan Export Zone, Mariveles, Bataan, is in the
business of manufacturing sports shoes. Respondent S.S. Ventures Labor Union (Union) is a
labor organization registered with the DOLE.
March 21, 2000, the Union filed with DOLE-Region III a petition for certification election in
behalf of the rank-and-file employees
August 21, 2000, Ventures filed a Petition to cancel the Unions certificate of registration
alleging that the Union deliberately and maliciously included the names of more or less 82
former employees no longer connected with Ventures in its list of members who attended the
organizational meeting and in the adoption/ratification of its constitution and by-laws; that No
organizational meeting and ratification actually took place; and the Unions application for
registration was not supported by at least 20% of the rank-and-file employees of Ventures.
Regional Director of DOLE- Region III favored Ventures and resolved to Cancel the Certificate
of the union. On appeal, the BLR Director granted the Unions appeal and reversing the decision
of RD. Ventures went to the CA. The CA dismissed Ventures petition as well as the MR. Hence,
this petition for review
ISSUE:
Whether the registration of the Union must be cancelled.
RULING:
NO. The right to form, join, or assist a union is specifically protected by Art. XIII, Section 3 of
the Constitution and such right, according to Art. III, Sec. 8 of the Constitution and Art. 246 of
the Labor Code, shall not be abridged. Once registered with the DOLE, a union is considered a
legitimate labor organization endowed with the right and privileges granted by law to such
organization. While a certificate of registration confers a union with legitimacy with the
concomitant right to participate in or ask for certification election in a bargaining unit, the
registration may be canceled or the union may be decertified as the bargaining unit, in which
case the union is divested of the status of a legitimate labor organization. Among the grounds for
cancellation is the commission of any of the acts enumerated in Art. 239(a) of the Labor Code,
such as fraud and misrepresentation in connection with the adoption or ratification of the unions
constitution and like documents. The Court, has in previous cases, said that to decertify a union,
it is not enough to show that the union includes ineligible employees in its membership. It must
also be shown that there was misrepresentation, false statement, or fraud in connection with the
application for registration and the supporting documents, such as the adoption or ratification of

the constitution and by-laws or amendments thereto and the minutes of ratification of the
constitution or by-laws, among other documents.
The evidence presented by Ventures consist mostly of separate hand-written statements of 82
employees who alleged that they were unwilling or harassed signatories to the attendance sheet
of the organizational meeting. However these evidence was presented seven months after the
union filed its petition for cancellation of registration. Hence these statements partake of the
nature of withdrawal of union membership executed after the Unions filing of a petition for
certification election on March 21, 2000. We have said that the employees withdrawal from a
labor union made before the filing of the petition for certification election is presumed voluntary,
while withdrawal after the filing of such petition is considered to be involuntary and does not
affect the same. Now then, if a withdrawal from union membership done after a petition for
certification election has been filed does not vitiate such petition, it is but logical to assume that
such withdrawal cannot work to nullify the registration of the union. The Court is inclined to
agree with the CA that the BLR did not abuse its discretion nor gravely err when it concluded
that the affidavits of retraction of the 82 members had no evidentiary weight.
The registration or the recognition of a labor union after it has submitted the corresponding
papers is not ministerial on the part of the BLR. It becomes mandatory for the BLR to check if
the requirements under Art. 234 of the Labor Code have been sedulously complied with. If the
unions application is infected by falsification and like serious irregularities, especially those
appearing on the face of the application and its attachments, a union should be denied
recognition as a legitimate labor organization. The issuance to the Union of Certificate of
Registration, in the case at bar, necessarily implies that its application for registration and the
supporting documents thereof are prima facie free from any vitiating irregularities.
The relevance of the 82 individuals active participation in the Unions organizational meeting
and the signing ceremonies thereafter comes in only for purposes of determining whether or not
the Union, even without the 82, would still meet what Art. 234(c) of the Labor Code requires to
be submitted, requiring that the union applicant must file the names of all its members
comprising at least twenty percent (20%) of all the employees in the bargaining unit where it
seeks to operate.
In its union records on file with this Bureau, respondent union submitted the names of 542
members. This number easily complied with the 20% requirement, be it 1,928 or 2,202
employees in the establishment. Even subtracting the 82 employees from 542 leaves 460 union
members, still within 440 or 20% of the maximum total of 2,202 rank-and-file employees of the
employer Venture.
Whatever misgivings the petitioner may have with regard to the 82 dismissed employees is better
addressed in the inclusion-exclusion proceedings during a pre-election conference. The issue
surrounding the involvement of the 82 employees is a matter of membership or voter eligibility.
It is not a ground to cancel union registration.

For fraud and misrepresentation to be grounds for cancellation of union registration under Article
239, the nature of the fraud and misrepresentation must be grave and compelling enough to
vitiate the consent of a majority of union members.
WHEREFORE, the petition is DENIED.
From Atty. Daan^^
Posted November 13, 2013 by vbdiaz in LABOR LAW

WARREN VS. BLR Leave a comment


WARREN MANUFACTURING WORKERS UNION (WMWU), petitioner, vs. THE
BUREAU OF LABOR RELATIONS; PHILIPPINE AGRICULTURAL, COMMERCIAL
AND INDUSTRIAL WORKERS UNION (PACIWU); and SAMAHANG MANGGAGAWA
SA WARREN MANUFACTURING CORP.-ALLIANCE OF NATIONALIST AND
GENUINE LABOR ORGANIZATIONS (SMWMC-ANGLO), respondents.,
G.R. No. L-76185, Mar 30, 1988
FACTS:
On June 13, 1985, Philippine Agricultural, Commercial and Industrial Workers Union
(PACIWU) filed a petition for certification election, alleging compliance with the jurisdictional
requirements.
On July 7, 1985, petitioner thru counsel filed a motion to dismiss the petition on the ground that
there exists a C.B.A. between the respondent and the Warren Mfg. Union which took effect upon
its signing on July 16, 1985 and to expire on July 31, 1986.
While the petition was under hearing, PACIWU filed a Notice of Strike and on conciliation
meeting, a Return-to-Work Agreement was signed and stipulating that to resolve the issue of
union representation at Warren Mfg. Corp., parties have agreed to the holding of a consent
election among the rank and file on August 25, 1985 at the premises of the company to be
supervised by MOLE. It is clearly understood that the certified union in the said projected
election shall respect and administer the existing CBA at the company until its expiry date on
July 31, 1986.
On 12 August 1985, an Order was issued by BLR, directing that a consent election be held
among the rank and file workers of the company, with the following contending unions:
1. Philippine Agricultural, Commercial and Industrial Workers Union (PACIWU);
2. Warren Mfg. Workers Union (WMWU);
3. No Union.
On August 25, 1985, said consent election was held, and yielded the following results:

PACIWU- 94
WMWU-193
On June 5, 1986, the PACIWU filed a petition for certification election followed by the filing of
a petition for the same purposes by the Samahan ng Manggagawa sa Warren Manufacturing
Corporation-Alliance of Nationalist and Genuine Labor Organizations (Anglo for short) which
petitions were both opposed by Warren Manufacturing Corporation on the grounds that neither
petition has 30% support; that both are barred by the one-year no certification election law and
the existence of a duly ratified CBA.
The Med-Arbiter-NCR, MOLE ordered on August 18, 1986 the holding of a certification election
within 20 days from receipt to determine the exclusive bargaining representative of all the rank
and file employees of the Warren Manufacturing Corporation, with the choices:
1. Philippine Agricultural, Commercial and Industrial Workers Union (PACIWU);
2. Warren Mfg. Workers Union;
3. Samahan ng Manggagawa sa Warren Mfg. Corporation-ANGLO; and
4. No Union.
Both Warren Manufacturing Corporation and petitioner filed separate motions treated as appeals
by the BLR which was dismissed. Hence, this petition solely by WMWU. Petitioner invoked the
one-year no certification election rule and the principle of the Contract Bar Rule.
ISSUE:
Whether the Certification Election should be granted.
RULING:
YES. The records show that petitioner admitted that what was held on August 25, 1985 at the
Companys premises and which became the root of this controversy, was a consent election and
not a certification election. A consent election is an agreed one, its purpose being merely to
determine the issue of majority representation of all the workers in the appropriate collective
bargaining unit while a certification election is aimed at determining the sole and exclusive
bargaining agent of all the employees in an appropriate bargaining unit for the purpose of
collective bargaining. From the very nature of consent election, it is a separate and distinct
process and has nothing to do with the import and effect of a certification election. Neither does
it shorten the terms of an existing CBA nor entitle the participants thereof to immediately
renegotiate an existing CBA although it does not preclude the workers from exercising their right
to choose their sole and exclusive bargaining representative after the expiration of the 60 day
freedom period.
The election held on August 25, 1985 was not for the purpose of determining which labor union
should be the bargaining representative in the negotiation for a collective contract, there being an
existing collective bargaining agreement yet to expire on July 31, 1986; but only to determine
which labor union shall administer the said existing contract.

Article 257 of the New Labor Code expressly states that No certification election issue shall be
entertained if a collective agreement which has been submitted in accordance with Article 231 of
the Code exists between the employer and a legitimate labor organization except within 60 days
prior to the expiration of the life of such certified collective bargaining agreement. There should
be no obstacle to the right of the employees to petition for a certification election at the proper
time.
As aforestated, the existing collective bargaining agreement was due to expire on July 31, 1986.
The Med-Arbiter found that the petition and intervention were supported by more than 30% of
the members of the bargaining unit. Because of this, Article 258 of the Labor Code makes it
mandatory for the BLR to conduct a certification election. Once it has been verified that the
petition for certification election has the support of at least 30% of the employees in the
bargaining unit, it must be granted. It becomes under the circumstances, mandatory .
WHEREFORE, the instant Petition is DISMISSED.
From Atty. Daan^^
Posted November 13, 2013 by vbdiaz in LABOR LAW

HOLY CROSS VS. JOAQUIN Leave a comment


G.R. No. 110007 October 18, 1996
HOLY CROSS OF DAVAO COLLEGE, INC., petitioner,
vs.
HON. JEROME JOAQUIN, in his capacity as Voluntary Arbitrator, and HOLY CROSS
OF DAVAO COLLEGE UNION-KALIPUNAN NG MANGGAGAWANG PILIPINO
(KAMAPI), respondents.
FACTS: A collective bargaining agreement, effective from June 1, 1986 to May 31, 1989 was
entered into between petitioner Holy Cross of Davao College, Inc. (hereafter Holy Cross), an
educational institution, and the affiliate labor organization representing its employees,
respondent Holy Cross of Davao College Union-KAMAPI (hereafter KAMAPI).
Shortly before the expiration of the agreement, KAMAPI President, Jose Lagahit, wrote Holy
Cross under date of April 12, 1989 expressing his unions desire to renew the agreement, withal
seeking its extension for two months, or until July 31, 1989. Granted.
Thereafter an election of officers was held, and at which Rodolfo Gallera won election as
president. To the surprise of many, and with resultant dissension among the membership, Gallera
forthwith initiated discussions for the unions disaffiliation from the KAMAPI Federation.
Galleras group subsequently formed a separate organization known as the Holy Cross of Davao
College Teachers Union, and elected its own officers.

For its part, the existing union, KAMAPI, sent to the School its proposals for a new collective
bargaining contract; this it did on July 31, 1989, the expiry date of the two-month extension it
had sought.
Holy Cross thereafter stopped deducting from the salaries and wages of its teachers and
employees the corresponding union dues and special assessments (payable by union members),
and agency fees (payable by non-members), in accordance with the check-off clause of the CBA,
4 prompting KAMAPI, on September 1, 1989, to demand an explanation.
* In the meantime, there ensued between the two unions (KAMAPI vs. GALLERA GROUP) a
full-blown action on the basic issue of representation, which was to last for some two years.
KAMAPI won.
After its success in the certification election case KAMAPI presented a revised bargaining
proposals to Holy Cross; asking for Holy Cross counter-proposal. Holy cross refused to submit
its counter-proposal (based on many many grounds not relative to the topic in our syllabus ).
* Several conciliation meetings were thereafter held between them, and when these failed to
bring about any amicable settlement, the parties agreed to submit the case to voluntary
arbitration. On both issues, Voluntary Arbitrator Jerome C. Joaquin found in favor of KAMAPI.
Petition by Holy Cross with the SC, denied.
ISSUE: Whether or not an employer is liable to pay to the union of its employees, the amounts it
failed to deduct from their salaries as union dues (with respect to union members) or agency
fees (as regards those not union members) in accordance with the check-off provisions of the
collective bargaining contract (CBA) which it claims to have been automatically extended.
HELD: NO
A check-off is a process or device whereby the employer, on agreement with the union
recognized as the proper bargaining representative, or on prior authorization from its employees,
deducts union dues or agency fees from the latters wages and remits them directly to the union.
Indeed, this Court has acknowledged that the system of check-off is primarily for the benefit of
the union and, only indirectly, of the individual laborers.
No provision of law makes the employer directly liable for the payment to the labor organization
of union dues and assessments that the former fails to deduct from its employees salaries and
wages pursuant to a check-off stipulation. The employers failure to make the requisite
deductions may constitute a violation of a contractual commitment for which it may incur
liability for unfair labor practice. 23 But it does not by that omission, incur liability to the union
for the aggregate of dues or assessments uncollected from the union members, or agency fees for
non-union employees.
The obligation to pay union dues and agency fees obviously devolves not upon the employer, but
the individual employee. It is a personal obligation not demandable from the employer upon

default or refusal of the employee to consent to a check-off. The only obligation of the employer
under a check-off is to effect the deductions and remit the collections to the union.
The principle of unjust enrichment necessarily precludes recovery of union dues or agency
fees from the employer, these being, to repeat, obligations pertaining to the individual worker
in favor of the bargaining union. Where the employer fails or refuses to implement a check-off
agreement, logic and prudence dictate that the union itself undertake the collection of union dues
and assessments from its members (and agency fees from non-union employees); this, of course,
without prejudice to suing the employer for unfair labor practice.
PETITION GRANTED.
_______________
NOTES:
Issues presented by them for resolution to the voluntary arbitrator:
a. Whether or not the CBA which expired on May 31, 1989 was automatically renewed and did
not serve merely as a holdover CBA; and
b. Whether or not there was refusal to negotiate on the part of the Holy Cross of Davao College.
The legal basis of check-off is thus found in statute or in contract. 19 Statutory limitations on
check-offs generally require written authorization from each employee to deduct wages;
however, a resolution approved and adopted by a majority to the union members at a general
meeting will suffice when the right to check-off has been recognized by the employer, including
collection of reasonable assessments in connection with mandatory activities of the union, or
other special assessments and extraordinary fees.
Authorization to effect a check-off of union dues is co-terminous with the union affiliation or
membership of employees.
On the other hand, the collection of agency fees in an amount equivalent to union dues and
fees, from employees who are not union members, is recognized by Article 248 (e) of the Labor
Code. No requirement of written authorization from the non-union employee is imposed. The
employees acceptance of benefits resulting from a collective bargaining agreement justifies the
deduction of agency fees from his pay and the unions entitlement thereto. In this aspect, the
legal basis of the unions right to agency fees is neither contractual nor statutory, but quasicontractual, deriving from the established principle that non-union employees may not unjustly
enrich themselves by benefiting from employment conditions negotiated by the bargaining
union.
From Atty. Bayani^^
Posted November 13, 2013 by vbdiaz in LABOR LAW

GABRIEL VS. SECRETARY OF LABOR Leave a


comment
G.R. No. 115949 March 16, 2000
EVANGELINE J. GABRIEL, TERESITA C. LUALHATI, EVELYN SIA, RODOLFO
EUGENIO, ISAGANI MAKISIG, and DEMETRIO SALAS, petitioners,
vs.
THE HONORABLE SECRETARY OF LABOR AND EMPLOYMENT and SIMEON
SARMIENTO et. al (AND ALL OTHER SOLID BANK UNION MEMBERS)
FACTS: Petitioners comprise the Executive Board of the SolidBank Union, the duly recognized
collective bargaining agent for the rank and file employees of Solid Bank Corporation. Private
respondents are members of said union.
The unions Executive Board decided to retain anew the service of Atty. Ignacio P. Lacsina (now
deceased) as union counsel in connection with the negotiations for a new Collective Bargaining
Agreement (CBA); majority of all union members approved and signed a resolution confirming
the decision of the executive board to engage the services of Atty. Lacsina as union counsel.
As approved, the resolution provided that ten percent (10%) of the total economic benefits that
may be secured through the negotiations be given to Atty. Lacsina as attorneys fees. It also
contained an authorization for SolidBank Corporation to check-off said attorneys fees from the
first lump sum payment of benefits to the employees under the new CBA and to turn over said
amount to Atty. Lacsina and/or his duly authorized representative.
The bank then, on request of the union, made payroll deductions for attorneys fees from the
CBA benefits paid to the union members in accordance with the abovementioned resolution.
Private respondents instituted a complaint against the petitioners and the union counsel before
the Department of Labor and Employment (DOLE) for illegal deduction of attorneys fees as
well as for quantification of the benefits in the 1992 CBA.
Med-arbiter granted the complaint; Secretary partially granted and the Order of the Med-Arbiter
dated 22 April 1993 is hereby modified as follows: (1) that the ordered refund shall be limited to
those union members who have not signified their conformity to the check-off of attorneys fees;
and (2) the directive on the payment of 5% attorneys fees should be deleted for lack of basis.
Hence, this petition.
ISSUE: WON the deductions made by petioner-company is valid.
HELD: NO
Private respondents contention: claim that the check-off provision in question is illegal because
it was never submitted for approval at a general membership meeting called for the purpose and
that it failed to meet the formalities mandated by the Labor Code.

In check-off, the employer, on agreement with the Union, or on prior authorization from
employees, deducts union dues or agency fees from the latters wages and remits them directly to
the union. It assures continuous funding; for the labor organization. As this Court has
acknowledged, the system of check-off is primarily for the benefit of the union and only
indirectly for the individual employees.
The pertinent legal provisions on check-offs are found in Article 222 (b) and Article 241 (o) of
the Labor Code.
Art. 222 (b) states:
No attorneys fees, negotiation fees or similar charges of any kind arising from any collective
bargaining negotiations or conclusions of the collective agreement shall be imposed on any
individual member of the contracting union: Provided, however, that attorneys fees may be
charged against unions funds in an amount to be agreed upon by the parties. Any contract,
agreement or arrangement of any sort to the contrary shall be null and void. (Emphasis ours)
Art. 241 (o) provides:
Other than for mandatory activities under the Code, no special assessment, attorneys fees,
negotiation fees or any other extraordinary fees may be checked off from any amount due to an
employee without an individual written authorization duly signed by the employee. The
authorization should specifically state the amount, purpose and beneficiary of the deduction.
(Emphasis ours).
Art. 241 has three (3) requisites for the validity of the special assessment for unions incidental
expenses, attorneys fees and representation expenses. These are: 1) authorization by a written
resolution of the majority of all the members at the general membership meeting called for the
purpose; (2) secretarys record of the minutes of the meeting; and (3) individual written
authorization for check off duly signed by the employees concerned.
Clearly, attorneys fees may not be deducted or checked off from any amount due to an employee
without his written consent.
After a thorough review of the records, we find that the General Membership Resolution of
October 19, 1991 of the SolidBank Union did not satisfy the requirements laid down by law and
jurisprudence for the validity of the ten percent (10%) special assessment for unions incidental
expenses, attorneys fees and representation expenses. There were no individual written check
off authorizations by the employees concerned and so the assessment cannot be legally deducted
by their employer.
From all the foregoing, we are of the considered view that public respondent did not act with
grave abuse of discretion in ruling that the workers through their union should be made to
shoulder the expenses incurred for the services of a lawyer. And accordingly the reimbursement
should be charged to the unions general fund or account. No deduction can be made from the
salaries of the concerned employees other than those mandated by law.
Petition is DENIED.

From Atty. Bayani^^


Posted November 13, 2013 by vbdiaz in LABOR LAW

VERCELES VS. DOLE Leave a comment


G.R. No. 152322 February 15, 2005
ERNESTO C. VERCELES, DIOSDADO F. TRINIDAD, SALVADOR G. BLANCIA,
ROSEMARIE DE LUMBAN, FELICITAS F. RAMOS, MIGUEL TEAO, JAIME
BAUTISTA and FIDEL ACERO, as Officers of the University of the East Employees
Association v. BUREAU OF LABOR RELATIONS-DEPARTMENT OF LABOR AND
EMPLOYMENT, DEPARTMENT OF LABOR AND EMPLOYMENT-NATIONAL
CAPITAL REGION, RODEL E. DALUPAN, EFREN J. DE OCAMPO, PROCESO
TOTTO, JR., ELIZABETH ALARCA, ELVIRA S. MANALO, and RICARDO UY
FACTS: The case arose from a memorandum filed by Petitoners against Private Respondent for
allegedly spreading false rumors and creating disinformation among the members of the said
association.
The rumors, according to Petitioners happened when Private respondents, in filing a complaint
before the DOLE-NCR complained of petitioners refusal to render financial and other reports,
and deliberate refusal to call general and special meetings. According to the findings of CA, the
financial statements for the years 1995 up to 1997 were submitted to DOLE-NCR only on 06
February 1998 while that for the year 1998 was submitted only on 16 March 1999. The last
associations meeting was conducted on 21 April 1995, and the copy of the minutes thereon was
submitted to BLR-DOLE only on 24 February 1998.
Petitioners do not hide the fact that they belatedly submitted their financial reports and the
minutes of their meetings to the DOLE.
Petitioners Contention: The issue of belatedly submitting these reports, according to the
petitioners, had been rendered moot and academic by their eventual compliance. Besides, this
has been the practice of the association. Moreover, the petitioners likewise maintain that the
passage of General Assembly Resolution No. 10 dated 10 December 1997 and Resolution No. 8,
Series of 2000, following the application of the principle that the sovereign majority rules, cured
any liability that may have been brought about by their belated actions.
ISSUE: Whether or not the non-holding of meetings and non-submission of reports by the
petitioners moot and academic, and whether the decision to hold meetings and submit reports
contradict and override the sovereign will of the majority?
RULING: We do not believe so.
This issue was precipitated by the Court of Appeals decision affirming the order of DOLE
Regional Director Maximo B. Lim for the petitioners to hold a general membership meeting
wherein they make open and available the unions/associations books of accounts and other

documents pertaining to the union funds, and to regularly conduct special and general
membership meetings in accordance with the unions constitution and by-laws.
The passage of General Assembly Resolution No. 10 dated 10 December 1997 and Resolution
No. 8, Series of 2000, which supposedly cured the lapses committed by the associations officers
and reiterated the approval of the general membership of the acts and collateral actions of the
associations officers cannot redeem the petitioners from their predicament. The obligation to
hold meetings and render financial reports is mandated by UEEAs constitution and by-laws.
This fact was never denied by the petitioners. Their eventual compliance, as what happened in
this case, shall not release them from the obligation to accomplish these things in the future.
Prompt compliance in rendering financial reports together with the holding of regular meetings
with the submission of the minutes thereon with the BLR-DOLE and DOLE-NCR shall negate
any suspicion of dishonesty on the part of UEEAs officers. This is not only true with UEEA, but
likewise with other unions/associations, as this matter is imbued with public interest.
Undeniably, transparency in the official undertakings of union officers will bolster genuine trade
unionism in the country.
From Atty. Daan^^
Posted November 13, 2013 by vbdiaz in LABOR LAW

St James School vs Samahang Manggagawa ng St James


Leave a comment
St James School vs Samahang Manggagawa ng St James
GR 151326
Facts:
Respondent Samahang Manggagawa filed a petition for a certification election to determine the
collective bargaining representative of the motorpool, construction and transportation employees
of St. James School of QC. The certification election was held at the DOLE office in which there
were 149 eligible voters. 84 cast their votes. St. James filed a protest challenging the 84 votes,
alleging that it had 179 rank and file employees and none of them voted in the certification
election. It argued that those who voted were not its regular employees but construction workers
of an independent contractor.
Issue:
1. WON the formation of the labor union was valid
2. WON the certification election was valid
Held:

1. Yes. The records show that prior to the holding of a certification election, St. James filed a
petition for cancellation of Samahang Manggagawa on the ground of lack of employer-employee
relations between the union members and St. James. This went all the way up to the CA which
ruled that the independent contractor is a labor only contractor, and thus, an agent of St. James
which is the real employer.
2. Yes. St. James has five campuses. The members of the Samahan are employees of the Tandang
Sora campus. The Samahan seeks to represent the motor pool, construction, and transportation
employees of the T. Sora campus. Thus, the computation of the quorum should be based on it,
not on the total number of employees in all five campuses of St. James.
from Atty. Alba^^
Posted November 6, 2013 by vbdiaz in LABOR LAW

Philippine Skylanders vs NLRC Leave a comment


Philippine Skylanders vs NLRC
GR 127374
Facts:
In November 1993 the Philippine Skylanders Employees Association (PSEA), a local labor
union affiliated with the Philippine Association of Free Labor Unions (PAFLU), won in the
certification election conducted among the rank and file employees of Philippine Skylanders,
Inc. (PSI). Its rival union, Philippine Skylanders Employees Association-WATU (PSEA-WATU)
immediately protested the result of the election before the Secretary of Labor.
Several months later, PSEA sent PAFLU a notice of disaffiliation.
PSEA subsequently affiliated itself with the National Congress of Workers (NCW), changed its
name to Philippine Skylanders Employees Association National Congress of Workers (PSEANCW), and to maintain continuity within the organization, allowed the former officers of PSEAPAFLU to continue occupying their positions as elected officers in the newly-formed PSEANCW.
On 17 March 1994 PSEA-NCW entered into a collective bargaining agreement with PSI which
was immediately registered with the Department of Labor and Employment.
Meanwhile, apparently oblivious to PSEAs shift of allegiance, PAFLU Secretary General
Serafin Ayroso wrote Mariles C. Romulo requesting a copy of PSIs audited financial statement.
On 30 July 1994 PSI through its personnel manager Francisco Dakila denied the request citing as
reason PSEAs disaffiliation from PAFLU and its subsequent affiliation with NCW.
Issue: WON PSEAs disaffiliation is legitimate.

Held:
At the outset, let it be noted that the issue of disaffiliation is an inter-union conflict the
jurisdiction of which properly lies with the Bureau of Labor Relations (BLR) and not with the
Labor Arbiter.
We upheld the right of local unions to separate from their mother federation on the ground that as
separate and voluntary associations, local unions do not owe their creation and existence to the
national federation to which they are affiliated but, instead, to the will of their members. Yet the
local unions remain the basic units of association, free to serve their own interests subject to the
restraints imposed by the constitution and by-laws of the national federation, and free also to
renounce the affiliation upon the terms laid down in the agreement which brought such affiliation
into existence.
There is nothing shown in the records nor is it claimed by PAFLU that the local union was
expressly forbidden to disaffiliate from the federation nor were there any conditions imposed for
a valid breakaway. As such, the pendency of an election protest involving both the mother
federation and the local union did not constitute a bar to a valid disaffiliation.
It was entirely reasonable then for PSI to enter into a collective bargaining agreement with
PSEA-NCW. As PSEA had validly severed itself from PAFLU, there would be no restrictions
which could validly hinder it from subsequently affiliating with NCW and entering into a
collective bargaining agreement in behalf of its members.
Policy considerations dictate that in weighing the claims of a local union as against those of a
national federation, those of the former must be preferred. Parenthetically though, the desires of
the mother federation to protect its locals are not altogether to be shunned. It will however be to
err greatly against the Constitution if the desires of the federation would be favored over those of
its members. That, at any rate, is the policy of the law. For if it were otherwise, instead of
protection, there would be disregard and neglect of the lowly workingmen.
from Atty. Alba^^
Posted November 6, 2013 by vbdiaz in LABOR LAW

Philippine Scout Veterans Security vs Torres Leave a


comment
Philippine Scout Veterans Security vs Torres
GR 92357
Facts:
On April 6, 1989, private respondent labor union, PGA Brotherhood Association Union of
Filipino Workers (UFW), hereinafter referred to as the Union filed a petition for Direct

Certification/Certification Election among the rank and file employees of Philippine Scout
Veterans Security and Investigation Agency (PSVSIA), GVM Security and Investigations
Agency, Inc. (GVM). and Abaquin Security and Detective Agency, Inc. (ASDA). These three
agencies were collectively referred to by private respondent Union as the PGA Security
Agency, which is actually the first letters of the corporate names of the agencies.
On April 11, 1989, summons was issued to the management of PSVSIA, GVM, ASDA (PGA
Security Agency) at 82 E. Rodriquez Avenue, Quezon City.
On April 11, 26, 1986, petitioners filed a single comment alleging therein that the said three
security agencies have separate and distinct corporate personalities while PGA Security Agency
is not a business or corporate entity and does not possess any personality whatsoever; the petition
was unclear as to whether the rank-and-file employees mentioned therein refer to those of the
three security agencies collectively and if so, the labor union cannot seek a certification election
in three separate bargaining units in one petition.
Issue: WON petitioners can interfere with the certification election proceeding.
Held:
Except where the employer has to file a petition for certification election pursuant to Article 258
of the Labor Code because of a request to bargain collectively, it has nothing to do with a
certification election which is the sole concern of the workers. Its role in a certification election
has aptly been described in Trade Unions of the Philippines and Allied Services (TUPAS) v.
Trajano, as that of a mere by-stander. It has no legal standing in a certification election as it
cannot oppose the petition or appeal the Med-Arbiters orders related thereto. An employer that
involves itself in a certification election lends suspicion to the fact that it wants to create a
company union.
This Courts disapprobation of management interference in certification elections is even more
forceful in Consolidated Farms, Inc. v. Noriel, where we held:
On a matter that should be the exclusive concern of labor, the choice of a collective bargaining
representative, the employer is definitely an intruder. His participation, to say the least, deserves
no encouragement. This Court should be the last agency to lend support to such an attempt at
interference with a purely internal affair of labor.
from Atty. Alba^^
Posted November 6, 2013 by vbdiaz in LABOR LAW

MERALCO vs SOLE Leave a comment


MERALCO vs Secretary of Labor
GR 91902

Facts:
On November 22, 1988, the Staff and Technical Employees Association of MERALCO, a labor
organization of staff and technical employees of MERALCO, filed a petition for certification
election, seeking to represent regular employees of MERALCO. MERALCO moved for the
dismissal of the petition on the grounds that the employees sought to be represented by petitioner
are security services personnel who are prohibited from joining or assisting the rank-and-file
union, among others.
Issue: WON security guards may join rank-and-file or supervisors union.
Held:
While therefore under the old rules, security guards were barred from joining a labor
organization of the rank and file, under RA 6715, they may now freely join a labor organization
of the rank and file or that of the supervisory union, depending on their rank.
We are aware however of possible consequences in the implementation of the law in allowing
security personnel to join labor unions within the company they serve. The law is apt to produce
divided loyalties in the faithful performance of their duties. Economic reasons would present the
employees concerned with the temptation to subordinate their duties to the allegiance they owe
the union of which they are members, aware as they are that it is usually union action that
obtains for them increased pecuniary benefits.
Thus, in the event of a strike declared by their union, security personnel may neglect or
outrightly abandon their duties, such as protection of property of their employer and the persons
of its officials and employees, the control of access to the employers premises, and the
maintenance of order in the event of emergencies and untoward incidents.
It is hoped that the corresponding amendatory and/or suppletory laws be passed by Congress to
avoid possible conflict of interest in security personnel.
Petition dismissed.
from Atty. Alba^^
Posted November 6, 2013 by vbdiaz in LABOR LAW

Cirtek Employees Labor Union vs Cirtek Electronics Leave


a comment
Cirtek Employees Labor Union vs Cirtek Electronics
GR 190515
Facts:

This resolves the motion for reconsideration and supplemental motion for reconsideration filed
by respondent, Cirtek Electronics, Inc., of the Courts Decision dated November 15, 2010.
Respondent-movant maintains that the Secretary of Labor cannot insist on a ruling beyond the
compromise agreement entered into by the parties; and that, as early as February 5, 2010,
petitioner Union had already filed with the Department of Labor and Employment (DOLE) a
resolution of disaffiliation from the Federation of Free Workers resulting in the latters lack of
personality to represent the workers in the present case.
Issue: WON petitioner lost its personality to represent the workers because of its disaffiliation
from the Federation of Free Workers.
Held:
The issue of disaffiliation is an intra-union dispute which must be resolved in a different forum
in an action at the instance of either or both the FFW and the Union or a rival labor organization,
not the employer.
Indeed, as respondent-movant itself argues, a local union may disaffiliate at any time from its
mother federation, absent any showing that the same is prohibited under its constitution or rule.
Such, however, does not result in it losing its legal personality altogether. Verily, Anglo-KMU v.
Samahan Ng Mga Manggagawang Nagkakaisa Sa Manila Bay Spinning Mills At J.P. Coats
enlightens:
A local labor union is a separate and distinct unit primarily designed to secure and maintain an
equality of bargaining power between the employer and their employee-members. A local union
does not owe its existence to the federation with which it is affiliated. It is a separate and distinct
voluntary association owing its creation to the will of its members. The mere act of affiliation
does not divest the local union of its own personality, neither does it give the mother federation
the license to act independently of the local union. It only gives rise to a contract of agency
where the former acts in representation of the latter. (emphasis and underscoring supplied)
MR denied.
from Atty. Alba^^
Posted November 6, 2013 by vbdiaz in LABOR LAW

Central Negros Electric Cooperative vs DOLE Leave a


comment
Central Negros Electric Cooperative vs DOLE
GR 94045
Facts:

On August 15, 1987, CENECO entered into a collective bargaining agreement with CURE, a
labor union representing its rank-and-file employees, providing for a term of three years
retroactive to April 1, 1987 and extending up to March 31, 1990. On December 28, 1989, CURE
wrote CENECO proposing that negotiations be conducted for a new collective bargaining
agreement (CBA).
On January 18, 1990, CENECO denied CUREs request on the ground that, under applicable
decisions of the Supreme Court, employees who at the same time are members of an electric
cooperative are not entitled to form or join a union.
Prior to the submission of the proposal for CBA renegotiation, CURE members, in a general
assembly held on December 9, 1989, approved Resolution No. 35 whereby it was agreed that
tall union members shall withdraw, retract, or recall the union members membership from
Central Negros Electric Cooperative, Inc. in order to avail (of) the full benefits under the existing
Collective Bargaining Agreement entered into by and between CENECO and CURE, and the
supposed benefits that our union may avail of under the renewed CBA.
However, the withdrawal from membership was denied by CENECO on February 27, 1990
under Resolution No. 90.
Issue: WON the employees of CENECO who withdrew their membership from the cooperative
are entitled to form or join CURE for purposes of the negotiations for a collective bargaining
agreement proposed by the latter.
Held:
The right of the employees to self-organization is a compelling reason why their withdrawal
from the cooperative must be allowed. As pointed out by CURE, the resignation of the memberemployees is an expression of their preference for union membership over that of membership in
the cooperative. The avowed policy of the State to afford fall protection to labor and to promote
the primacy of free collective bargaining mandates that the employees right to form and join
unions for purposes of collective bargaining be accorded the highest consideration.
Thus, member employees of a cooperative may withdraw as members of the cooperative in order
to join labor union. Membership in a cooperative is voluntary; inherent in it is the right not to
join.
NOTES: (San Jose Electric Service Cooperative vs. Ministry of Labor)
1. A cooperative, therefore, is by its nature different from an ordinary business concern being run
either, by persons, partnerships or corporations. Its owners and/or members are the ones who run
and operate the business while the others are its employees.
2. An employee therefore of such a cooperative who is a member and co-owner thereof cannot
invoke the right to collective bargaining for certainly an owner cannot bargain with himself or
his co-owners. Employees of cooperatives who are themselves members of the cooperative have

no right to form or join labor organizations for purposes of collective bargaining for being
themselves co-owners of the cooperative.
3. However, in so far as it involves cooperatives with employees who are not members or coowners thereof, certainly such employees are entitled to exercise the rights of all workers to
organization, collective bargaining, negotiations and others as are enshrined in the Constitution
and existing laws of the country.
Posted November 6, 2013 by vbdiaz in LABOR LAW

NATIONAL UNION OF WORKERS IN HOTELS,


RESTAURANTS AND ALLIED INDUSTRIES- MANILA
PAVILION HOTEL CHAPTER vs. SEC. OF LABOR
Leave a comment
G.R. No. 181531

July 31, 2009

NATIONAL UNION OF WORKERS IN HOTELS, RESTAURANTS AND ALLIED


INDUSTRIES- MANILA PAVILION HOTEL CHAPTER vs. SECRETARY OF LABOR
AND EMPLOYMENT, BUREAU OF LABOR RELATIONS, HOLIDAY INN MANILA
PAVILION HOTEL LABOR UNION AND ACESITE PHILIPPINES HOTEL
CORPORATION
FACTS: A certification election was conducted on June 16, 2006 among the rank-and-file
employees of respondent Holiday Inn Manila Pavilion Hotel (the Hotel) with the following
results:
EMPLOYEES IN VOTERS LIST =
TOTAL VOTES CAST =
NUWHRAIN-MPHC =
HIMPHLU =
NO UNION =
SPOILED =
SEGREGATED =

353
346
151
169
1
3
22

In view of the significant number of segregated votes, contending unions, petitioner,


NUHWHRAIN-MPHC, and respondent Holiday Inn Manila Pavillion Hotel Labor Union
(HIMPHLU), referred the case back to Med-Arbiter to decide which among those votes would
be opened and tallied. 11 votes were initially segregated because they were cast
by dismissed employees, albeit the legality of their dismissal was still pending before the Court
of Appeals. Six other votes were segregated because the employees who cast them were already
occupying supervisory positions at the time of the election. Still five other votes were segregated
on the ground that they were cast by probationary employees and, pursuant to the existing
Collective Bargaining Agreement (CBA), such employees cannot vote. It bears noting early on,

however, that the vote of one Jose Gatbonton (Gatbonton), a probationary employee, was
counted.
Med-Arbiter Calabocal ruled for the opening of 17 out of the 22 segregated votes, specially those
cast by the 11 dismissed employees and those cast by the six supposedly supervisory employees
of the Hotel.
Petitioner, which garnered 151 votes, appealed to the Secretary of Labor and Employment
(SOLE), arguing that the votes of the probationary employees should have been opened
considering that probationary employee Gatbontons vote was tallied. And petitioner averred that
respondent HIMPHLU, which garnered 169 votes, should not be immediately certified as the
bargaining agent, as the opening of the 17 segregated ballots would push the number of valid
votes cast to 338 (151 + 169 + 1 + 17), hence, the 169 votes which HIMPHLU garnered would
be one vote short of the majority which would then become 169.
Secretary affirmed the decision of the med-arbiter. In fine, the SOLE concluded that the
certification of HIMPHLU as the exclusive bargaining agent was proper.
ISSUES: (1) whether employees on probationary status at the time of the certification elections
should be allowed to vote (2) whether HIMPHLU was able to obtain the required majority for it
to be certified as the exclusive bargaining agent.
HELD:
I. On the first issue, the Court rules in the affirmative.
The inclusion of Gatbontons vote was proper not because it was not questioned but because
probationary employees have the right to vote in a certification election. The votes of the six
other probationary employees should thus also have been counted. As Airtime Specialists, Inc. v.
Ferrer-Calleja holds:
In a certification election, all rank and file employees in the appropriate bargaining unit, whether
probationary or permanent are entitled to vote. This principle is clearly stated in Art. 255 of the
Labor Code which states that the labor organization designated or selected by the majority of
the employees in an appropriate bargaining unit shall be the exclusive representative of the
employees in such unit for purposes of collective bargaining. Collective bargaining covers all
aspects of the employment relation and the resultant CBA negotiated by the certified union
binds all employees in the bargaining unit. Hence, all rank and file employees,
probationary or permanent, have a substantial interest in the selection of the bargaining
representative. The Code makes no distinction as to their employment status as basis for
eligibility in supporting the petition for certification election. The law refers to all the
employees in the bargaining unit. All they need to be eligible to support the petition is to
belong to the bargaining unit. (Emphasis supplied)

For purposes of this section (Rule II, Sec. 2 of Department Order No. 40-03, series of 2003),
any employee, whether employed for a definite period or not, shall beginning on the first
day of his/her service, be eligible for membership in any labor organization.
All other workers, including ambulant, intermittent and other workers, the self-employed, rural
workers and those without any definite employers may form labor organizations for their mutual
aid and protection and other legitimate purposes except collective bargaining. (Emphasis
supplied)
The provision in the CBA disqualifying probationary employees from voting cannot
override the Constitutionally-protected right of workers to self-organization, as well as the
provisions of the Labor Code and its Implementing Rules on certification elections and
jurisprudence thereon.
A law is read into, and forms part of, a contract. Provisions in a contract are valid only if they are
not contrary to law, morals, good customs, public order or public policy.
II. As to whether HIMPHLU should be certified as the exclusive bargaining agent, the
Court rules in the negative.
It is well-settled that under the so-called double majority rule, for there to be a valid
certification election, majority of the bargaining unit must have voted AND the winning
union must have garnered majority of the valid votes cast.
Prescinding from the Courts ruling that all the probationary employees votes should be deemed
valid votes while that of the supervisory employees should be excluded, it follows that the
number of valid votes cast would increase from 321 to 337. Under Art. 256 of the Labor Code,
the union obtaining the majority of the valid votes cast by the eligible voters shall be certified as
the sole and exclusive bargaining agent of all the workers in the appropriate bargaining unit. This
majority is 50% + 1. Hence, 50% of 337 is 168.5 + 1 or at least 170.
HIMPHLU obtained 169 while petitioner received 151 votes. Clearly, HIMPHLU was not able
to obtain a majority vote. The position of both the SOLE and the appellate court that the opening
of the 17 segregated ballots will not materially affect the outcome of the certification election as
for, so they contend, even if such member were all in favor of petitioner, still, HIMPHLU would
win, is thus untenable.
It bears reiteration that the true importance of ascertaining the number of valid votes cast is for it
to serve as basis for computing the required majority, and not just to determine which union won
the elections. The opening of the segregated but valid votes has thus become material.
To be sure, the conduct of a certification election has a two-fold objective: to determine the
appropriate bargaining unit and to ascertain the majority representation of the bargaining
representative, if the employees desire to be represented at all by anyone. It is not simply the
determination of who between two or more contending unions won, but whether it effectively

ascertains the will of the members of the bargaining unit as to whether they want to be
represented and which union they want to represent them.
Having declared that no choice in the certification election conducted obtained the required
majority, it follows that a run-off election must be held to determine which between
HIMPHLU and petitioner should represent the rank-and-file employees.
PETITION GRANTED.
_____________
NOTES:
A run-off election refers to an election between the labor unions receiving the 2 highest number
of votes in a certification or consent election with 3 or more choices, where such a certified or
consent election results in none of the 3 or more choices receiving the majority of the valid votes
cast; provided that the total number of votes for all contending unions is at least 50% of the
number of votes cast.
From Atty. Bayani^^

Posted October 25, 2012 by vbdiaz in LABOR LAW

SAN MIGUEL CORP EMPLOYEES UNION-PTGWO


vs. CONFESOR Leave a comment
G.R. No. 111262 September 19, 1996
SAN MIGUEL CORPORATION EMPLOYEES UNION-PTGWO, represented by its
President RAYMUNDO HIPOLITO, JR. vs. HON. MA. NIEVES D. CONFESOR,
Secretary of Labor, Dept. of Labor & Employment, SAN MIGUEL CORPORATION,
MAGNOLIA CORPORATION (Formerly, Magnolia Plant) and SAN MIGUEL FOODS,
INC. (Formerly, B-Meg Plant)
FACTS: On June 28, 1990, petitioner-union San Miguel Corporation Employees Union
PTGWO entered into a CBA with private respondent San Miguel Corporation (SMC) to take
effect upon the expiration of the previous CBA or on June 30, 1989.
This CBA provided, among others, that:

ARTICLE XIV
DURATION OF AGREEMENT
Sec. 1. This Agreement which shall be binding upon the parties hereto and their respective
successors-in-interest, shall become effective and shall remain in force and effect until June 30,
1992.
Sec. 2. In accordance with Article 253-A of the Labor Code as amended, the term of this
Agreement insofar as the representation aspect is concerned, shall be for five (5) years from July
1, 1989 to June 30, 1994. Hence, the freedom period for purposes of such representation shall be
sixty (60) days prior to June 30, 1994.
Sec. 3. Sixty (60) days prior to June 30, 1992 either party may initiate negotiations of all
provisions of this Agreement, except insofar as the representation aspect is concerned. If no
agreement is reached in such negotiations, this Agreement shall nevertheless remain in force up
to the time a subsequent agreement is reached by the parties.
Meanwhile, effective October 1, 1991, Magnolia and Feeds and Livestock Division were spunoff and became two separate and distinct corporations: Magnolia Corporation (Magnolia) and
San Miguel Foods, Inc. (SMFI). Notwithstanding the spin-offs, the CBA remained in force and
effect.
After June 30, 1992, the CBA was renegotiated in accordance with the terms of the CBA and
Article 253-A of the Labor Code. Negotiations started sometime in July, 1992 with the two
parties submitting their respective proposals and counterproposals.
During the negotiations, the petitioner-union insisted that the bargaining unit of SMC should still
include the employees of the spun-off corporations: Magnolia and SMFI; and that the
renegotiated terms of the CBA shall be effective only for the remaining period of two years or
until June 30, 1994.
SMC, on the other hand, contended that the members/employees who had moved to Magnolia
and SMFI, automatically ceased to be part of the bargaining unit at the SMC. Furthermore, the
CBA should be effective for three years in accordance with Art. 253-A of the Labor Code.
Unable to agree on these issues with respect to the bargaining unit and duration of the CBA,
petitioner-union declared a deadlock on September 29, 1990.
(Notice of strikeSecretary assumed jurisdiction)
Secretarys decision: the CBA shall be effective for the period of 3 years from June 30, 1992; and
that such CBA shall cover only the employees of SMC and not of Magnolia and SMFI.

ISSUES: 1) Whether or not the duration of the renegotiated terms of the CBA is to be effective
for three years of for only two years; and 2) Whether or not the bargaining unit of SMC includes
also the employees of the Magnolia and SMFI.
HELD: We agree with the Secretary of Labor.
Pertinent to the first issue is Art. 253-A of the Labor Code as amended which reads:
Art. 253-A. Terms of a CBA. Any CBA that the parties may enter into shall, insofar as the
representation aspect is concerned, be for a term of 5 years. No petition questioning the majority
status of the incumbent bargaining agent shall be entertained and no certification election shall
be conducted by the Department of Labor and Employment outside of the sixty-day period
immediately before the date of expiry of such five year term of the CBA. All other provisions of
the CBA shall be renegotiated not later than 3 years after its execution. Any agreement on such
other provisions of the CBA entered into within 6 months from the date of expiry of the term of
such other provisions as fixed in such CBA, shall retroact to the day immediately following such
date. If any such agreement is entered into beyond six months, the parties shall agree on the
duration of retroactivity thereof. In case of a deadlock in the renegotiation of the CBA, the
parties may exercise their rights under this Code. (Emphasis supplied.)
The representation aspect refers to the identity and majority status of the union that negotiated
the CBA as the exclusive bargaining representative of the appropriate bargaining unit concerned.
All other provisions simply refers to the rest of the CBA, economic as well as non-economic
provisions, except representation.
The law is clear and definite on the duration of the CBA insofar as the representation aspect is
concerned, but is quite ambiguous with the terms of the other provisions of the CBA. It is a
cardinal principle of statutory construction that the Court must ascertain the legislative intent for
the purpose of giving effect to any statute.
(as usual mahabang conversation ng mga framers)
Obviously, the framers of the law wanted to maintain industrial peace and stability by having
both management and labor work harmoniously together without any disturbance. Thus, no
outside union can enter the establishment within 5 years and challenge the status of the
incumbent union as the exclusive bargaining agent. Likewise, the terms and conditions of
employment (economic and non-economic) can not be questioned by the employers or
employees during the period of effectivity of the CBA. The CBA is a contract between the
parties and the parties must respect the terms and conditions of the agreement. Notably, the
framers of the law did not give a fixed term as to the effectivity of the terms and conditions
of employment. It can be gleaned from their discussions that it was left to the parties to fix
the period.
The issue as to the term of the non-representation provisions of the CBA need not belaboured.
The parties, by mutual agreement, enter into a renegotiated contract with a term of three
(3) years or one which does not coincide with the said 5-year term, and said agreement is

ratified by majority of the members in the bargaining unit, the subject contract is valid and
legal and therefore, binds the contracting parties.
Thus, we do not find any grave abuse of discretion on the part of the Secretary of Labor in ruling
that the effectivity of the renegotiated terms of the CBA shall be for 3 years.
II. Undeniably, the transformation of the companies was a management prerogative and business
judgment which the courts can not look into unless it is contrary to law, public policy or morals.
Neither can we impute any bad faith on the part of SMC so as to justify the application of the
doctrine of piercing the corporate veil.18 Ever mindful of the employees interests, management
has assured the concerned employees that they will be absorbed by the new corporations without
loss of tenure and retaining their present pay and benefits according to the existing
CBAs. 19 They were advised that upon the expiration of the CBAs, new agreements will be
negotiated between the management of the new corporations and the bargaining
representatives of the employees concerned.
Indubitably, therefore, Magnolia and SMFI became distinct entities with separate juridical
personalities. Thus, they can not belong to a single bargaining unit.
Moreover, in determining an appropriate bargaining unit, the test of grouping is mutuality or
commonality of interests. The employees sought to be represented by the collective bargaining
agent must have substantial mutual interests in terms of employment and working conditions as
evinced by the type of work they performed. 22 Considering the spin-offs, the companies would
consequently have their respective and distinctive concerns in terms of the nature of work,
wages, hours of work and other conditions of employment. Interests of employees in the
different companies perforce differ. The nature of their products and scales of business may
require different skills which must necessarily be commensurated by different compensation
packages. The different companies may have different volumes of work and different working
conditions. For such reason, the employees of the different companies see the need to group
themselves together and organize themselves into distinctive and different groups. It would then
be best to have separate bargaining units for the different companies where the employees can
bargain separately according to their needs and according to their own working conditions.
WHEREFORE, the petition is DISMISSED for lack of merit.
From Atty. Bayani^^
Posted October 25, 2012 by vbdiaz in LABOR LAW

LMG vs.SEC. OF LABOR Leave a comment


LMG CHEMICALS CORP, LMG CHEMICALS CORP vs.THE SECRETARY OF THE
DEPARTMENT OF LABOR AND EMPLOYMENT, THE HON. LEONARDO A.
QUISUMBING, and CHEMICAL WORKERS UNION
G.R. No. 127422

April 17, 2001

FACTS: LMG Chemicals Corp, (petitioner) is a domestic corp engaged in the manufacture and
sale of various kinds of chemical substances, including aluminum sulfate which is essential in
purifying water, and technical grade sulfuric acid used in thermal power plants. Petitioner has
three divisions, namely: the Organic Division, Inorganic Division and the Pinamucan Bulk
Carriers. There are two unions within petitioners Inorganic Division. One union represents the
daily paid employees and the other union represents the monthly paid employees. Chemical
Workers Union, respondent, is a duly registered labor organization acting as the collective
bargaining agent of all the daily paid employees of petitioners Inorganic Division.
Sometime in December 1995, the petitioner and the respondent started negotiation for a new
CBA as their old CBA was about to expire. They were able to agree on the political provisions of
the new CBA, but no agreement was reached on the issue of wage increase. The economic issues
were not also settled.
With the CBA negotiations at a deadlock (StrikeSecretary assumed jurisdiction)
Secretary of Labor and Employment granted an increase of P140 (higher than the offer of
petitioner-company of P135). Also, as to the effectivity of the new CBASec held:
3. Effectivity of the new CBA
Article 253-A of the Labor Code, as amended, provides that when no new CBA is signed
during a period of six months from the expiry date of the old CBA, the retroactivity period
shall be according to the parties agreement, Inasmuch as the parties could not agree on this
issue and since this Office has assumed jurisdiction, then this matter now lies at the discretion
of the Secretary of labor and Employment. Thus the new Collective Bargaining Agreement
which the parties will sign pursuant to this Order shall retroact to January 1, 1996.
petitioner contends that public respondent committed grave abuse of discretion when he ordered
that the new CBA which the parties will sign shall retroact to January 1, 1996
ISSUE: Whether or not the new CBA shall retroact?
HELD:
Petitioner insists that public respondents discretion on the issue of the date of the effectivity of
the new CBA is limited to either: (1) leaving the matter of the date of effectivity of the new CBA
is limited to either: (1) leaving the matter of the date of effectivity of the new CBA to the
agreement of the parties or (2) ordering that the terms of the new CBA be prospectively applied.
It must be emphasized that respondent Secretary assumed jurisdiction over the dispute because it
is impressed with national interest. As noted by the Secretary, the petitioner corp was then
supplying the sulfate requirements of MWSS as well as the sulfuric acid of NAPOCOR, and
consequently, the continuation of the strike would seriously affect the water supply of Metro

Manila and the power supply of the Luzon Grid. Such authority of the Secretary to assume
jurisdiction carries with it the power to determine the retroactivity of the parties CBA.
It is well settled in our jurisprudence that the authority of the Secretary of Labor to assume
jurisdiction over a labor dispute causing or likely to cause a strike or lockout in an industry
indispensable to national interest includes and extends to all questions and controversies
arising therefrom. The power is plenary and discretionary in nature to enable him to
effectively and efficiently dispose of the primary dispute.
This Court held in St. Lukes Medical Center, Inc. vs. Torres:
Therefore in the absence of the specific provision of law prohibiting retroactivity of the
effectivity of the arbitral awards issued by the Secretary of Labor pursuant to Article
263(g) of the Labor Code, such as herein involved, public respondent is deemed vested with
plenary powers to determine the effectivity thereof.
PETITION DENIED.
From Atty. Bayani

Posted October 25, 2012 by vbdiaz in LABOR LAW

NLU vs. PHILIPPINE AMERICAN CIGAR &


CIGARETTE MANUFACTURING CO., INC. Leave a
comment
PHILIPPINE AMERICAN CIGAR & CIGARETTE FACTORY WORKERS
INDEPENDENT UNION (NLU) vs. PHILIPPINE AMERICAN CIGAR & CIGARETTE
MANUFACTURING CO., INC.
G.R. No. L-18364, Feb 28, 1963.

FACTS:

October 23, 1958, Apolonio San Joses brother, Francisco San Jose, who is also a regular worker
of the respondent and a member of the complainant union, filed a charge for ULP against herein
respondent, which case is still pending.

Subsequent to the filing of the said charge, the respondent by its manager Chue Yiong,
summoned and advised union president Lazaro Peralta that if Francisco San Jose will not
withdraw his charge against the company, the company will also dismiss his brother Apolonio
San Jose, to which the union president replied that should not be the attitude of the company
because Apolonio has nothing to do with his brothers case.
On January 24, 1959, respondent did dismiss Apolonio San Jose without just and valid cause and
in gross violation of the operative CBA between the complainant union and respondent
corporation.
ISSUE:
Whether the dismissal of a relative of an employee who filed an action against the employer is an
ULP.

RULING:
YES. Section 4(a) (5) of Republic Act No. 875, provides that :
(a) It shall be ULP for an employer:
(5) To dismiss, discharge, or otherwise prejudice or discriminate against an employee
for having filed charges or for having given or being about to give testimony under this Act.
Although the cited law pertains to the specific employee who filed a case or given a testimony
against the employer, it should be construed in line with the spirit and purpose of said Section 4
and of the legislation of which it forms part namely, to assure absolute freedom of the
employees and laborers to establish labor organizations and unions, as well as to prefer charges
before the proper organs of the Government for violations of our labor laws.
If the dismissal of an employee due to the filing by him of said charges would be and is an undue
restraint upon said freedom, the dismissal of his brother owing to the non-withdrawal of the
charges of the former, would be and constitute as much a restraint upon the same freedom. In
fact, it may be greater and more effective restraint thereto. Indeed, a complainant may be willing
to risk the hazards of a possible and even probable retaliatory action by the employer in the form
of a dismissal or another discriminatory act against him personally, considering that nobody is
perfect, that everybody commits mistakes and that there is always a possibility that the employer
may find in the records of any employee, particularly if he has long been in the service, some act
or omission constituting a fault or negligence which may be an excuse for such dismissal or
discrimination. Yet, such complainant may not withstand the pressure that would result if his
brother or another member of his immediate family were threatened with such action unless the
charges in question were withdrawn.

What is prohibited to be done directly shall not be allowed to be accomplished indirectly. Thus in
the Matter of Quidnick Dye Works Inc. and Federation of Dyers, Finishers, Printers and
Bleachers of America (2 NLRB 963) it was held that the dismissal of a laborer on account of
union activities of his brother constituted an ULP.
The discharge of relatives of an employee who has himself been discriminately discharged, for
no other reason than the relation, is itself a discriminatory discharge, in violation of the Act. An
illustration is Memphis Furniture Co. (3 NLRB 26 [1937]), where the evidence indicated that
the sole reason for the dismissal of a female employee was that she was the wife of an employee
who has been discharged. It was held that the discharge under the circumstances was
discriminatory and a violation of the Act, even though discharged female employee was not
herself a member of any union. The respondent thus made union membership and activities a bar
to the employment not only of the union member himself but of members of his family as well. A
more effective mode of discouraging of union affiliation could hardly be found then the
knowledge that such activities put not merely the union members employment but that of those
closely related to him in jeopardy.
In addition to violating Section 4(a) (5) of Republic Act No. 875, the discharge of Apolonio San
Jose is, therefore, an ULP.
WHEREFORE, the decision appealed from is hereby reversed. Respondent is also guilty of ULP.

From Atty. Daan^^


Posted October 25, 2012 by vbdiaz in LABOR LAW

WMWU vs. BLR Leave a comment


WARREN MANUFACTURING WORKERS UNION (WMWU) vs. BLR; PHILIPPINE
AGRICULTURAL, COMMERCIAL AND INDUSTRIAL WORKERS UNION
(PACIWU); and SAMAHANG MANGGAGAWA SA WARREN MANUFACTURING
CORP.-ALLIANCE OF NATIONALIST AND GENUINE LABOR ORGANIZATIONS
(SMWMC-ANGLO),
G.R. No. L-76185, Mar 30, 1988

FACTS:
On June 13, 1985, Philippine Agricultural, Commercial and Industrial Workers Union
(PACIWU) filed a petition for certification election, alleging compliance with the jurisdictional
requirements.

On July 7, 1985, petitioner thru counsel filed a motion to dismiss the petition on the ground that
there exists a C.B.A. between the respondent and the Warren Mfg. Union which took effect upon
its signing on July 16, 1985 and to expire on July 31, 1986.
While the petition was under hearing, PACIWU filed a Notice of Strike and on conciliation
meeting, a Return-to-Work Agreement was signed and stipulating that to resolve the issue of
union representation at Warren Mfg. Corp., parties have agreed to the holding of a consent
election among the rank and file on August 25, 1985 at the premises of the company to be
supervised by MOLE. It is clearly understood that the certified union in the said projected
election shall respect and administer the existing CBA at the company until its expiry date on
July 31, 1986.
On 12 August 1985, an Order was issued by BLR, directing that a consent election be held
among the rank and file workers of the company, with the following contending unions:
1. Philippine Agricultural, Commercial and Industrial Workers Union (PACIWU);
2. Warren Mfg. Workers Union (WMWU);
3. No Union.
On August 25, 1985, said consent election was held, and yielded the following results:
PACIWU- 94
WMWU-193
On June 5, 1986, the PACIWU filed a petition for certification election followed by the filing of
a petition for the same purposes by the Samahan ng Manggagawa sa Warren Manufacturing
Corporation-Alliance of Nationalist and Genuine Labor Organizations (Anglo for short) which
petitions were both opposed by Warren Manufacturing Corporation on the grounds that neither
petition has 30% support; that both are barred by the one-year no certification election law and
the existence of a duly ratified CBA.
The Med-Arbiter-NCR, MOLE ordered on August 18, 1986 the holding of a certification election
within 20 days from receipt to determine the exclusive bargaining representative of all the rank
and file employees of the Warren Manufacturing Corporation, with the choices:
1. Philippine Agricultural, Commercial and Industrial Workers Union (PACIWU);
2. Warren Mfg. Workers Union;
3. Samahan ng Manggagawa sa Warren Mfg. Corporation-ANGLO; and
4. No Union.

Both Warren Manufacturing Corporation and petitioner filed separate motions treated as appeals
by the BLR which was dismissed. Hence, this petition solely by WMWU. Petitioner invoked the
one-year no certification election rule and the principle of the Contract Bar Rule.

ISSUE:
Whether the Certification Election should be granted.

RULING:
YES. The records show that petitioner admitted that what was held on August 25, 1985 at the
Companys premises and which became the root of this controversy, was a consent election and
not a certification election. A consent election is an agreed one, its purpose being merely to
determine the issue of majority representation of all the workers in the appropriate collective
bargaining unit while a certification election is aimed at determining the sole and exclusive
bargaining agent of all the employees in an appropriate bargaining unit for the purpose of
collective bargaining. From the very nature of consent election, it is a separate and distinct
process and has nothing to do with the import and effect of a certification election. Neither does
it shorten the terms of an existing CBA nor entitle the participants thereof to immediately
renegotiate an existing CBA although it does not preclude the workers from exercising their right
to choose their sole and exclusive bargaining representative after the expiration of the 60 day
freedom period.
The election held on August 25, 1985 was not for the purpose of determining which labor union
should be the bargaining representative in the negotiation for a collective contract, there being an
existing collective bargaining agreement yet to expire on July 31, 1986; but only to determine
which labor union shall administer the said existing contract.
Article 257 of the New Labor Code expressly states that No certification election issue shall be
entertained if a collective agreement which has been submitted in accordance with Article 231 of
the Code exists between the employer and a legitimate labor organization except within 60 days
prior to the expiration of the life of such certified collective bargaining agreement. There should
be no obstacle to the right of the employees to petition for a certification election at the proper
time.
As aforestated, the existing collective bargaining agreement was due to expire on July 31, 1986.
The Med-Arbiter found that the petition and intervention were supported by more than 30% of
the members of the bargaining unit. Because of this, Article 258 of the Labor Code makes it
mandatory for the BLR to conduct a certification election. Once it has been verified that the
petition for certification election has the support of at least 30% of the employees in the
bargaining unit, it must be granted. It becomes under the circumstances, mandatory .

WHEREFORE, the instant Petition is DISMISSED.

From Atty. Daan^^


Posted October 23, 2012 by vbdiaz in LABOR LAW

ACOJE VS. NAMAWU Leave a comment


ACOJE WORKERS UNION vs. NATIONAL MINES AND ALLIED WORKERS UNION
(NAMAWU), ACOJE MINES COMPANY and COURT OF INDUSTRIAL RELATIONS
G.R. No. L-18848

April 23, 1963

FACTS:
Department of Labor, through the BLR, conducted on June 9, 1961, a consent election among
the workers of the Acoje Mining Company at Santa Cruz, Zambales, in which 5 labor unions
participated, namely, the Acoje United Workers Union, the Acoje Labor Union (PELTA), the
Acoje Labor Union (PLUM), respondent National Mines and Allied Workers Union
(NAMAWU), and petitioner Acoje Workers Union. NAMAWU won in the said election.
Petitioner Union which had been defeated by respondent Union by a margin of 282 votes
had filed a motion to invalidate said election upon several grounds. After hearing, the lower court
issued, on July 21, 1961, the order appealed from holding that said motion was without merit,
and certifying respondent Union NAMAWU as the sole and exclusive bargaining agent of all the
workers of the Company. MR of petitioner was denied, hence this present appeal by certiorari,
and petitioner now maintains that the lower court should have invalidated the aforementioned
election for the same was the result of acts of terrorism, force, threat and intimidation employed
by agents of respondent Union. The petitioner also questioned the list of qualified voters that
was used during the election which was based on the payroll of the employees.
ISSUE:
Can a payroll be used as the basis for qualified employee- voters?

RULING:
YES. It appears that labor unions concerned agreed, not only to the holding of the
aforementioned election, but also to the use of the Company payroll of March 31, 1961, as the
basis for determining who are qualified to vote subject to the approval of the lower court. The

Company presented its aforementioned payroll to said court and stated that the labor unions had
been furnished copy thereof, at least 3 days prior thereto. Said labor unions were given an
opportunity to make their comments and observations on the list of workers contained in the
payroll and to ask or suggest the inclusion or exclusion of names therein or therefrom.
Petitioners representative then stated that it would abide by whatever ruling the court may make
on the matter of inclusion and exclusion of voters. Indeed, on May 19, 1961, the court issued the
corresponding order for the holding of the election and made its ruling on the question as to who
were qualified to vote, and petitioner did not move for a reconsideration of said ruling. Hence,
petitioner may no longer contest the accuracy of the aforementioned voters list.
As to the allegation of petitioner that there are many cases where the workers were threatened,
coerced and intimidated to vote for the NAMAWU, is but a general allegation, without anything
to indicate the number of workers involved, without the supporting affidavit of any of them, and
without an offer to introduce their testimony or the testimony of any of them was. Petitioners
contention is insufficient to warrant the invalidation of the aforementioned election.
WHEREFORE, the order appealed from is hereby affirmed.

From Atty. Daan^^


Posted October 23, 2012 by vbdiaz in LABOR LAW

BENGUET CONSOLIDATED, INC. vs. BCI EMPLOYEES


& WORKERS UNION-PAFLU, PHILIPPINE
ASSOCIATION OF FREE LABOR UNIONS, CIPRIANO
CID and JUANITO GARCIA Leave a comment
BENGUET CONSOLIDATED, INC. vs. BCI EMPLOYEES & WORKERS UNIONPAFLU, PHILIPPINE ASSOCIATION OF FREE LABOR UNIONS, CIPRIANO CID and
JUANITO GARCIA
G.R. No. L-24711,; Apr 30, 1968

FACTS:
On June 23, 1959, the Benguet-Balatoc Workers Union (BBWU), for and in behalf of all
Benguet Consolidated, Inc (BENGUET) employees in its mines and milling establishment
located at Balatoc, Antamok and Acupan, Mt. Province, entered into a Collective Bargaining
Contract (CONTRACT) with BENGUET. The CONTRACT was stipulated to be effective for a

period of 4-1/2 years, or from June 23, 1959 to December 23, 1963. It likewise embodied a NoStrike, No-Lockout clause.
3 years later, or on April 6, 1962, a certification election was conducted by the Department of
Labor among all the rank and file employees of BENGUET in the same collective bargaining
units. BCI EMPLOYEES & WORKERS UNION (UNION) obtained more than 50% of the total
number of votes, defeating BBWU. The Court of Industrial Relations certified the UNION as the
sole and exclusive collective bargaining agent of all BENGUET employees as regards rates of
pay, wages, hours of work and such other terms and conditions of employment allowed them by
law or contract.
Later on, the UNION filed a notice of strike against BENGUET. UNION members who were
BENGUET employees in the mining camps at Acupan, Antamok and Balatoc, went on strike.
The strike was attended by violence, some of the workers and executives of the BENGUET were
prevented from entering the premises and some of the properties of the BENGUET were
damaged as a result of the strike. Eventually, the parties agreed to end the dispute. BENGUET
and UNION executed the AGREEMENT. PAFLU placed its conformity thereto. About a year
later or on January 29, 1964, a collective bargaining contract was finally executed between
UNION-PAFLU and BENGUET.
Meanwhile, BENGUET sued UNION, PAFLU and their Presidents to recover the amount the
former incurred for the repair of the damaged properties resulting from the strike. BENGUET
also argued that the UNION violated the CONTRACT which has a stipulation not to strike
during the effectivity thereof.
Defendants unions and their presidents defended that: (1) they were not bound by the
CONTRACT which BBWU, the defeated union, had executed with BENGUET; (2) the strike
was due, among others, to unfair labor practices of BENGUET; and (3) the strike was lawful and
in the exercise of the legitimate rights of UNION-PAFLU under Republic Act 875.
The trial court dismissed the complaint on the ground that the CONTRACT, particularly the NoStrike clause, did not bind defendants. BENGUET interposed the present appeal.

ISSUE:
Did the Collective Bargaining Contract executed between Benguet and BBWU on June 23, 1959
and effective until December 23, 1963 automatically bind UNION-PAFLU upon its certification,
on August 18, 1962, as sole bargaining representative of all BENGUET employees

RULING:

NO. BENGUET erroneously invokes the so-called Doctrine of Substitution referred to in


General Maritime Stevedores Union v. South Sea Shipping Lines where it was ruled that:
We also hold that where the bargaining contract is to run for more than two years, the principle
of substitution may well be adopted and enforced by the CIR to the effect that after two years of
the life of a bargaining agreement, a certification election may be allowed by the CIR, that if a
bargaining agent other than the union or organization that executed the contract, is elected, said
new agent would have to respect said contract, but that it may bargain with the management for
the shortening of the life of the contract if it considers it too long, or refuse to renew the contract
pursuant to an automatic renewal clause.
BENGUETs reliance upon the Principle of Substitution is totally misplaced. This principle,
formulated by the NLRB as its initial compromise solution to the problem facing it when there
occurs a shift in employees union allegiance after the execution of a bargaining contract with
their employer, merely states that even during the effectivity of a collective bargaining agreement
executed between employer and employees thru their agent, the employees can change said agent
but the contract continues to bind them up to its expiration date. They may bargain however for
the shortening of said expiration date.
In formulating the substitutionary doctrine, the only consideration involved was the
employees (principal) interest in the existing bargaining agreement. The agents (union) interest
never entered the picture. The majority of the employees, as an entity under the statute, is the
true party in interest to the contract, holding rights through the agency of the union
representative. Thus, any exclusive interest claimed by the agent is defeasible at the will of the
principal. The substitutionary doctrine only provides that the employees cannot revoke the
validly executed collective bargaining contract with their employer by the simple expedient of
changing their bargaining agent. And it is in the light of this that the phrase said new agent
would have to respect said contract must be understood. It only means that the employees, thru
their new bargaining agent, cannot renege on their collective bargaining contract, except of
course to negotiate with management for the shortening thereof.
The substitutionary doctrine cannot be invoked to support the contention that a newly certified
collective bargaining agent automatically assumes all the personal undertakings like the nostrike stipulation here in the collective bargaining agreement made by the deposed union.
When BBWU bound itself and its officers not to strike, it could not have validly bound also all
the other rival unions existing in the bargaining units in question. BBWU was the agent of the
employees, not of the other unions which possess distinct personalities.
UNION, as the newly certified bargaining agent, could always voluntarily assume all the
personal undertakings made by the displaced agent. But as the lower court found, there was no
showing at all that, prior to the strike, UNION formally adopted the existing CONTRACT as its
own and assumed all the liabilities imposed by the same upon BBWU. Defendants were neither
signatories nor participants in the CONTRACT.
Everything binding on a duly authorized agent, acting as such, is binding on the principal; not
vice-versa, unless there is mutual agency, or unless the agent expressly binds himself to the party

with whom he contracts. Here, it was the previous agent who expressly bound itself to the other
party, BENGUET. UNION, the new agent, did not assume this undertaking of BBWU.
Since defendants were not contractually bound by the no-strike clause in the CONTRACT, for
the simple reason that they were not parties thereto, they could not be liable for breach of
contract to plaintiff.
WHEREFORE, the judgment of the lower court appealed from is hereby affirmed.

From Atty. Daan^^


Posted October 23, 2012 by vbdiaz in LABOR LAW

ISAE vs. QUISUMBING Leave a comment


INTERNATIONAL SCHOOL ALLIANCE OF EDUCATORS (ISAE), petitioner, vs. HON.
LEONARDO A. QUISUMBING in his capacity as the Secretary of Labor and
Employment; HON. CRESENCIANO B. TRAJANO in his capacity as the Acting Secretary
of Labor and Employment; DR. BRIAN MACCAULEY in his capacity as the
Superintendent of International School-Manila; and INTERNATIONAL SCHOOL, INC.,
respondents.,
G.R. No. 128845, June 1, 2000

FACTS:

Private respondent International School, Inc. (School), pursuant to PD 732, is a domestic


educational institution established primarily for dependents of foreign diplomatic personnel and
other temporary residents. The decree authorizes the School to employ its own teaching and
management personnel selected by it either locally or abroad, from Philippine or other
nationalities, such personnel being exempt from otherwise applicable laws and regulations
attending their employment, except laws that have been or will be enacted for the protection of
employees. School hires both foreign and local teachers as members of its faculty, classifying the
same into two: (1) foreign-hires and (2) local-hires.
The School grants foreign-hires certain benefits not accorded local-hires. Foreign-hires are also
paid a salary rate 25% more than local-hires.
When negotiations for a new CBA were held on June 1995, petitioner ISAE, a legitimate labor
union and the collective bargaining representative of all faculty members of the School,

contested the difference in salary rates between foreign and local-hires. This issue, as well as the
question of whether foreign-hires should be included in the appropriate bargaining unit,
eventually caused a deadlock between the parties.
ISAE filed a notice of strike. Due to the failure to reach a compromise in the NCMB, the matter
reached the DOLE which favored the School. Hence this petition.

ISSUE:
Whether the foreign-hires should be included in bargaining unit of local- hires.

RULING:
NO. The Constitution, Article XIII, Section 3, specifically provides that labor is entitled to
humane conditions of work. These conditions are not restricted to the physical workplace the
factory, the office or the field but include as well the manner by which employers treat their
employees.
Discrimination, particularly in terms of wages, is frowned upon by the Labor Code. Article 248
declares it an unfair labor practice for an employer to discriminate in regard to wages in order to
encourage or discourage membership in any labor organization.
The Constitution enjoins the State to protect the rights of workers and promote their welfare, In
Section 18, Article II of the constitution mandates to afford labor full protection. The State has
the right and duty to regulate the relations between labor and capital. These relations are not
merely contractual but are so impressed with public interest that labor contracts, collective
bargaining agreements included, must yield to the common good.
However, foreign-hires do not belong to the same bargaining unit as the local-hires.
A bargaining unit is a group of employees of a given employer, comprised of all or less than all
of the entire body of employees, consistent with equity to the employer indicate to be the best
suited to serve the reciprocal rights and duties of the parties under the collective bargaining
provisions of the law.
The factors in determining the appropriate collective bargaining unit are (1) the will of the
employees (Globe Doctrine); (2) affinity and unity of the employees interest, such as substantial
similarity of work and duties, or similarity of compensation and working conditions (Substantial
Mutual Interests Rule); (3) prior collective bargaining history; and (4) similarity of employment
status. The basic test of an asserted bargaining units acceptability is whether or not it is
fundamentally the combination which will best assure to all employees the exercise of their
collective bargaining rights.

In the case at bar, it does not appear that foreign-hires have indicated their intention to be
grouped together with local-hires for purposes of collective bargaining. The collective bargaining
history in the School also shows that these groups were always treated separately. Foreign-hires
have limited tenure; local-hires enjoy security of tenure. Although foreign-hires perform similar
functions under the same working conditions as the local-hires, foreign-hires are accorded
certain benefits not granted to local-hires such as housing, transportation, shipping costs, taxes
and home leave travel allowances. These benefits are reasonably related to their status as foreignhires, and justify the exclusion of the former from the latter. To include foreign-hires in a
bargaining unit with local-hires would not assure either group the exercise of their respective
collective bargaining rights.
WHEREFORE, the petition is GIVEN DUE COURSE. The petition is hereby GRANTED IN
PART.

From Atty. Daan^^


Posted October 23, 2012 by vbdiaz in LABOR LAW

CMC-ACE-UFSW vs. LAGUESMA Leave a comment


CAPITOL MEDICAL CENTER OF CONCERNED EMPLOYEES-UNIFIED FILIPINO
SERVICE WORKERS, (CMC-ACE-UFSW) vs. HON. BIENVENIDO E. LAGUESMA,
Undersecretary of the Department of Labor and Employment; CAPITOL MEDICAL
CENTER EMPLOYEES ASSOCIATION-ALLIANCE OF FILIPINO WORKERS AND
CAPITOL MEDICAL CENTER INCORPORATED AND DRA. THELMA CLEMENTE,
President, respondents.,
G.R. No. 118915,; Feb 4, 1997.

FACTS:
Respondent union filed petition for certification election. The Med-Arbiter granted the petition
for certification election. Respondent Capitol Medical Center (CMC) appealed to the Office of
the Secretary. But the Order granting the certification election was affirmned.
On December 9, 1992, elections were held with respondent union garnering 204 votes, 168 in
favor of no union and 8 spoiled ballots out of a total of 380 votes cast. Med-Arbiter issued an
Order certifying respondent union as the sole and exclusive bargaining representative of the rank
and file employees at CMC.
Respondent CMC again appealed to the Office of the Secretary of Labor the result of the
election, it was denied. MR also denied. Respondent CMCs contention was the supposed

pendency of its petition for cancellation of respondent unions certificate of registration. In the
said case, the Med-Arbiter therein issued an Order which declared respondent unions certificate
of registration as null and void. However, this order was reversed on appeal by the Officer-inCharge of the BLR in her Order. The said Order dismissed CMCs motion for cancellation of the
certificate of registration of respondent union and declared that it was not only a bona fide
affiliate or local of a federation, but a duly registered union as well.
Respondent union, after being declared as the certified bargaining agent of the rank-and-file
employees of respondent CMC, presented proposals for the negotiation of a CBA. However,
CMC contended that CBA negotiations should be suspended in view of the Order declaring the
registration of respondent union as null and void. In spite of the refusal of respondent CMC,
respondent union still persisted in its demand for CBA negotiations, claiming that it has already
been declared as the sole and exclusive bargaining agent of the rank-and-file employees of the
hospital.
Due to respondent CMCs refusal to bargain collectively, respondent union filed a notice of strike
and later staged a strike on April 15, 1993. The case was certified to the NLRC for compulsory
arbitration.
It is at this point that petitioner union, on March 24, 1994, filed a petition for certification
election among the regular rank-and-file employees of the Capitol Medical Center Inc. It alleged
in its petition that: 1) three hundred thirty one (331) out of the four hundred (400) total rank-andfile employees of respondent CMC signed a petition to conduct a certification election; and 2)
that the said employees are withdrawing their authorization for the said union to represent them
as they have joined and formed the union Capitol Medical Center Alliance of Concerned
Employees (CMC-ACE). They also alleged that a certification election can now be conducted as
more that 12 months have lapsed since the last certification election was held.
Respondent union opposed the petition and moved for its dismissal. It contended that it is the
certified bargaining agent of the rank-and-file employees of the Hospital, which was confirmed
by the Secretary of DOLE and by this Court. It also alleged that it was not negligent in asserting
its right as the certified bargaining agent for it continuously demanded the negotiation of a CBA
with the hospital despite the latters avoidance to bargain collectively.
May 12, 1994, Med-Arbiter Brigida Fadrigon, issued an Order granting the petition for
certification election among the rank and file employees. On appeal by respondent union, the
public respondent Laguesma reversed and favored the respondent union. Hence this petition.

ISSUE:
Was there a bargaining deadlock between CMC and respondent union.

RULING:
While it is true that one year had lapsed since the time of declaration of a final certification
result, and that there is no collective bargaining deadlock, public respondent did not commit
grave abuse of discretion when it ruled in respondent unions favor since the delay in the forging
of the CBA could not be attributed to the fault of the latter.
After respondent union was certified as the bargaining agent of CMC, it invited the employer
hospital to the bargaining table by submitting its economic proposal for a CBA. However, CMC
refused to negotiate with respondent union and instead challenged the latters legal personality
through a petition for cancellation of the certificate of registration which eventually reached this
Court. The decision affirming the legal status of respondent union should have left CMC with no
other recourse but to bargain collectively; but still it did not. Respondent union was left with no
other recourse but to file notice of strike against CMC for unfair labor practice with the NCMB.
This eventually led to a strike.
A deadlock is the counteraction of things producing entire stoppage; There is a deadlock when
there is a complete blocking or stoppage resulting from the action of equal and opposed forces.
The word is synonymous with the word impasse, which presupposes reasonable effort at good
faith bargaining which, despite noble intentions, does not conclude in agreement between the
parties.
Although there is no deadlock in its strict sense as there is no counteraction of forces present
in this case nor reasonable effort at good faith bargaining, such can be attributed to CMCs
fault as the bargaining proposals of respondent union were never answered by CMC. In fact,
what happened in this case is worse than a bargaining deadlock for CMC employed all legal
means to block the certification of respondent union as the bargaining agent of the rank-and-file;
and use it as its leverage for its failure to bargain with respondent union. We can only conclude
that CMC was unwilling to negotiate and reach an agreement with respondent union. CMC has
not at any instance shown willingness to discuss the economic proposals given by respondent
union.
It is only just and equitable that the circumstances in this case should be considered as similar in
nature to a bargaining deadlock when no certification election could be held. This is also to
make sure that no floodgates will be opened for the circumvention of the law by unscrupulous
employers to prevent any certified bargaining agent from negotiating a CBA. Thus, Section 3,
Rule V, Book V of the Implement Rules should be interpreted liberally so as to include a
circumstance, e.g. where a CBA could not be concluded due to the failure of one party to
willingly perform its duty to bargain collectively.
WHEREFORE, the petition is hereby DISMISSED.

From Atty. Daan^^

Posted October 23, 2012 by vbdiaz in LABOR LAW

SHELL OIL WORKERS UNION vs. SHELL COMPANY


OF THE PHILIPPINES, LTD., and THE COURT OF
INDUSTRIAL RELATIONS Leave a comment
SHELL OIL WORKERS UNION vs. SHELL COMPANY OF THE PHILIPPINES, LTD.,
and THE COURT OF INDUSTRIAL RELATIONS
G.R. No. L-28607, May 31, 1971.

FACTS:
Respondent Shell Company of the Philippines (COMPANY) dissolved its security guard section
stationed at its Pandacan Installation, notwithstanding its (guard section) continuance and that
such is assured by an existing collective bargaining contract. The respondent company
transferred 18 security guards to its other department and consequently hired a private security
agency to undertake the work of said security guards. This resulted in a strike called by petitioner
Shell Oil Workers Union (UNION), The President certified it to respondent Court of Industrial
Relations (CIR). CIR declared the strike illegal on the ground that such dissolution was a valid
exercise of a management prerogative. Thus this appeal is taken.
Petitioner argued that the 18 security guards affected are part of the bargaining unit and covered
by the existing collective bargaining contract, as such, their transfers and eventual dismissals are
illegal being done in violation of the existing contract. The Company maintained that in
contracting out the security service and redeploying the 18 security guards affected, it was
merely performing its legitimate prerogative to adopt the most efficient and economical method
of operation, that said action was motivated by business consideration in line with past
established practice and made after notice to and discussion with the Union, that the 18 guards
concerned were dismissed for wilfully refusing to obey the transfer order, and that the strike
staged by the Union is illegal.

ISSUE:
Whether the existing collective bargaining contract on maintaining security guard section, among
others, constitute a bar to the decision of the management to contract out security guards.

RULING:

YES. The strike was legal because there was a violation of the collective bargaining agreement
by Company. It was part of the CBA that the Security Guard Section will remain. Yet, the
Company did not comply with the stipulation in CBA. It was thus an assurance of security of
tenure, at least, during the lifetime of the agreement. For what is involved is the integrity of the
agreement reached, the terms of which should be binding on both parties
The stand of Shell Company as to the scope of management prerogative is not devoid of
plausibility, management prerogative of the Company would have been valid if it were not bound
by what was stipulated in CBA. The freedom to manage the business remains with management.
It cannot be denied the faculty of promoting efficiency and attaining economy by a study of what
units are essential for its operation. To it belongs the ultimate determination of whether services
should be performed by its personnel or contracted to outside agencies. However, while
management has the final say on such matter, the labor union is not to be completely left out.
An unfair labor practice is committed by a labor union or its agent by its refusal to bargain
collectively with the employer. Collective bargaining does not end with the execution of an
agreement, being a continuous process, the duty to bargain necessarily imposing on the parties
the obligation to live up to the terms of such a collective bargaining agreement if entered into, it
is undeniable that non-compliance therewith constitutes an unfair labor practice.
The right to self-organization guarded by the Industrial Peace Act explicitly includes the right to
engage in concerted activities for the purpose of collective bargaining and to the mutual aid or
protection. The employee, tenant or laborer is inhibited from striking or walking out of his
employment only when so enjoined by the CIR and after a dispute has been submitted thereto
and pending award or decision by the court of such dispute.
In the present case, the employees or laborers may strike before being ordered not to do so and
before an industrial dispute is submitted to the CIR, subject to the power of the latter, after
hearing when public interest so requires or when the dispute cannot, in its opinion, be promptly
decided or settled, to order them to return to work, with the consequence that if the strikers fail to
return to work, when so ordered, the court may authorize the employer to accept other employees
or laborers. Thus a strike may not be staged only when, during the pendency of an industrial
dispute, the CIR has issued the proper injunction against the laborers (section 19,
Commonwealth Act No. 103, as amended).
WHEREFORE, the decision of respondent Court of Industrial Relations of August 5, 1967 is
reversed.
**NOTE:
BELIEF IN GOOD FAITH THAT EMPLOYER COMMITTED UNFAIR LABOR PRACTICE
RENDERS STRIKE LEGAL:

It is not even required that there be in fact an unfair labor practice committed by the employer. It
suffices, if such a belief in good faith is entertained by labor, as the inducing factor for staging a
strike. So it was declared: As a consequence, we hold that the strike in question had been called
to offset what petitioners were wanted in believing in good faith to be unfair labor practices on
the part of Management, that petitioners were not bound, therefore, to wait for the expiration of
thirty (30) days from notice of strike before staging the same, that said strike was not,
accordingly, illegal and that the strikers had not thereby lost their status as employees of
respondents herein.

From Atty. Daan^^


Posted October 23, 2012 by vbdiaz in LABOR LAW

UST FACULTY UNION VS. BITONIO Leave a comment


G.R. No. 131235 November 16, 1999
UST FACULTY UNION (USTFU), GIL Y. GAMILLA, CORAZON QUI, NORMA
CALAGUAS, IRMA POTENCIANO, LUZ DE GUZMAN, REMEDIOS GARCIA, RENE
ARNEJO, EDITHA OCAMPO, CESAR REYES, CELSO NIERRA, GLICERIA
BALDRES, MA. LOURDES MEDINA, HIDELITA GABO, MAFEL YSRAEL, LAURA
ABARA, NATIVIDAD SANTOS, FERDINAND LIMOS, CARMELITA ESPINA,
ZENAIDA FAMORCA, PHILIP AGUINALDO, BENEDICTA ALAVA and LEONCIO
CASAL, petitioners,
vs.
Dir. BENEDICTO ERNESTO R. BITONIO JR. of the Bureau of Labor Relations, MedArbiter TOMAS F. FALCONITIN of The National Capital Region, Department of Labor
and Employment (DOLE), EDUARDO J. MARIO JR., MA. MELVYN ALAMIS,
NORMA COLLANTES, URBANO ALABAGIA, RONALDO ASUNCION, ZENAIDA
BURGOS, ANTHONY CURA, FULVIO M. GUERRERO, MYRNA HILARIO,
TERESITA MEER, FERNANDO PEDROSA, NILDA REDOBLADO, RENE SISON,
EVELYN TIROL and ROSIE ALCANTARA,respondents.
FACTS: Private Responednts are duly elected officers of the UST Faculty Union (USTFU). The
union has a subsisting five-year CBA with UST. The petitioners on the other hand, questioned
before the Med-Arbiter, that the COMELEC was not constituted in accordance with USTFUs
constitution and by-laws (CBL) and that no rules had been issued to govern the conduct of the 05
October 1996 election. Med-Arbiter issued a TRO enjoining the conduct of elections. However, a
general faculty assembly was held as scheduled. The general assembly was attended by members
of the USTFU and, as admitted by the appellants, also by non-USTFU members [who] are
members in good standing of the UST Academic Community Collective Bargaining Unit. On
this occasion, appellants were elected as USTFUs new set of officers by acclamation and
clapping of hands.

On 03 December 1996, appellants and UST allegedly entered into another CBA covering the
period from 01 June 1996 to 31 May 2001. Said CBA was ratified by a majority of the UST
faculty community.
ISSUE: WON the election of the officers in this case was valid
HELD: NO. The importance of a unions constitution and bylaws cannot be overemphasized.
They embody a covenant between a union and its members and constitute the fundamental law
governing the members rights and obligations. As such, the unions constitution and bylaws
should be upheld, as long as they are not contrary to law, good morals or public policy.
A union election is held pursuant to the unions constitution and bylaws, and the right to vote in
it is enjoyed only by union members. A union election should be distinguished from a
certification election, which is the process of determining, through secret ballot, the sole
and exclusive bargaining agent of the employees in the appropriate bargaining unit, for
purposes of collective bargaining. Specifically, the purpose of a certification election is to
ascertain whether or not a majority of the employees wish to be represented by a labor
organization and, in the affirmative case, by which particular labor organization.
In a certification election, all employees belonging to the appropriate bargaining unit can
vote. Therefore, a union member who likewise belongs to the appropriate bargaining unit is
entitled to vote in said election. However, the reverse is not always true; an employee
belonging to the appropriate bargaining unit but who is not a member of the union cannot
vote in the union election, unless otherwise authorized by the constitution and bylaws of the
union. Verily, union affairs and elections cannot be decided in a non-union activity.
In both elections, there are procedures to be followed. Thus, the October 4, 1996 election cannot
properly be called a union election, because the procedure laid down in the USTFUs CBL for
the election of officers was not followed. It could not have been a certification election either,
because representation was not the issue, and the proper procedure for such election was not
followed. The participation of non-union members in the election aggravated its irregularity.
From Atty. Renes^^
Posted October 23, 2012 by vbdiaz in LABOR LAW

SAN PEDRO HOSPITAL VS. SEC. OF LABOR Leave a


comment
G.R. No. 104624 October 11, 1996
SAN PEDRO HOSPITAL OF DIGOS, INC., petitioner,
vs.
SECRETARY OF LABOR, THE SAN PEDRO HOSPITAL EMPLOYEES UNION
NATIONAL FEDERATION OF LABOR, respondents.

FACTS: Petitioner had a three-year collective bargaining agreement (CBA) covering the period
December 15, 1987 until December 15, 1990, with herein private respondent, Nagkabiusang
Mamumuo sa San Pedro Hospital of Digos National Federation of Labor (NAMASAP-NFL),
the exclusive bargaining agent of the hospitals rank-and-file workers. After the parties failed to
reach agreement on the issues of raising wages, the union during the meeting of February 19,
1991 declared a deadlock.
On February 20, 1991, respondent union saturated petitioners premises with streamers and
picketed the hospital. The operations of the hospital having come to a grinding halt, the hospital
management considered the union actions as tantamount to a strike. On May 28, 1991,
respondent union struck. Despite the NCMBs call for a conciliation conference, nurses and
nurse aides who were members of the union abandoned their respective department and joined
the picket line a week later. Doctors began leaving the hospital and the number of patients
dwindled. The last patient was discharged on June 10, 1991.
On June 12, 1991, a Notice of Temporary Suspension of Operation was issued by petitioner
hospital and submitted to the local office of the NCMB on June 14, 1991. Then Secretary of
Labor Nieves Confessor assumed jurisdiction over the labor dispute and issued an order directing
all workers to return to work. However, this order was received by petitioner only on June 20,
1991. In the meantime, it had already notified the DOLE via its letter dated June 13, 1991, which
was received by the DOLE on June 14, 1991, that it would temporarily suspend operations for
six (6) months effective June 15, 1991, or up to December 15, 1991. Petitioner thus refused the
return of its striking workers on account of such suspension of operations.
ISSUE: WON the Secretary can validly compel the employer to enter into a new CBA even
during temporary suspension of operations (what if in permanent closure?)
Temporary suspension of operations is recognized as a valid exercise of management
prerogative provided it is not carried out in order to circumvent the provisions of the
Labor Code or to defeat the rights of the employees under the Code. The determination to
cease or suspend operations is a prerogative of management that the State usually does not
interfere with, as no business can be required to continue operating at a loss simply to maintain
the workers in employment. Such an act would be tantamount to a taking of property without due
process of law, which the employer has a right to resist. But where it is shown that the closure is
motivated not by a desire to prevent further losses, but to discourage the workers from
organizing themselves into a union for more effective negotiation with management, the State is
bound to intervene.
The burden of proving that such a temporary suspension is bona fide falls upon the employer. In
this instance, petitioner had to establish the fact of its precarious financial health, that its
cessation of operation was really necessitated by its financial condition, and that said condition
would probably be alleviated or improved, or its losses abated, by undertaking such suspension
of operation. The fact that the conciliator never asked for them is no sufficient excuse for not
presenting the same, as such was petitioners duty. Neither is it acceptable for petitioner to allege
that latest financial statement (for the year 1991) were still being prepared by its accountants and

not yet ready for submission, since the financial statement for the prior years 1989 and 1990
would have sufficed.
It is a hornbook rule that employers who contemplate terminating the services of their workers
must base their decisions on more than just flimsy excuses, considering that the dismissal of an
employee from work involves not only the loss of his position but, what is more important, his
means of livelihood. The same principle applies in temporary suspension of operations, as in this
case, considering that it involves laying off employees for a period of six months. Petitioner,
having wretchedly failed to justify by even the most rudimentary proof its temporary suspension
of operations, must bear the consequences thereof. We thus hold that the Secretary of Labor and
Employment did not act with grave abuse of discretion in finding the temporary suspension
unjustified and illegal.
The order of the secretary in ordering the hospital to enter into a new CBA was valid.
Secretary was of the impression that petitioner would operate again after the lapse of the sixmonth suspension of operations on December 16, 1991, and so ordered the parties to enter into
and formalize a new CBA to govern their relations upon resumption of operations. On the other
hand, the aforequoted portion of the Order must be understood in the context of the Secretarys
finding that the temporary suspension was only for circumventing the return-to-work order, but
in spite of which he held that he could not order petitioner to continue operations as this would
infringe on its inherent right to manage and conduct its own business affairs; he thus ordered
instead the payment of backwages to the returning workers who were refused admittance by
petitioner on June 21, 1991. And as above adverted to, he also ordered the parties to execute a
new CBA to govern their relations upon the expiry of the period of suspension and the
resumption of normal operations.
Did the Secretary act in excess of jurisdiction in imposing the wage increase and union shop
provision on the petitioner? We hold that he did not. While petitioner cannot be forced to
abandon its suspension of operations even if said suspension be declared unjustified, illegal and
invalid, neither can petitioner evade its obligation to bargain with the union, using the cessation
of its business as reason therefor. For, as already indicated above, the employer-employee
relationship was merely suspended (and not terminated) for the duration of the temporary
suspension. Using the suspension as an excuse to evade the duty to bargain is further proof of its
illegality. It shows abuse of this option and bad faith on the part of petitioner. And since it
refused to bargain, without valid and sufficient cause, the Secretary in the exercise of his powers
under Article 263(i) of the Labor Code to decide and resolve labor disputes, properly granted the
wage increase and imposed the union shop provision.
Notwithstanding that respondent Secretary did not act with grave abuse of discretion in
issuing the challenged Orders, we cannot ignore the supervening event which occurred
after December 15, 1991, i.e., the subsequent permanent cessation of petition of petitioner
on account of losses. Thus, despite the absence of grave abuse of discretion on the part of the
respondent Secretary, this Court cannot impose upon petitioner the directive to enter into a
new CBA with the union for the very simple reason that to do so would be to compel

petitioner to continue its business when it had already decided to close shop, and that
would be judicial tyranny on our part.
From Atty. Renes^^
Posted October 23, 2012 by vbdiaz in LABOR LAW

ORIENTAL vs. SEC. OF LABOR Leave a comment


G.R. No. 116751 August 28, 1998
ORIENTAL TIN CAN LABOR UNION, petitioner,
vs.
SECRETARY OF LABOR AND EMPLOYMENT, ORIENTAL TIN CAN WORKERS
UNION FEDERATION OF FREE WORKERS [OTCWU-FFW] and ORIENTAL TIN
CAN AND METAL SHEET MANUFACTURING,respondents.
G.R. No. 116779 August 28, 1998
ORIENTAL TIN CAN AND METAL SHEET MANUFACTURING CO., INC., petitioner,
vs.
HON. BIENVENIDO E. LAGUESMA, UNDERSECRETARY OF LABOR AND
EMPLOYMENT, ORIENTAL TIN CAN WORKERS UNION FFW and ORIENTAL
TIN CAN LABOR UNION, respondents.
FACTS: Oriental Tin Can and Metal Sheet Manufacturing Company, Inc. (the company) is
engaged in the manufacture of tin can containers and metal sheets. On March 3, 1994, it entered
into a collective bargaining agreement (CBA) with petitioner Oriental Tin Can Labor Union
(OTCLU) as the existing CBA was due to expire on April 15, 1994. Four days later, 248 of the
companys rank-and-file employees authorized the Federation of Free Workers (FFW) to file a
petition for certification election. On March 10, 1994, however, this petition was repudiated via a
written waiver by 115 of the signatories who, along with other employees totalling 897, ratified
the CBA on the same date.
On March 18, 1994, armed with Charter Certificate No. IV-MEE-089, respondent Oriental Tin
Can Workers Union Federation of Free Workers (OTCWU-FFW) filed a petition for
certification election with the National Capital Region office of the Department of Labor and
Employment (DOLE), pursuant to Article 256 of the Labor Code. Purporting to represent the
regular rank-and-file employees of the company, the petition was accompanied by the authentic
signatures of 25% of the employees/workers in the bargaining unit.
The above petition for certification elections was opposed by the OTCLU. For its part, the
company filed a comment alleging inter alia that the new CBA was ratified by 897 out of
the 1,020 rank-and-file employees within the bargaining unit. The OTCLU then filed a
motion to dismiss and/or position paper reiterating its position that the petition did not comply

with the 25% signature requirement and maintaining that the new CBA was a bar to a
certification election.
The certification election was allowed.
ISSUE: WON the company has a personality to challenge the conduct of a certification
elections.
HELD: NONE
It is a well-established rule that certification elections are exclusively the concern of employees;
hence, the employer lacks the legal personality to challenge the same. Law and policy demand
that employers take a strict, hands-off stance in certification elections. The bargaining
representative of employees should be chosen free from any extraneous influence of
management. A labor bargaining representative, to be effective, must owe its loyalty to the
employees alone and to no other.
The only instance when an employer may concern itself with employee representation activities
is when it has to file the petition for certification election because there is no existing CBA in the
unit and it was requested to bargain collectively, pursuant to Article 258 of the Labor code. After
filing the petition, the role of the employer ceases and it becomes a mere bystander. The
companys interference in the certification election below by actively opposing the same is
manifestly uncalled-for and unduly creates a suspicion that it intends to establish a company
union.
The designation or selection of the bargaining representative without, however, going
through the process set out by law for the conduct of a certification election applies only
when representation is not in issue. There is no problem if a union is unanimously chosen by a
majority of the employees as their bargaining representative, but a question of representation
arising from the presence of more than one union in a bargaining unit aspiring to be the
employees representative, can only be resolved by holding a certification election under the
supervision of the proper government authority.
NOTE: It is uncontroverted that the petition for certification election in this case was filed on
March 18, 1994, twenty-eight days before the expiration of the existing CBA on April 15, 1994,
and well within the 60-day period provided for by the Code. The OTCLU, however, is concerned
with the effect of the employees ratification of the new CBA on the timely filing of the petition
for certification election. Would such ratification nullify the petition?
The law dictates a negative reply. The filing of a petition for certification election during the 60day freedom period gives rise to a representation case that must be resolved even though a new
CBA has been entered into within that period. This is clearly provided for in the aforequoted
Section 4, Rule V, Book V of the Omnibus Rules Implementing the Labor Code. The reason
behind this rule is obvious. A petition for certification election is not necessary where the
employees are one in their choice of a representative in the bargaining process. Moreover, said
provision of the Omnibus Rules manifests the intent of the legislative authority to allow, if not

encourage, the contending unions in a bargaining unit to hold a certification election during the
freedom period. The agreement prematurely signed by the union and the company during the
freedom period does not affect the petition for certification election filed by another union.
As regards the 25% support requirement, the same has been met. As previously held by the SC,
once the required percentage requirement has been reached, the employees withdrawal from
union membership (waiver in this case) taking place after the filing of the petition for
certification election will not affect the petition. On the contrary, the presumption arises that the
withdrawal was not free but was procured through duress, coercion or for a valuable
consideration. Hence, the subsequent disaffiliation of the 6 employees from the union will not be
counted against or deducted from the previous number who had signed up for certification
From Atty. Renes^^
Posted October 23, 2012 by vbdiaz in LABOR LAW

GMC vs. CA Leave a comment


GENERAL MILLING CORPORATION vs HON. COURT OF APPEALS, GENERAL
MILLING CORPORATION INDEPENDENT LABOR UNION (GMC-ILU), and RITO
MANGUBAT
G.R. No. 146728

February 11, 2004

FACTS: In its two plants located at Cebu City and Lapu-Lapu City, petitioner General Milling
Corporation (GMC) employed 190 workers. They were all members of private respondent
General Milling Corporation Independent Labor Union. On April 28, 1989, GMC and the union
concluded a collective bargaining agreement (CBA) which included the issue of representation
effective for a term of three years. The day before the expiration of the CBA, the union sent
GMC a proposed CBA, with a request that a counter-proposal be submitted within ten (10) days.
However, GMC had received collective and individual letters from workers who stated that they
had withdrawn from their union membership, on grounds of religious affiliation and personal
differences. Believing that the union no longer had standing to negotiate a CBA, GMC did not
send any counter-proposal.
On December 16, 1991, GMC wrote a letter to the unions officers, Rito Mangubat and Victor
Lastimoso. The letter stated that it felt there was no basis to negotiate with a union which no
longer existed, but that management was nonetheless always willing to dialogue with them on
matters of common concern and was open to suggestions on how the company may improve its
operations. In answer, the union officers wrote a letter dated December 19, 1991 disclaiming any
massive disaffiliation or resignation from the union and submitted a manifesto, signed by its
members, stating that they had not withdrawn from the union.
NLRC held that the action of GMC in not negotiating was ULP.

ISSUE: WON the company (GMC) should have entered into collective bargaining with the
union
HELD: The law mandates that the representation provision of a CBA should last for five
years. The relation between labor and management should be undisturbed until the last 60
days of the fifth year. Hence, it is indisputable that when the union requested for a renegotiation
of the economic terms of the CBA on November 29, 1991, it was still the certified collective
bargaining agent of the workers, because it was seeking said renegotiation within five (5)
years from the date of effectivity of the CBA on December 1, 1988. The unions proposal
was also submitted within the prescribed 3-year period from the date of effectivity of the
CBA, albeit just before the last day of said period. It was obvious that GMC had no valid
reason to refuse to negotiate in good faith with the union. For refusing to send a counterproposal to the union and to bargain anew on the economic terms of the CBA, the company
committed an unfair labor practice under Article 248 of the Labor Code.
ART. 253-A. Terms of a collective bargaining agreement. Any Collective Bargaining
Agreement that the parties may enter into shall, insofar as the representation aspect is
concerned, be for a term of five (5) years. No petition questioning the majority status of the
incumbent bargaining agent shall be entertained and no certification election shall be conducted
by the Department of Labor and Employment outside of the sixty-day period immediately before
the date of expiry of such five year term of the Collective Bargaining Agreement. All other
provisions of the Collective Bargaining Agreement shall be renegotiated not later than three (3)
years after its execution.
ART. 248. Unfair labor practices of employers. It shall be unlawful for an employer to
commit any of the following unfair labor practice:
(g) To violate the duty to bargain collectively as prescribed by this Code;
Under Article 252 abovecited, both parties are required to perform their mutual obligation to
meet and convene promptly and expeditiously in good faith for the purpose of negotiating an
agreement. The union lived up to this obligation when it presented proposals for a new CBA to
GMC within three (3) years from the effectivity of the original CBA. But GMC failed in its duty
under Article 252. What it did was to devise a flimsy excuse, by questioning the existence of the
union and the status of its membership to prevent any negotiation.
ART. 250. Procedure in collective bargaining. The following procedures shall be observed in
collective bargaining:
(a) When a party desires to negotiate an agreement, it shall serve a written notice upon the other
party with a statement of its proposals. The other party shall make a reply thereto not later than
ten (10) calendar days from receipt of such notice.
GMCs failure to make a timely reply to the proposals presented by the union is indicative of its
utter lack of interest in bargaining with the union. Its excuse that it felt the union no longer
represented the workers, was mainly dilatory as it turned out to be utterly baseless.

Failing to comply with the mandatory obligation to submit a reply to the unions proposals, GMC
violated its duty to bargain collectively, making it liable for unfair labor practice.
From Atty. Renes^^
Posted October 23, 2012 by vbdiaz in LABOR LAW

THE INSULAR LIFE ASSURANCE CO., LTD.,


EMPLOYEES ASSOCIATION-NATU, FGU INSURANCE
GROUP WORKERS and EMPLOYEES ASSOCIATIONNATU, and INSULAR LIFE BUILDING EMPLOYEES
ASSOCIATION-NATU vs. THE INSULAR LIFE
ASSURANCE CO., LTD., FGU INSURANCE GROUP,
JOSE M. OLBES and COURT OF
INDUSTRIAL RELATIONS Leave a comment
G.R. No. L-25291 January 30, 1971
THE INSULAR LIFE ASSURANCE CO., LTD., EMPLOYEES ASSOCIATION-NATU,
FGU INSURANCE GROUP WORKERS and EMPLOYEES ASSOCIATION-NATU, and
INSULAR LIFE BUILDING EMPLOYEES ASSOCIATION-NATU
vs.
THE INSULAR LIFE ASSURANCE CO., LTD., FGU INSURANCE GROUP, JOSE M.
OLBES and COURT OF INDUSTRIAL RELATIONS
FACTS: The Insular Life Assurance Co., Ltd., Employees Association-NATU, FGU Insurance
Group Workers & Employees Association-NATU, and Insular Life Building Employees
Association-NATU (hereinafter referred to as the Unions), while still members of the Federation
of Free Workers (FFW), entered into separate CBAs with the Insular Life Assurance Co., Ltd.
and the FGU Insurance Group (hereinafter referred to as the Companies).
Two of the lawyers of the Unions then were Felipe Enaje and Ramon Garcia; the latter was
formerly the secretary-treasurer of the FFW and acting president of the Insular Life/FGU unions
and the Insular Life Building Employees Association. Garcia, as such acting president, in a
circular issued in his name and signed by him, tried to dissuade the members of the Unions from
disaffiliating with the FFW and joining the National Association of Trade Unions (NATU), to no
avail.
Enaje and Garcia soon left the FFW and secured employment with the Anti-Dummy Board of the
Department of Justice. Thereafter, the Companies hired Garcia in the latter part of 1956 as
assistant corporate secretary and legal assistant in their Legal Department. Enaje was hired as

personnel manager of the Companies, and was likewise made chairman of the negotiating panel
for the Companies in the collective bargaining with the Unions.
Unions jointly submitted proposals to the Companies; negotiations were conducted on the
Unions proposals, but these were snagged by a deadlock on the issue of union shop, as a result
of which the Unions filed on January 27, 1958 a notice of strike for deadlock on collective
bargaining. The issue was dropped subsequently (in short, nagkasundo). But, the parties
negotiated on the labor demands but with no satisfactory result due to a stalemate on the matter
of salary increases.
Meanwhile, 87 unionists were reclassified as supervisors without increase in salary nor in
responsibility while negotiations were going on in the Department of Labor after the notice
to strike was served on the Companies. These employees resigned from the Unions.
On May 21, 1958 the Companies through their acting manager and president, sent to each
of the strikers a letter (exhibit A) quoted verbatim as follows:
We recognize it is your privilege both to strike and to conduct picketing.
However, if any of you would like to come back to work voluntarily, you may:
1. Advise the nearest police officer or security guard of your intention to do so.
2. Take your meals within the office.
3. Make a choice whether to go home at the end of the day or to sleep nights at the office
where comfortable cots have been prepared.
4. Enjoy free coffee and occasional movies.
5. Be paid overtime for work performed in excess of eight hours.
6. Be sure arrangements will be made for your families.
7. The decision to make is yours whether you still believe in the motives of the strike or
in the fairness of the Management.

Unions, however, continued on strike, with the exception of a few unionists who were convinced
to desist by the aforesaid letter
From the date the strike was called on May 21, 1958, until it was called off on May 31, 1958,
some management men tried to break thru the Unions picket lines xxx succeeded in
penetrating the picket lines in front of the Insular Life Building, thus causing injuries to the
picketers and also to the strike-breakers due to the resistance offered by some picketers.

Alleging that some non-strikers were injured and with the use of photographs as evidence,
the Companies then filed criminal charges against the strikers with the City Fiscals Office
of Manila.xxx
Another letter was sent by the company to the individual strikers:
The first day of the strike was last 21 May 1958.
Our position remains unchanged and the strike has made us even more convinced of our
decision.
We do not know how long you intend to stay out, but we cannot hold your positions open for
long. We have continued to operate and will continue to do so with or without you.
If you are still interested in continuing in the employ of the Group Companies, and if there are no
criminal charges pending against you, we are giving you until 2 June 1958 to report for work at
the home office. If by this date you have not yet reported, we may be forced to obtain your
replacement.
Before, the decisions was yours to make.
So it is now.
Incidentally, all of the more than 120 criminal charges filed against the members of the
Unions, except 3, were dismissed by the fiscals office and by the courts. These three cases
involved slight physical injuries against one striker and light coercion against two others.
At any rate, because of the issuance of the writ of preliminary injunction against them as well as
the ultimatum of the Companies giving them until June 2, 1958 to return to their jobs or else be
replaced, the striking employees decided to call off their strike and to report back to work on
June 2, 1958.
* However, before readmitting the strikers, the Companies required them not only to secure
clearances from the City Fiscals Office of Manila but also to be screened by a management
committee among the members of which were Enage and Garcia. The screening committee
initially rejected 83 strikers with pending criminal charges. However, all non-strikers with
pending criminal charges which arose from the breakthrough incident were readmitted
immediately by the Companies without being required to secure clearances from the fiscals
office. Subsequently, when practically all the strikers had secured clearances from the
fiscals office, the Companies readmitted only some but adamantly refused readmission to
34 officials and members of the Unions who were most active in the strike, on the ground
that they committed acts inimical to the interest of the respondents, without however
stating the specific acts allegedly committed. Some 24 of the above number were ultimately
notified months later that they were being dismissed retroactively as of June 2, 1958 and given
separation pay checks computed under Rep. Act 1787, while others (ten in number) up to now
have not been readmitted although there have been no formal dismissal notices given to them.

CIR prosecutor filed a complaint for unfair labor practice against the Companies under Republic
Act 875. The complaint specifically charged the Companies with (1) interfering with the
members of the Unions in the exercise of their right to concerted action, by sending out
individual letters to them urging them to abandon their strike and return to work, with a promise
of comfortable cots, free coffee and movies, and paid overtime, and, subsequently, by warning
them that if they did not return to work on or before June 2, 1958, they might be replaced; and
(2) discriminating against the members of the Unions as regards readmission to work after the
strike on the basis of their union membership and degree of participation in the strike.
ISSUE: Whether or not respondent company is guilty of ULP
HELD: YES
The act of an employer in notifying absent employees individually during a strike following
unproductive efforts at collective bargaining that the plant would be operated the next day and
that their jobs were open for them should they want to come in has been held to be an unfair
labor practice, as an active interference with the right of collective bargaining through
dealing with the employees individually instead of through their collective bargaining
representatives.
Although the union is on strike, the employer is still under obligation to bargain with the union
as the employees bargaining representative.
Individual solicitation of the employees or visiting their homes, with the employer or his
representative urging the employees to cease union activity or cease striking, constitutes unfair
labor practice. All the above-detailed activities are unfair labor practices because they tend to
undermine the concerted activity of the employees, an activity to which they are entitled free
from the employers molestation.
Indeed, when the respondents offered reinstatement and attempted to bribe the strikers with
comfortable cots, free coffee and occasional movies, overtime pay for work performed in
excess of eight hours, and arrangements for their families, so they would abandon the strike
and return to work, they were guilty of strike-breaking and/or union-busting and, consequently,
of unfair labor practice. It is equivalent to an attempt to break a strike for an employer to offer
reinstatement to striking employees individually, when they are represented by a union, since the
employees thus offered reinstatement are unable to determine what the consequences of
returning to work would be.
ULP also: (super short cut na to) Hiring of Enage and Garcia with attractive compensations;
respondents reclassified 87 employees as supervisors without increase in salary or in
responsibility, in effect compelling these employees to resign from their unions; respondents,
thru their president and manager, respondent Jose M. Olbes, brought three truckloads of nonstrikers and others, escorted by armed men, who, despite the presence of eight entrances to the
three buildings occupied by the Companies, entered thru only one gate less than two meters wide
and in the process, crashed thru the picket line posted in front of the premises of the Insular Life
Building. This resulted in injuries on the part of the picketers and the strike-breakers;

respondents brought against the picketers criminal charges, only three of which were not
dismissed, and these three only for slight misdemeanors. As a result of these criminal actions, the
respondents were able to obtain an injunction from the court of first instance restraining the
strikers from stopping, impeding, obstructing, etc. the free and peaceful use of the Companies
gates, entrance and driveway and the free movement of persons and vehicles to and from, out
and in, of the Companies buildings.
Verily, the above actuations of the respondents before and after the issuance of the letters, exhibit
A and B, yield the clear inference that the said letters formed of the respondents scheme to
preclude if not destroy unionism within them.
II. The respondents did not merely discriminate against all the strikers in general. They separated
the active from the less active unionists on the basis of their militancy, or lack of it, on the picket
lines. Unionists belonging to the first category were refused readmission even after they were
able to secure clearances from the competent authorities with respect to the criminal charges
filed against them.
It is noteworthy that perhaps in an anticipatory effort to exculpate themselves from charges of
discrimination in the readmission of strikers returning to work the respondents delegated the
power to readmit to a committee.
III. Anent the third assignment of error, the record shows that not a single dismissed striker was
given the opportunity to defend himself against the supposed charges against him. As earlier
mentioned, when the striking employees reported back for work on June 2, 1958, the respondents
refused to readmit them unless they first secured the necessary clearances; but when all, except
three, were able to secure and subsequently present the required clearances, the respondents still
refused to take them back.
Indeed, the individual cases of dismissed officers and members of the striking unions do not
indicate sufficient basis for dismissal.
From Atty. Bayani^^

Posted October 23, 2012 by vbdiaz in LABOR LAW

CREA vs. BRILLANTES and CALTEX


(PHILIPPINES), Inc. Leave a comment

G.R. No. 123782 September 16, 1997


CALTEX REFINERY EMPLOYEES ASSOCIATION (CREA) vs.HON. JOSE S.
BRILLANTES, in his capacity as Acting Secretary of the Department of Labor and
Employment, and CALTEX (PHILIPPINES), Inc.
FACTS: Anticipating the expiration of their CBA on July 31, 1995, petitioner and private
respondent negotiated the terms and conditions of employment to be contained in a new CBA.
The negotiation between the two parties was participated in by the NCMB and the Office of the
Secretary of Labor and Employment. Some items in the new CBA were amicably arrived at and
agreed upon, but others were unresolved.
To settle the unresolved issues, eight meetings between the parties were conducted. Because the
parties failed to reach any significant progress in these meetings, petitioner declared a deadlock.
On July 24, 1995, petitioner filed a notice of strike. 6 conciliation meetings conducted by the
NCMB failed, failed. Marathon meetings at the plant level, but this remedy proved also
unavailing.
Secretary assumed jurisdiction and ordered Accordingly, any strike or lockout, whether actual
or intended, is hereby enjoined.xxx But the members of petitioner defied them and continued
their mass action (despite repeated orders)
Thereafter, the contending parties filed their position papers pertaining to unresolved issues.
Because of the strike, private respondent terminated the employment of some officers of
petitioner union. The legality of these dismissals brought additional contentious issues.
Again, the parties tried to resolve their differences through conciliation. Failing to come to any
substantial agreement, the parties decided to refer the problem to the secretary of labor and
employment.
(guys, what the SC did in this case, is to decide upon the conflicting issues ng parties with
regard dun sa CBA kaya nilagay ko na lang lahat)
ISSUE: (1) petitioner questions public respondents resolution of five issues in the CBA,
specifically on wage increase, union security clause, retirement benefits or application of the new
retirement plan, signing bonus and grievance and arbitration machineries; and (2) Whether or
not the Honorable Secretary of Labor and Employment committed grave abuse of discretion in
resolving the instant labor dispute.
HELD: The petition is partly meritorious.
1. Wage Increase. Petitioner maintains that the salaries of Shell Refinery employees be used as a
reference point in upgrading the compensation of private respondents employees because
these two companies are in the same industry and their refineries are both in Batangas. Thus,
the wage increase of petitioners members should be 15%/15%/15%. Private respondent

counters with a proposed 9% 7% 7% increase for the same period with automatic adjustment
should the increase fall short of the inflation rate.
The alleged similarity in the situation of Caltex and Shell cannot be considered a valid ground
for a demand of wage increase, in the absence of a showing that the two companies are also
similar in substantial aspects, as discussed above.
True, union members have the right to demand wage increases through their collective
force; but it is equally cogent that they should also be able to justify an appreciable
increase in wages. We observe that private respondents detailed allegations on productivity are
unrebutted. It is noteworthy that petitioner ignored this argument of private respondent and based
its demand for wage increase not on the ground that they were as productive as the Shell
employees. Thus, we cannot attribute grave abuse of discretion to public respondent.
2. Union Security Clause. Petitioner argues that in spite of the provisions on the union security
clause, it may expel a member only on any of three grounds: non-payment of dues, subversion,
or conviction for a crime involving moral turpitude. If the employees act does not constitute any
of these three grounds, the member would continue to be employed by private respondent. Thus,
the disagreement between petitioner and private respondent on this issue is not only procedural
but also substantial.
We agree with petitioner. The disagreement between petitioner and private respondent on
the union security clause should have been definitively resolved by public respondent. The
labor secretary should take cognizance of an issue which is not merely incidental to but
essentially involved in the labor dispute itself, or which is otherwise submitted to him for
resolution. The secretary of labor assumed jurisdiction over this labor dispute in an industry
indispensable to national interest, precisely to settle once and for all the disputes over which he
has jurisdiction at his level. In not performing his duty, the secretary of labor committed a grave
abuse of discretion.
3. New Retirement Plan. Petitioner contends that 40 of its members who are still covered by
the Old Retirement Plan because they were not able to exercise the option to shift to the New
Retirement Plan, for one reason or another, when such option was given in the past are included
in the New Retirement Plan.
We hold that public respondent did not commit grave abuse of discretion in respecting the free
and voluntary decision of the employees in regard to the Provident Plan and the irrevocable onetime option provided for in the New Retirement Plan. Although the union has every right to
represent its members in the negotiation regarding the terms and conditions of their
employment, it cannot negate their wishes on matters which are purely personal and
individual to them. In this case, the forty employees freely opted to be covered by the Old Plan;
their decision should be respected. The company gave them every opportunity to choose, and
they voluntarily exercised their choice. The union cannot pretend to know better; it cannot
impose its will on them.

4. Grievance Machinery and Arbitration. Petitioner contends that public respondent derailed
the grievance and arbitration scheme proposed by the Union. Petitioners recommendation for a
single arbitrator is based on the proposition that if voluntary arbitration should be resorted to at
all, this recourse should entail the least possible expense.
No particular setup for a grievance machinery is mandated by law. Rather, Article 260 of the
Labor Code, as incorporated by RA 6715, provides for only a single grievance machinery in the
company to settle problems arising from interpretation or implementation of their CBA and
those arising from the interpretation or enforcement of company personnel policies.
We believe that the procedure described by public respondent sufficiently complies with the
minimum requirement of the law. Public respondent even provided for two steps in hearing
grievances prior to their referral to arbitration. The parties will decide on the number of
arbitrators who may hear a dispute only when the need for it arises. Even the law itself does not
specify the number of arbitrators. . In this matter, cost is not the only consideration; full
deliberation on the issues is another, and it is best accomplished in a hearing conducted by
three arbitrators. In effect, the parties are afforded the latitude to decide for themselves the
composition of the grievance machinery as they find appropriate to a particular situation. At
bottom, we cannot really impute grave abuse of discretion to public respondent on this issue.
5. Signing Bonus. Petitioner asseverates that the signing bonus is an existing benefit embodied
in the old CBA. 42 It explains that public respondent erred in removing the award of a signing
bonus xxx
Although proposed by petitioner, 45 the signing bonus was not accepted by private
respondent. 46 Besides, a signing bonus is not a benefit which may be demanded under the law.
Rather, it is now claimed by petitioner under the principle of maintenance of existing benefits
of the old CBA. However, as clearly explained by private respondent, a signing bonus may
not be demanded as a matter of right. If it is not agreed upon by the parties or unilaterally
offered as an additional incentive by private respondent, the condition for awarding it must
be duly satisfied. In the present case, the condition sine qua non for its grant a non-strike
was not complied with. In fact, private respondent categorically sated in its counter-proposal
to the exclusion of those agreed upon before that the new CBA would constitute the only
agreement between the parties.
II. In the present case, the foregoing requirement has been sufficiently met. Petitioners claim of
grave abuse of discretion is anchored on the simple fact that public respondent adopted largely
the proposals of private respondent. It should be understood that bargaining is not equivalent to
an adversarial litigation where rights and obligations are delineated and remedies applied. It is
simply a process of finding a reasonable solution to a conflict and harmonizing opposite
positions into a fair and reasonable compromise. When parties agree to submit unresolved
issues to the secretary of labor for his resolution, they should not expect their positions to
be adopted in toto. It is understood that they defer to his wisdom and objectivity in insuring
industrial peace. And unless they can clearly demonstrate bias, arbitrariness,
capriciousness or personal hostility on the part of such public officer, the Court will not
interfere or substitute the said officers judgment with its own.

From Atty. Bayani^^

Posted October 23, 2012 by vbdiaz in LABOR LAW

WARREN MANUFACTURING WORKERS UNION


V. BLR Leave a comment
WARREN MANUFACTURING WORKERS UNION V. BLR
This is a petition for review on certiorari with prayer for a preliminary injunction and/or the
issuance of a restraining order seeking to set aside: (1) Order of the Med-Arbiter dated August
18,1986 and (2) the Resolution dated October 7, 1986 of the Officer-in-Charge of the Bureau of
Labor dismissing the appeals of Warren Manufacturing Corporation and herein petitioner.
This certification case had its inception in an intra-union rivalry between the petitioner and
the respondent Philippine Agricultural, Commercial and Industrial Workers Union
(PACIWU for short) since 1985.
Facts:
This certification case had its inception in an intra-union rivalry between the petitioner and
the respondent Philippine Agricultural, Commercial and Industrial Workers Union
(PACIWU for short) since 1985.
On June 13,1985, PACIWU filed a petition for certification election, alleging compliance with
the jurisdictional requirements.
On July 7, 1985, respondent thru counsel filed a motion to dismiss the petition on the ground that
there exist a C.BA between the respondent and the Warren Mfg. Union which took effect
upon its signing on July 16, 1985 and to expire on July 31, 1986.
While the petition was under hearing, PACIWU filed a Notice of Strike and on conciliation
meeting, a Return-to-Work Agreement was signed on July 25,1985, stipulating, among others, as
follows:
To resolve the issue of union representation at Warren Mfg- Corp. parties have agreed to the
holding of a consent election among the rank and file on August 25, 1985 at the premises of
the company to be supervised by MOLE.

It is cleanly understood that the certified union in the said projected election shall respect and
administer the existing CBA at the company until its expiry date on July 31, 1986.
On 12 August 1985, an Order was issued by this Office, directing that a consent election be held
among the rank and file workers of the company.
On August 25, 1985, said consent election was held, and yielded the following results:
PACIWU-94
WMWU-193
Feeling aggrieved, however, PACIWU filed an Election Protest.
In December, 1985 a Notice of Strike was again filed by the union this time with the Valenzuela
branch office of this Ministry, and after conciliation, the parties finally agreed.On the basis of a
Joint Motion to Dismiss filed by the parties, the Election Protest filed by the PACIWU was
ordered dismissed.
On June 5, 1986, the PACIWU filed a petition for certification election followed by the filing of
a petition for the same purposes by the Samahan ng Manggagawa sa Warren Manufacturing
Corporation-Alliance of Nationalist and Genuine Labor Organizations (Anglo for short) which
petitions were both opposed by Warren Manufacturing Corporation on the grounds that neither
petition has 30% support; that both are barred by the one-year no certification election law and
the existence of a duly ratified CBA. The therein respondent, therefore, prayed that the petitions
for certification election be dismissed.
As above stated, the Med-Arbiter of the National Capital Region, Ministry of Labor and
Employment, ordered on August 8, t 986 the holding of a certification election within twenty
20) days from receipt to determine the exclusive bargaining representative of all the rank and file
employees of the Warren se Manufacturing Corporation, with the above-mentioned choices.
Both Warren Manufacturing Corporation and petitioner herein filed separate motions,
treated as appeals by the Bureau of Labor Relations, which dismissed the same for lack of
merit.
Hence, this petition.
This petition was filed solely by the Warren Manufacturing Workers Union, with the company
itself opting not to appeal.
The Second Division of this Court in the resolution of November 3, 1986 without giving due
course to the petition, required the respondents to comment and issued the temporary,
restraining order prayed for .
Issues:

A. The holding of a certification election at the bargaining unit is patently premature and illegal.
B. The petition filed by private respondents do not have the statutory 30% support requirement.
C. Petitioner was denied administrative due process when excluded from med-arbitration
proceedings.
The petition is devoid of merit.
A.
Petitioners contention is anchored on the following grounds:
Section 3, Rule V of the Implementing Rules and Regulations of the Labor Code provides,
among others:
however no certification election may be held within one (1) year from the date of the
issuance of the declaration of a final certification result.
and
Article 257, Title VII, Book V of the Labor Code provides:
No certification election issue shall be entertained by the Bureau in any CBA existing between
the employer and a legitimate labor organization.
Otherwise stated, petitioner invoked the one-year no certification election rule and the principle
of the Contract Bar Rule.
This contention is untenable.
The records show that petitioner admitted that what was held on August 25,1985 at the
Companys premises and which became the root of this controversy, was a consent election and
not a certification election (Emphasis supplied). As correctly distinguished by private
respondent, a consent election is an agreed one, its purpose being merely to determine the
issue of majority representation of all the workers in the appropriate collective bargaining
unit while a certification election is aimed at determining the sole and exclusive bargaining
agent of all the employees in an appropriate bargaining unit for the purpose of collective
bargaining. From the very nature of consent election, it is a separate and distinct process and has
nothing to do with the import and effect of a certification election. Neither does it shorten the
terms of an existing CBA nor entitle the participants thereof to immediately renegotiate an
existing CBA although it does not preclude the workers from exercising their right to choose
their sole and exclusive bargaining representative after the expiration of the sixty (60) day
freedom period. In fact the Med-Arbiter in the Return to Work Agreement signed by the parties
emphasized the following:

To resolve the issue of union representation at Warren Mfg. Corp., parties have agreed to the
holding of a consent election among the rank and file on August 25,1985 at the premises of the
company to be supervised by the Ministry of Labor and Employment ..
It is clearly understood that the certified union in the said projected election shall respect and
administer the existing CBA at the company until its expiry date on July 31, 1986. (Rollo, pp. 46,
48-49).
It is, therefore, unmistakable that the election thus held on August 25, 1985 was not for the
purpose of determining which labor union should be the bargaining representative in the
negotiation for a collective contract, there being an existing CBA yet to expire on July 31,
1986; but only to determine which labor union shag administer the said existing contract.
Accordingly, the following provisions of the New Labor Code apply:
ART. 254. Duty to bargain collectively when there exists a CBA.When there is a CBA, the
duty to bargain collectively shall also mean that neither party shall terminate or modify the
agreement at least sixty (60) days prior to its expiration date. It shall be the duty of both parties
to keep the status quo and to continue in full force and effect the terms and conditions of the
existing agreement during the 60-day period and/or until a new agreement is reached by the
parties.
Corollary to the above, Article 257 of the New Labor Code expressly states that No certification
election issue shall be entertained if a collective agreement which has been submitted in
accordance with Article 231 of this Code exists between the employer and a legitimate labor
organization except within sixty (60) days prior to the expiration of the life of such certified
CBA. (Rollo, pp. 83-84)
Thus, as stated by this Court in General Textiles Allied Workers Association v. the Director of the
Bureau of labor Relations (84 SCRA 430 [19781) there should be no obstacle to the right of the
employees to petition for a certification election at the proper time. that is, within 60 days prior
to the expiration of the three year period
Finally, such premature agreement entered into by the petitioner and the Company on June
2, 1986 does not adversely affect the petition for certification election filed by respondent
PACIWU (Rollo, p. 85).
Section 4, Rule V, Book V of the Omnibus Rules Implementing the Labor Code clearly provides:
Section 4. Effect of Early Agreement.There representation case shall not, however, be
adversely affected by a collective agreement submitted before or during the last sixty days of a
subsisting agreement or during the pendency of the representation case.
As aforestated, the existing CBA was due to expire on July 31, 1 986. The Med-Arbiter found
that a sufficient number of employees signified their consent to the filing of the petition and 107
employees authorized intervenor to file a motion for intervention. Otherwise stated, he found that

the petition and intervention were supported by more than 30% of the members of the bargaining
unit. In the light of these facts, Article 258 of the Labor Code makes it mandatory for the Bureau
of Labor Relations to conduct a certification election (Samahang Manggagawa ng Pacific Mills,
Inc. v. Noriel, et al., 134 SCRA 152 [1985]). In the case of Federation of Free Workers (Bisig ng
Manggagawa sa UTEX v. Noriel etc., et al., 86 SCRA 132 [1978]), this Court was even more
specific when it stated No administrative agency can ignore the imperative tone of the above
article. The language used is one of command. Once it has been verified that the petition for
certification election has the support of at least 30% of the employees in the bargaining unit, it
must be granted, The specific word used can yield no other meaning. It becomes under the
circumstances, mandatory
The finality of the findings of fact of the Med-Arbiter that the petition and intervention
filed in the case at bar were supported by 30% of the members of the workers is clear and
definite.
WHEREFORE, the instant Petition is DISMISSED.
From Atty. Manalundong^^
Posted October 23, 2012 by vbdiaz in LABOR LAW

ME-SHURN CORPORATION V. ME-SHURN


WORKERS UNION Leave a comment
ME-SHURN CORPORATION V. ME-SHURN WORKERS UNION
Before us is a Petition for Review1 under Rule 45 of the Rules of Court, seeking to annul the
Decision of the Court of Appeals (CA).
Facts:
On June 7, 1998, the regular rank and file employees of Me-Shurn Corporation organized MeShurn Workers Union-FSM, an affiliate of the February Six Movement (FSM).6 Respondent
union had a pending application for registration with the Bureau of Labor Relations (BLR)
through a letter dated June 11, 1998.
Ten days later, or on June 17, 1998, petitioner corporation started placing on forced leave all the
rank and file employees who were members of the unions bargaining unit.
On June 23, 1998, respondent union filed a Petition for Certification Election with the MedArbitration Unit of the Department of Labor and Employment (DOLE), Regional Office No. 3.
Instead of filing an answer to the Petition, the corporation filed on July 27, 1998, a comment
stating that it would temporarily lay off employees and cease operations, on account of its
alleged inability to meet the export quota required by the Board of Investment.

While the Petition was pending, 184 union members allegedly submitted a retraction/withdrawal
thereof on July 14, 1998. As a consequence, the med-arbiter dismissed the Petition. On May 7,
1999, DOLE Undersecretary Rosalinda Dimapilis-Baldoz granted the unions appeal and ordered
the holding of a certification election among the rank and file employees of the corporation.
Meanwhile, on August 4, 1998, respondent union filed a Notice of Strike against petitioner
corporation on the ground of unfair labor practice (illegal lockout and union busting). This matter
was docketed as Case No. NCMB-RO3-BEZ-NZ-08-42-98.
On August 31, 1998, Chou Fang Kuen (alias Sammy Chou, the other petitioner herein) and
Raquel Lamayra (the Filipino administrative manager of the corporation) imposed a precondition
for the resumption of operation and the rehiring of laid off workers. He allegedly required the
remaining union officers to sign an Agreement containing a guarantee that upon their return to
work, no union or labor organization would be organized. Instead, the union officers were to
serve as mediators between labor and management. After the signing of the Agreement, the
operations of the corporation resumed in September 1998.
On November 5, 1998, the union reorganized and elected a new set of officers. Respondent
Rosalina Cruz was elected president. Thereafter, it filed two Complaints docketed as NLRC Case
Nos. RAB-III-11-9586-98 and RAB-III-09-0322-99. These cases were consolidated and assigned
to Labor Arbiter Henry Isorena for compulsory arbitration. Respondents charged petitioner
corporation with unfair labor practice, illegal dismissal, underpayment of wages and
deficiency in separation pay, for which they prayed for damages and attorneys fees.
The corporation countered that because of economic reversals, it was compelled to close and
cease its operations to prevent serious business losses.
Petitioner corporation questioned the legality of the representation of respondent union.
Allegedly, it was not the latter, but the Me-Shurn Independent Employees Union with
Christopher Malit as president that was recognized as the existing exclusive bargaining agent
of the rank and file employees and as the one that had concluded a Collective Bargaining
Agreement (CBA) with the corporation on May 19, 1999.16 Hence, the corporation asserted that
Undersecretary Dimapilis-Baldozs Decision ordering the holding of a certification election had
become moot and academic.
On the other hand, respondents contested the legality of the formation of the Me-Shurn
Independent Employees Union and petitioners recognition of it as the exclusive
bargaining agent of the employees. Respondents argued that the pendency of the
representation issue before the DOLE had barred the alleged recognition of the
aforementioned union.
Labor Arbiter Isorena dismissed the Complaints for lack of merit.
On appeal, the NLRC reversed the Decision of Labor Arbiter Isorena. Finding petitioners guilty
of unfair labor practice, the Commission ruled that the closure of the corporation shortly after
respondent union had been organized, as well as the dismissal of the employees, had been

effected under false pretenses. The true reason therefor was allegedly to bar the formation of
the union. Accordingly, the NLRC held that the illegally dismissed employees were entitled
to back wages.18
After the denial of their Motion for Reconsideration, petitioners elevated the cases to the CA via
a Petition for Certiorari under Rule 65.
Petitioners added that respondent unions personality to represent the affected employees had
already been repudiated by the workers themselves in the certification election conducted by the
DOLE. Pursuant to the Decision of Undersecretary Dimapilis-Baldoz in Case No. RO3 00 9806
RU 001, a certification election was held on September 7, 2000, at the premises of petitioner
corporation under the supervision of the DOLE.
Ruling of the Court of Appeals
The CA dismissed the Petition because of the failure of petitioners to submit sufficient proof of
business losses. It found that they had wanted merely to abort or frustrate the formation of
respondent union. The burden of proving that the dismissal of the employees was for a valid or
authorized cause rested on the employer.
The appellate court further affirmed the unions legal personality to represent the
employees. It held that (1) registration was not a prerequisite to the right of a labor
organization to litigate; and (2) the cases may be treated as representative suits, with
respondent union acting for the benefit of all its members.
Hence, this Petition.
Issues:
(1) Whether the dismissal of the employees of petitioner Meshurn Corporation is for an
authorized cause, and
(2) Whether respondents can maintain a suit against petitioners.
HELD:
The Petition lacks merit.
Validity of the Dismissal
The reason invoked by petitioners to justify the cessation of corporate operations was alleged
business losses. Yet, other than generally referring to the financial crisis in 1998 and to their
supposed difficulty in obtaining an export quota, interestingly, they never presented any report on
the financial operations of the corporation during the period before its shutdown. Neither did
they submit any credible evidence to substantiate their allegation of business losses.

We also take note of the allegation that after several years of attempting to organize a
union, the employees finally succeeded on June 7, 1998. Ten days later, without any valid
notice, all of them were placed on forced leave, allegedly because of lack of quota.
All these considerations give credence to their claim that the closure of the corporation was a
mere subterfuge, a systematic approach intended to dampen the enthusiasm of the union
members.
Third, as a condition for the rehiring of the employees, the union officers were made to sign an
agreement that they would not form any union upon their return to work. This move was contrary
to law.
Fourth, notwithstanding the Petition for Certification Election filed by respondents and despite
knowledge of the pendency thereof, petitioners recognized a newly formed union and hastily
signed with it an alleged Collective Bargaining Agreement. Their preference for the new union
was at the expense of respondent union. Moncada Bijon Factory v. CIR held that an employer
could be held guilty of discrimination, even if the preferred union was not company-dominated.
Fifth, petitioners were not able to prove their allegation that some of the employees contracts
had expired even before the cessation of operations. We find this claim inconsistent with their
position that all 342 employees of the corporation were paid their separation pay plus accrued
benefits in August 1998.
Sixth, proper written notices of the closure were not sent to the DOLE and the employees at least
one month before the effectivity date of the termination, as required under the Labor Code.
Notice to the DOLE is mandatory to enable the proper authorities to ascertain whether the
closure and/or dismissals were being done in good faith and not just as a pretext for evading
compliance with the employers just obligations to the affected employees. This requirement is
intended to protect the workers right to security of tenure. The absence of such requirement
taints the dismissal.
All these factors strongly give credence to the contention of respondents that the real reason
behind the shutdown of the corporation was the formation of their union. Note that, to constitute
an unfair labor practice, the dismissal need not entirely and exclusively be motivated by the
unions activities or affiliations. It is enough that the discrimination was a contributing factor. If
the basic inspiration for the act of the employer is derived from the affiliation or activities of the
union, the formers assignment of another reason, no matter how seemingly valid, is unavailing.
Legal Personality of Respondent Union
Neither are we prepared to believe petitioners argument that respondent union was not
legitimate. It should be pointed out that on June 29, 1998, it filed a Petition for Certification
Election. While this Petition was initially dismissed by the med-arbiter on the basis of a
supposed retraction, note that the appeal was granted and that Undersecretary Dimapilis-Baldoz
ordered the holding of a certification election.

The DOLE would not have entertained the Petition if the union were not a legitimate labor
organization within the meaning of the Labor Code. Under this Code, in an unorganized
establishment, only a legitimate union may file a petition for certification election.34 Hence,
while it is not clear from the record whether respondent union is a legitimate organization,
we are not readily inclined to believe otherwise, especially in the light of the pro-labor policies
enshrined in the Constitution and the Labor Code.
Verily, the union has the requisite personality to sue in its own name in order to challenge
the unfair labor practice committed by petitioners against it and its members. It would be
an unwarranted impairment of the right to self-organization through formation of labor
associations if thereafter such collective entities would be barred from instituting action in
their representative capacity.
Finally, in view of the discriminatory acts committed by petitioners against respondent
union prior to the holding of the the pending Petition for certification election the results
of that election cannot be said to constitute a repudiation by the affected employees of the
unions right to represent them in the present case.
WHEREFORE, the Petition is DENIED, and the assailed Decision AFFIRMED. Costs against
the certification election on September 27, 2000 acts that included their immediate grant of
exclusive recognition to another union as a bargaining agent despite petitioners.
SO ORDERED.
From Atty. Manalundong^^
Posted October 23, 2012 by vbdiaz in LABOR LAW

Sandowner Development Corp vs Drilon Leave a comment


Sandowner Development Corp vs Drilon
GR 82341

Facts:
Private respondent Hotel Mabuhay, Inc. offered to sell its assets and personal properties in the
premises to petitioner to which petitioner agreed. A deed of assignment was executed by
Mabuhay in favor of petitioner.
Respondent National Union of Workers in Hotel, Restaurant and Allied Services (NUWHRAIN
for short) picketed the premises of petitioner, barricaded the entrance to the leased premises and
denied petitioners officers, employees and guests free access to and egress from said premises.

Issue: WON the purchaser of the assets of an employer corporation can be considered a
successor employer of the latters employees.
Held:
The rule is that unless expressly assumed, labor contracts such as employment contracts and
collective bargaining agreements are not enforceable against a transferee of an enterprise, labor
contracts being in personam, thus binding only between the parties. This conclusion draws its
force from the right of an employer to select his employees and to decide when to engage them.
However, although the purchaser of the assets or enterprise is not legally bound to absorb in its
employ the employers of the seller of such assets or enterprise, the parties are liable to the
employees if the transaction between the parties is colored or clothed with bad faith.
In the case at bar, contrary to the claim of the public respondent that the transaction between
petitioner and Mabuhay was attended with bad faith, the court finds no cogent basis for such
contention. Thus, the absorption of the employees of Mabuhay may not be imposed on petitioner.
Petition Granted.

From Atty. Alba^^


Posted October 23, 2012 by vbdiaz in LABOR LAW

Rizal Cement Workers vs Madrigal Leave a comment


Rizal Cement Workers vs Madrigal
GR L-19767

Facts:
The petitioner Union staged a strike at the plant of the respondent Rizal Cement Co., Inc. in
Binangonan, Rizal. In the early morning of the following day, that is, on May 28, 1956, Candido
de Leon warehouseman-encargado at the Bodega Tanque, received a telephone call from one
Johnny de Leon, manager of the respondent Rizal Cement Co., Inc., with the information that the
Union staged a strike against the company on the previous day, May 27, 1956, in Binangonan,
Rizal De Leon further informed him that he should take precautionary measures in protecting
the properties of the company stored at the Bodega Tanque because of the strikers caused
damage to the factory in Binangonan and sabotage might occur. For this reason, he was advised
by the manager to request the members of the Union to stay meanwhile outside the premises of
the Bodega Tanque. What he did in the morning of May 28, 1956 was to station himself at the

gate of the compound. When the workers arrived for work at 7:00 a.m., he did not allow the 21
complaining workers who are members of the Union to enter the gate and allowed only those
who are not members of said Union.
Upon refusal of Candido de Leon to allow the complaining workers to work on that day, the
Union, sent a letter to the manager of the Bodega alleging discrimination. The manager sent a
reply denying such allegation.
Issue: WON there was discrimination against the employees who are not allowed to work
in the Bodega.
Held:
It is not herein controverted that the complainants were locked out or denied work by the
respondent Company. However, for the discrimination by reason of union membership to be
considered an unfair labor practice, the same must have been committed to courage or
discourage such membership in the union.
This cannot be said of the act of the Company complained of. As clearly established by the
evidence, its refusal to all complainants to work and requirement that the latter stay out of the
premises in the meantime was borne out of the Companys justified apprehension and fear that
sabotage might be committed in the warehouse where the products machinery and spare parts
were stored, as has been the case in Binangonan. It has never been shown that the act of the
Company was intended to induce the complainants to renounce their union-membership or as a
deterrent for non-members to affiliate therewith, nor as a retaliatory measure for activities in the
union or in furtherance of the cause of the union.
Decision affirmed.

From Atty. Alba^^


Posted October 23, 2012 by vbdiaz in LABOR LAW

Rivera vs Espiritu Leave a comment


Rivera vs Espiritu
GR 135547
Facts:
PAL was suffering from a difficult financial situation in 1998. It was faced with bankruptcy and
was forced to adopt a rehabilitation plan and downsized its labor force by more than 1/3. PALEA

(PAL Employees Association) went on a four-day strike to protest retrenchment measures in July
1998. PAL ceased operations on Sep 23, 1998.
PALEA board again wrote the President on Sep 28, 1998. Among others, it proposed the
suspension of the PAL-PALEA CBA for a period of ten years, subject to certain conditions.
PALEA members accepted such terms through a referendum on Oct 2, 1998. PAL resumed
domestic operations on Oct 7, 1998.
Seven officers and members of PALEA filed instant petition to annul the Sep 27, 1998 agreement
entered into between PAL and PALEA.
Issue: WON negotiations may be suspended for 10 years.
Held:
YES. CBA negotiations may be suspended for 10 years.
The assailed PAL-PALEA agreement was the result of voluntary collective bargaining
negotiations undertaken in the light of the severe financial situation faced by the employer, with
the peculiar and unique intention of not merely promoting industrial peace at PAL, but
preventing the latters closure.
There is no conflict between said agreement and Article 253-A of the Labor Code. CBA under
Article 253-A of the Labor Code has a two-fold purpose. One is to promote industrial stability
and predictability. Inasmuch as the agreement sought to promote industrial peace, at the PAL
during its rehabilitation, said agreement satisfied the first purpose of said article. The other
purpose is to assign specific timetable, wherein negotiations become a matter of right and
requirement. Nothing in Article 253-A prohibits the parties from waiving or suspending the
mandatory timetable and agreeing on the remedies to enforce the same.

From Atty. Alba^^


Posted October 23, 2012 by vbdiaz in LABOR LAW

PICOP Resources Inc vs Taneca Leave a comment


PICOP Resources Inc vs Taneca
GR 160828
Facts:
Respondents were regular rank-and-file employees of PRI and bona fide members
of Nagkahiusang Mamumuo sa PRI Southern Philippines Federation of Labor (NAMAPRI-

SPFL), which is the collective bargaining agent for the rank-and-file employees of petitioner
PRI. PRI has a CBA with NAMAPRI-SPFL. The CBA contained the following union security
provisions:

Article II- Union Security and Check-Off

Section 6. Maintenance of membership.

6.1 All employees within the appropriate bargaining unit who are members of the UNION at
the time of the signing of this AGREEMENT shall, as a condition of continued employment
by the COMPANY, maintain their membership in the UNION in good standing during the
effectivity of this AGREEMENT.

6.3 The COMPANY, upon the written request of the UNION and after compliance with the
requirements of the New Labor Code, shall give notice of termination of services of any
employee who shall fail to fulfill the condition provided in Section 6.1 and 6.2 of this Article

Atty. Fuentes sent a letter to the management of PRI demanding the termination of employees
who allegedly campaigned for, supported and signed the Petition for Certification Election of the
Federation of Free Workers Union (FFW) during the effectivity of the CBA. NAMAPRI-SPFL
considered said act of campaigning for and signing the petition for certification election of FFW
as an act of disloyalty and a valid basis for termination for a cause in accordance with its
Constitution and By-Laws, and the terms and conditions of the CBA, specifically Article II,
Sections 6.1 and 6.2 on Union Security Clause.

On October 16, 2000, PRI served notices of termination for causes to employees whom
NAMAPRIL-SPFL sought to be terminated on the ground of acts of disloyalty committed
against it when respondents allegedly supported and signed the Petition for Certification Election
of FFW before the freedom period during the effectivity of the CBA. A Notice dated October
21, 2000 was also served on the DOLE, Caraga Region.

Respondents then accused PRI of ULP.

Issue:
WON respondents were validly terminated.

Held:

Union security is a generic term, which is applied to and comprehends closed shop, union
shop, maintenance of membership, or any other form of agreement which imposes upon
employees the obligation to acquire or retain union membership as a condition affecting
employment. There is union shop when all new regular employees are required to join the union
within a certain period as a condition for their continued employment. There is maintenance of
membership shop when employees, who are union members as of the effective date of the
agreement, or who thereafter become members, must maintain union membership as a condition
for continued employment until they are promoted or transferred out of the bargaining unit, or
the agreement is terminated. A closed shop may be defined as an enterprise in which, by
agreement between the employer and his employees or their representatives, no person may be
employed in any or certain agreed departments of the enterprise unless he or she is, becomes,
and, for the duration of the agreement, remains a member in good standing of a union entirely
comprised of or of which the employees in interest are a part.

However, in terminating the employment of an employee by enforcing the union security clause,
the employer needs to determine and prove that: (1) the union security clause is applicable; (2)
the union is requesting for the enforcement of the union security provision in the CBA; and (3)
there is sufficient evidence to support the decision of the union to expel the employee from the
union. These requisites constitute just cause for terminating an employee based on the
union security provision of the CBA.

As to the first requisite, there is no question that the CBA between PRI and respondents included
a union security clause. Secondly, it is likewise undisputed that NAMAPRI-SPFL, in two (2)
occasions demanded from PRI, in their letters dated May 16 and 23, 2000, to terminate the
employment of respondents due to their acts of disloyalty to the Union. However, as to the third
requisite, we find that there is no sufficient evidence to support the decision of PRI to terminate
the employment of the respondents.

The mere signing of the authorization in support of the Petition for Certification Election of FFW
on March 19, 20 and 21, or before the freedom period, is not sufficient ground to terminate the
employment of respondents inasmuch as the petition itself was actually filed during the freedom
period. Nothing in the records would show that respondents failed to maintain their membership
in good standing in the Union. Respondents did not resign or withdraw their membership from
the Union to which they belong. Respondents continued to pay their union dues and never joined
the FFW.
Petition denied.
From Atty. Alba^^
Posted October 23, 2012 by vbdiaz in LABOR LAW

CATALINO ALGIRE and OTHER OFFICERS OF


UNIVERSAL ROBINA TEXTILE MONTHLY SALARIED
EMPLOYEES UNION (URTMSEU) vs. REGALADO DE
MESA, et al., and HON. SECRETARY OF LABOR Leave
a comment
CATALINO ALGIRE and OTHER OFFICERS OF UNIVERSAL ROBINA TEXTILE
MONTHLY SALARIED EMPLOYEES UNION (URTMSEU) vs. REGALADO DE
MESA, et al., and HON. SECRETARY OF LABOR
This petition for certiorari seeks to nullify and set aside the decision dated January 31, 1991 of
the Secretary of Labor which reversed on appeal the Order dated December 20, 1990 issued by
Med-arbiter Rolando S. dela Cruz declaring petitioners as the duly-elected officers of the
Universal Robina Textile Monthly Salaried Employees union (URTMSEU) as well as the order
dated March 5, 1991 denying petitioner Catalino Algires motion for reconsideration.
FACTS:
The case arose out of the election of the rightful officers to represent the union in the Collective
Bargaining Agreement (CBA) with the management of Universal Robina Textile at its plant in
Km. 50, Bo. San Cristobal, Calamba, Laguna.
Universal Robina Textile Monthly Salaried Employees Union, (URTMSEU), through private
respondent Regalado de Mesa, filed on September 4, 1990 a petition for the holding of an
election of union officers with the Arbitration Branch of the Department of Labor and
Employment (DOLE). Acting thereon, DOLEs med-arbiter Rolando S. de la Cruz issued an
Order dated October 19, 1990 directing that such an election be held.
In the pre-election conference, it was agreed that the election by secret ballot be conducted on
November 15, 1990 between petitioners (Catalino Algire, et al.) and private respondents

(Regalado de Mesa, et al.) under the supervision of DOLE through its duly appointed
representation officer.
The official ballot contained the following pertinent instructions:
Nais kong pakatawan sa grupo ni:
LINO ALGIRE REGALADO
and DE MESA
his officers and his
officers
1. Mark Check (/) or cross (x) inside the box specified above who among the two contending
parties you desire to be represented for the purpose of collecting bargaining.
2. This is a secret ballot. Dont write any other markings. 1
The result of the election were as follows:
Lino Algire group 133
Regalado de Mesa 133
Spoiled 6

Total votes cast 272


On November 19, 1990, Catalino Algire filed a Petition and/or Motion (RO 400-9009-AU-002),
which DOLEs Med Arbitration unit treated as a protest, to the effect that one of the ballots
wherein one voter placed two checks inside the box opposite the phrase Lino Algire and
his officers, hereinafter referred to as the questioned ballot, should not have been
declared spoiled, as the same was a valid vote in their favor. The group argued that the two
checks made even clearer the intention of the voter to exercise his political franchise in
favor of Algires group.
During the schedules hearing thereof, both parties agreed to open the envelope containing the
spoiled ballots and it was found out that, indeed, one ballot contained two (2) checks in the box
opposite petitioner Algires name and his officers.
On December 20, 1990, med-arbiter de la Cruz issued an order declaring the questioned
ballot valid, thereby counting the same in Algires favor and accordingly certified
petitioners group as the unions elected officers. 2
Regalado de Mesa, et al. appealed from the decision of the med-arbiter to the Secretary of Labor
in Case No. OS-A-1-37-91 (RO 400-9009-AU-002). On January 31, 1991, the latters office
granted the appeal and reversed the aforesaid Order. In its stead, it entered a new one ordering

the calling of another election of officers of the Universal Robina Textile Monthly Salaried
Employees Union (URTMSEU), with the same choices as in the election of
15 November, 1990, after the usual pre-election conference. 3
Director Maximo B. Lim of the Industrial Relations Division, Regional Office No. IV of the
DOLE set the hearing for another pre-election conference on March 22, 1991, reset to April 2,
1991, and finally reset to April 5, 1991.
Catalino Algires group filed a motion for reconsideration of the Order. It was denied for lack of
merit and the decision sought to be reconsidered was sustained.
ISSUES:
Algire, et al. filed this petition on the following issues:
(1) the Secretary of Labor erred in applying Sections 1 and 8 (6), Rule VI, Book V of the Rules
and Regulations implementing the Labor Code to the herein case, considering that the case is an
intra-union activity, which act constitutes a grave abuse in the exercise of authority amounting to
lack of jurisdiction.
(2) the assailed decision and order are not supported by law and evidence.
with an ex-parte motion for issuance of a temporary restraining order, alleging that the assailed
decision of the office of the Secretary of Labor as public respondent is by nature immediately
executory and the holding of an election at any time after April 5, 1991, would render the
petition moot and academic unless restrained by this Court.
On April 5, 1991, we issued a temporary restraining order enjoining the holding of another
election of union officers pursuant to the January 31, 1991 decision. 4
HELD:
There is no merit in the petition.
The contention of the petitioner is that a representation officer (referring to a person duly
authorized to conduct and supervise certification elections in accordance with Rule VI of the
Implementing Rules and Regulations of the Labor Code) can validly rule only on on-the-spot
questions arising from the conduct of the elections, but the determination of the validity of the
questioned ballot is not within his competence. Therefore, any ruling made by the representation
officer concerning the validity of the ballot is deemed an absolute nullity because such is the
allegation it was done without or in excess of his functions amounting to lack of jurisdiction.
To resolve the issue of union representation at the Universal Robina Textile plant, what was
agreed to be held at the companys premises and which became the root of this controversy,
was a election, not a certification election. consent

It is unmistakable that the election held on November 15, 1990 was a consent election and not a
certification election. It was an agreed one, the purpose being merely to determine the issue of
majority representation of all the workers in the appropriate collective bargaining unit. It is a
separate and distinct process and has nothing to do with the import and effort of a certification
election. 5
The ruling of DOLEs representative in that election that the questioned ballot is spoiled is not
based on any legal provision or rule justifying or requiring such action by such officer but simply
in pursuance of the intent of the parties, expressed in the written instructions contained in the
ballot, which is to prohibit unauthorized markings thereon other than a check or a cross,
obviously intended to identify the votes in order to preserve the sanctity of the ballot, which is in
fact the objective of the contending parties.
If indeed petitioners group had any opposition to the representation officers ruling that
the questioned ballot was spoiled, it should have done so seasonably during the canvass of
votes. Its failure or inaction to assail such ballots validity shall be deemed a waiver of any
defect or irregularity arising from said election. Moreover, petitioners even question at this
stage the clear instruction to mark a check or cross opposite the same of the candidates
group, arguing that such instruction was not clear, as two checks may be interpreted that
a voter may vote for Lino Algire but not with (sic) his officers or
vice-versa, 6notwithstanding the fact that a pre-election conference had already been held
where no such question was raised.
In any event, the choice by the majority of employees of the union officers that should best
represent them in the forthcoming collective bargaining negotiations should be achieved through
the democratic process of an election, the proper forum where the true will of the majority may
not be circumvented but clearly defined. The workers must be allowed to freely express their
choice once and for all in a determination where anything is open to their sound judgment and
the possibility of fraud and misrepresentation is minimized, if not eliminated, without any
unnecessary delay and/or maneuvering.
WHEREFORE, the petition is DENIED and the challenged decision is hereby AFFIRMED.
From Atty. Manalundong^^
Posted October 23, 2012 by vbdiaz in LABOR LAW

ORIENTAL v. LAGUESMA Leave a comment


ORIENTAL v. LAGUESMA
G.R. No. 116779

FACTS:
Respondent (in G.R. No. 116751) and petitioner (in sister case G.R. No.
116779), Oriental Tin Can and Metal Sheet Manufacturing Company, Inc. (the company) is
engaged in the manufacture of tin can containers and metal sheets. On March 3, 1994, it entered
into a collective bargaining agreement (CBA) with petitioner Oriental Tin Can Labor Union
(OTCLU) as the existing CBA was due to expire on April 15, 1994. Four days later, 248 of the
companys rank-and-file employees authorized the Federation of Free Workers (FFW) to file a
petition for certification election. On March 10, 1994, however, this petition was repudiated via a
written waiver by 115 of the signatories who, along with other employees totaling 897, ratified
the CBA on the same date.

On March 18, 1994, armed with Charter Certificate No. IV-MEE-089, respondent Oriental Tin
Can Workers Union Federation of Free Workers (OTCWU-FFW) filed a petition for
certification election with the National Capital Region office of the Department of Labor and
Employment (DOLE), pursuant to Article 256 of the Labor Code. Purporting to represent the
regular rank-and-file employees of the company, the petition was accompanied by the authentic
signatures of 25% of the employees/workers in the bargaining unit.

The OTCLU filed a manifestation and motion on April 15, 1994, praying for the dismissal of the
petition for certification election on the ground that it was not endorsed by at least 25% of the
employees of the bargaining unit. Some of the employees who initially signed the petition had
allegedly withdrawn in writing such support prior to the filing of the same.

The OTCWU-FFW filed a reply to said manifestation and motion, claiming that the retraction of
support for the petition was not verified under oath and, therefore, had no legal and binding
effect. It further asserted that the petition had the required support of more than 25% of all the
employees in the bargaining unit.

ISSUE: Whether or not a certification election is necessary

RULING:
Since the question of right of representation as between competing labor
organizations in a bargaining unit is imbued with public interest, the law governs the choice of a
collective bargaining representative which shall be the duly certified agent of the employees
concerned. An official certification becomes necessary where the bargaining agent fails to
present adequate and reasonable proof of its majority authorization and where the
employer demands it, or when the employer honestly doubts the majority representation of
several contending bargaining groups. In fact, Article 255 of the Labor Code allows the

majority of the employees in an appropriate collective bargaining unit to designate or select the
labor organization which shall be their exclusive representative for the purpose of collective
bargaining.

However, in this case, the law dictates that the filing of a petition for certification election during
the 60-day freedom period gives rise to a representation case that must be resolved even though a
new CBA has been entered into within that period. This is clearly provided for in the
aforequoted Section 4, Rule V, Book V of the Omnibus Rules Implementing the Labor Code.
The reason behind this rule is obvious. A petition for certification election is not necessary
where the employees are one in their choice of a representative in the bargaining process.

From Atty. Soriano^^


Posted October 23, 2012 by vbdiaz in LABOR LAW

STANDARD CHARTERED BANK EMPLOYEES UNION


(NUBE) vs. CONFESOR, in her capacity as SECRETARY
OF LABOR AND EMPLOYMENT; and the STANDARD
CHARTERED BANK Leave a comment
STANDARD CHARTERED BANK EMPLOYEES UNION (NUBE) vs. The Honorable
MA. NIEVES R. CONFESOR, in her capacity as SECRETARY OF LABOR AND
EMPLOYMENT; and the STANDARD CHARTERED BANK
G.R. No. 114974

June 16, 2004

FACTS: Before the commencement of the negotiation for the new CBA between the bank and
the Union, the Union, through Divinagracia, suggested to the Banks Human Resource Manager
and head of the negotiating panel, Cielito Diokno, that the bank lawyers should be excluded from
the negotiating team. The Bank acceded. Meanwhile, Diokno(head of the negotiating team for
the bank) suggested to Divinagracia that Jose P. Umali, Jr., the President of the National
Union of Bank Employees (NUBE), the federation to which the Union was affiliated, be
excluded from the Unions negotiating panel. However, Umali was retained as a member
thereof.
There was deadlock in the negotiations. Both parties alleged ULP. Bank alleged that the Union
violated its no strike- no lockout clause by filing a notice of strike before the NCMB.
Considering that the filing of notice of strike was an illegal act, the Union officers should be
dismissed. Union alleged unfair labor practice when the bank allegedly interfered with the
Unions choice of negotiator. It argued that, Dioknos suggestion that the negotiation be

limited as a family affair was tantamount to suggesting that Federation President Jose
Umali, Jr. be excluded from the Unions negotiating panel. It further argued that, damage or
injury to the public interest need not be present in order for unfair labor practice to prosper. The
Union also contended that the Bank merely went through the motions of collective
bargaining without the intent to reach an agreement
ISSUE:
1. WON there was interference
2. WON the bank committed surface bargaining
HELD:
1. NONE
Article 248(a) of the Labor Code, considers it an unfair labor practice when an employer
interferes, restrains or coerces employees in the exercise of their right to self-organization or the
right to form association. The right to self-organization necessarily includes the right to
collective bargaining. Parenthetically, if an employer interferes in the selection of its negotiators
or coerces the Union to exclude from its panel of negotiators a representative of the Union, and if
it can be inferred that the employer adopted the said act to yield adverse effects on the free
exercise to right to self-organization or on the right to collective bargaining of the employees,
ULP under Article 248(a) in connection with Article 243 of the Labor Code is committed.
In order to show that the employer committed ULP under the Labor Code, substantial evidence
is required to support the claim. Substantial evidence has been defined as such relevant
evidence as a reasonable mind might accept as adequate to support a conclusion. In the case
at bar, the Union bases its claim of interference on the alleged suggestions of Diokno to
exclude Umali from the Unions negotiating panel.
The circumstances that occurred during the negotiation do not show that the suggestion made by
Diokno to Divinagracia is an anti-union conduct from which it can be inferred that the Bank
consciously adopted such act to yield adverse effects on the free exercise of the right to selforganization and collective bargaining of the employees, especially considering that such was
undertaken previous to the commencement of the negotiation and simultaneously with
Divinagracias suggestion that the bank lawyers be excluded from its negotiating panel.
The records show that after the initiation of the collective bargaining process, with the inclusion
of Umali in the Unions negotiating panel, the negotiations pushed through. The complaint was
made only on August 16, 1993 after a deadlock was declared by the Union on June 15, 1993.
It is clear that such ULP charge was merely an afterthought. The accusation occurred after
the arguments and differences over the economic provisions became heated and the parties had
become frustrated. It happened after the parties started to involve personalities. As the public
respondent noted, passions may rise, and as a result, suggestions given under less adversarial

situations may be colored with unintended meanings. Such is what appears to have happened in
this case.
1. NO. Surface bargaining is defined as going through the motions of negotiating without
any legal intent to reach an agreement.
The Union alleges that the Bank violated its duty to bargain; hence, committed ULP under
Article 248(g) when it engaged in surface bargaining. It alleged that the Bank just went through
the motions of bargaining without any intent of reaching an agreement, as evident in the Banks
counter-proposals. It explained that of the 34 economic provisions it made, the Bank only made 6
economic counterproposals. Further, as borne by the minutes of the meetings, the Bank, after
indicating the economic provisions it had rejected, accepted, retained or were open for
discussion, refused to make a list of items it agreed to include in the economic package.
The minutes of meetings from March 12, 1993 to June 15, 1993 do not show that the Bank had
any intention of violating its duty to bargain with the Union. Records show that after the Union
sent its proposal to the Bank on February 17, 1993, the latter replied with a list of its counterproposals on February 24, 1993. Thereafter, meetings were set for the settlement of their
differences. The minutes of the meetings show that both the Bank and the Union exchanged
economic and non-economic proposals and counter-proposals.
The Union has not been able to show that the Bank had done acts, both at and away from
the bargaining table, which tend to show that it did not want to reach an agreement with
the Union or to settle the differences between it and the Union. Admittedly, the parties were
not able to agree and reached a deadlock. However, it is herein emphasized that the duty to
bargain does not compel either party to agree to a proposal or require the making of a
concession.
Hence, the parties failure to agree did not amount to ULP under Article 248(g) for violation of
the duty to bargain.
NOTE: (on the allegation of the banks refusal to give certain information) The Union, did not,
as the Labor Code requires, send a written request for the issuance of a copy of the data about the
Banks rank and file employees. Moreover, as alleged by the Union, the fact that the Bank made
use of the aforesaid guestimates, amounts to a validation of the data it had used in its
presentation.
From Atty. Renes^^
Posted October 23, 2012 by vbdiaz in LABOR LAW

COMPLEX ELECTRONICS CORP vs. NLRC, COMPLEX


ELECTRONICS EMPLOYEES ASSOCIATION (CEEA),

represented by Union President, TALAVERA Leave a


comment
COMPLEX ELECTRONICS CORP vs. NLRC, COMPLEX ELECTRONICS
EMPLOYEES ASSOCIATION (CEEA), represented by Union President, TALAVERA
G.R. No. 122136 July 19, 1999
FACTS: Due to losses on production of the petitioner, it was constrained to cease operations. In
the evening of April 6, 1992, the machinery, equipment and materials being used for production
at Complex were pulled-out from the company premises and transferred to the premises of Ionics
Circuit, Inc. (Ionics) at Cabuyao, Laguna. The following day, a total closure of company
operation was effected at Complex.
A complaint was, thereafter, filed with the Labor Arbitration Branch of the NLRC for unfair
labor practice, illegal closure/illegal lockout, money claims for vacation leave, sick leave, unpaid
wages, 13th month pay, damages and attorneys fees. The Union alleged that the pull-out of the
machinery, equipment and materials from the company premises, which resulted to the sudden
closure of the company was in violation of Section 3 and 8, Rule XIII, Book V of the Labor
Code of the Philippines and the existing CBA. Ionics was impleaded as a party defendant
because the officers and management personnel of Complex were also holding office at Ionics
with Lawrence Qua as the President of both companies.
The Union anchors its position on the fact that Lawrence Qua is both the president of Complex
and Ionics and that both companies have the same set of Board of Directors. It claims that
business has not ceased at Complex but was merely transferred to Ionics, a runaway shop. To
prove that Ionics was just a runaway shop, petitioner asserts that out of the 80,000 shares
comprising the increased capital stock of Ionics, it was Complex that owns majority of said
shares with P1,200,000.00 as its capital subscription and P448,000.00 as its paid up investment,
compared to P800,000.00 subscription andP324,560.00 paid-up owing to the other stockholders,
combined. Thus, according to the Union, there is a clear ground to pierce the veil of corporate
fiction.
ISSSUE: WON Ionics is merely a runaway shop
HELD: NO
A runaway shop is defined as an industrial plant moved by its owners from one location to
another to escape union labor regulations or state laws, but the term is also used to describe a
plant removed to a new location in order to discriminate against employees at the old plant
because of their union activities. It is one wherein the employer moves its business to another
location or it temporarily closes its business for anti-union purposes. A runaway shop in this
sense, is a relocation motivated by anti-union animus rather than for business reasons.

In this case, however, Ionics was not set up merely for the purpose of transferring the
business of Complex. At the time the labor dispute arose at Complex, Ionics was already
existing as an independent company. As earlier mentioned, it has been in existence since
July 5, 1984 (8 years prior to the dispute). It cannot, therefore, be said that the temporary
closure in Complex and its subsequent transfer of business to Ionics was for anti-union
purposes. The Union failed to show that the primary reason for the closure of the
establishment was due to the union activities of the employees.
The mere fact that one or more corporations are owned or controlled by the same or single
stockholder is not a sufficient ground for disregarding separate corporate personalities. Mere
ownership by a single stockholder or by another corporation of all or nearly all of the capital
stock of a corporation is not of itself sufficient ground for disregarding the separate corporate
personality.
At first glance after reading the decision a quo, it would seem that the closure of respondents
operation is not justified. However, a deeper examination of the records along with the evidence,
would show that the closure, although it was done abruptly as there was no compliance with the
30-day prior notice requirement, said closure was not intended to circumvent the provisions of
the Labor Code on termination of employment. The closure of operation by Complex on April 7,
1992 was not without valid reasons. Customers of respondent alarmed by the pending labor
dispute and the imminent strike to be foisted by the union, as shown by their strike vote, directed
respondent Complex to pull-out its equipment, machinery and materials to other safe bonded
warehouse. Respondent being mere consignees of the equipment, machinery and materials were
without any recourse but to oblige the customers directive. The pull-out was effected on April
6, 1992. We can see here that Complexs action, standing alone, will not result in illegal closure
that would cause the illegal dismissal of the complainant workers. Hence, the Labor Arbiters
conclusion that since there were only 2 of respondents customers who have expressed pull-out
of business from respondent Complex while most of the customers have not and, therefore, it is
not justified to close operation cannot be upheld. The determination to cease operation is a
prerogative of management that is usually not interfered with by the State as no employer can be
required to continue operating at a loss simply to maintain the workers in employment. That
would be taking of property without due process of law which the employer has the right to
resist.
From Atty. Renes^^
Posted October 23, 2012 by vbdiaz in LABOR LAW

CATHAY PACIFIC STEEL CORPORATION et al vs. CA


et al Leave a comment
CATHAY PACIFIC STEEL CORPORATION, BENJAMIN CHUA JR., VIRGILIO
AGERO, and LEONARDO VISORRO, JR. vs. CA, CAPASCO UNION OF
SUPERVISORY EMPLOYEES (CUSE) and ENRIQUE TAMONDONG III

G.R. No. 164561 August 30, 2006


FACTS: Sometime in June 1996, the supervisory personnel of CAPASCO launched a move to
organize a union among their ranks, later known as private respondent CUSE. Private respondent
Tamondong actively involved himself in the formation of the union and was even elected as one
of its officers after its creation. Consequently, petitioner CAPASCO sent a memo to private
respondent Tamondong requiring him to explain and to discontinue from his union activities,
with a warning that a continuance thereof shall adversely affect his employment in the company.
Private respondent Tamondong ignored said warning and made a reply letter invoking his right
as a supervisory employee to join and organize a labor union. In view of that, on 6 February
1997, petitioner CAPASCO through a memo terminated the employment of private respondent
Tamondong on the ground of loss of trust and confidence, citing his union activities as acts
constituting serious disloyalty to the company.
Private respondent Tamondong challenged his dismissal for being illegal and as an act involving
unfair labor practice by filing a Complaint for Illegal Dismissal and Unfair Labor Practice before
the NLRC, Regional Arbitration Branch IV. According to him, there was no just cause for his
dismissal and it was anchored solely on his involvement and active participation in the
organization of the union of supervisory personnel in CAPASCO. Though private respondent
Tamondong admitted his active role in the formation of a union composed of supervisory
personnel in the company, he claimed that such was not a valid ground to terminate his
employment because it was a legitimate exercise of his constitutionally guaranteed right to selforganization.
In contrast, petitioner CAPASCO contended that by virtue of private respondent Tamondongs
position as Personnel Superintendent and the functions actually performed by him in the
company, he was considered as a managerial employee, thus, under the law he was prohibited
from joining a union as well as from being elected as one of its officers. Accordingly, petitioners
maintained their argument that the dismissal of private respondent Tamondong was perfectly
valid based on loss of trust and confidence because of the latters active participation in the
affairs of the union.
ISSUE: WON the dismissal of Tamondong was valid
HELD: INVALID
[Private respondent] Tamondong may have possessed enormous powers and was performing
important functions that goes with the position of Personnel Superintendent, nevertheless, there
was no clear showing that he is at liberty, by using his own discretion and disposition, to lay
down and execute major business and operational policies for and in behalf of CAPASCO.
[Petitioner] CAPASCO miserably failed to establish that [private respondent] Tamondong was
authorized to act in the interest of the company using his independent judgment. x x x. Withal,
[private respondent] Tamondong may have been exercising certain important powers, such as
control and supervision over erring rank-and-file employees, however, x x x he does not possess
the power to hire, transfer, terminate, or discipline erring employees of the company. At the most,
the record merely showed that [private respondent] Tamondong informed and warned rank-and-

file employees with respect to their violations of CAPASCOs rules and regulations. x x x. [Also,
the functions performed by private respondent such as] issuance of warning to employees with
irregular attendance and unauthorized leave of absences and requiring employees to explain
regarding charges of abandonment of work, are normally performed by a mere supervisor, and
not by a manager.
Being a supervisory employee of CAPASCO, he cannot be prohibited from joining or
participating in the union activities of private respondent CUSE, and in making such a
conclusion, the Court of Appeals did not act whimsically, capriciously or in a despotic manner,
rather, it was guided by the evidence submitted before it. Thus, given the foregoing findings of
the Court of Appeals that private respondent is a supervisory employee, it is indeed an
unfair labor practice on the part of petitioner CAPASCO to dismiss him on account of his
union activities, thereby curtailing his constitutionally guaranteed right to selforganization.
From Atty. Renes^^
Posted October 23, 2012 by vbdiaz in LABOR LAW

UNION OF FILIPRO EMPLOYEES vs. NLRC and


NESTLE PHILIPPINES, INC. Leave a comment
UNION OF FILIPRO EMPLOYEES vs. NLRC and NESTLE PHILIPPINES, INC.
G.R. No. 91025 : December 19, 1990.
FACTS: On June 22, 1988, the petitioner Union of the Filipro Employees, the sole and exclusive
bargaining agent of all rank-and-file employees of Nestle Philippines, (private respondent) filed a
Notice of Strike at the DOLE raising the issues of CBA deadlock and unfair labor practice.
Private respondent assailed the legal personality of the proponents of the said notice of strike to
represent the Nestle employees, before the NCMB. This notwithstanding, the NCMB proceeded
to invite the parties to attend the conciliation meetings and to which private respondent failed to
attend contending that it will deal only with a negotiating panel duly constituted and mandated in
accordance with the UFE Constitution and By-laws. Thereafter, Company terminated from
employment all UFE Union officers, and all the members of the negotiating panel for instigating
and knowingly participating in a strike staged at the Makati, Alabang, Cabuyao and Cagayan de
Oro on September 11, 1987 without any notice of strike filed and a strike vote obtained for the
purpose. The union filed a complaint for illegal dismissal. LA upheld the validity of the
dismissal; NLRC en banc affirmed. Subsequently, company concluded separate CBAs with the
general membership of the union at Cebu/Davao and Cagayan de Oro units; Assailing the
validity of these agreements, the union filed a case of ULP against the company with the NLRCNCR Arbitration Branch Efforts to resolve the dispute amicably were taken by the NCMB but
yielded negative result. Petitioner filed a motion asking the Secretary of Labor to assume
jurisdiction over the dispute of deadlock in collective bargaining between the parties. On October
28, 1988, Labor Secretary Franklin Drilon certified to the NLRC the said dispute between the

UFE and Nestle, Philippines.. which reads as follows: xxx The NLRC is further directed to call
all the parties immediately and resolve the CBA deadlock within twenty (20) days from
submission of the case for resolution. Second Division of the NLRC promulgated a resolution
granting wage increase and other benefits to Nestles employees, ruling on non-economic issues,
as well as absolving the private respondent of the Unfair Labor Practice charge. Petitioner finds
said resolution to be inadequate and accordingly, does not agree therewith. It filed a motion for
reconsideration, denied. Hence, this petition. ISSUE: (relative to the topic) WON WHETHER
OR NOT THE SECOND DIVISION OF THE NLRC ACTED WITHOUT JURISDICTION IN
RENDERING THE ASSAILED RESOLUTION, THE SAME BEING RENDERED ONLY BY
A DIVISION OF THE PUBLIC RESPONDENT AND NOT BY EN BANC; HELD: This case
was certified on October 28, 1988 when existing rules prescribed that, it is incumbent upon the
Commission en banc to decide or resolve a certified dispute. However, R.A. 6715 took effect
during the pendency of this case. Aside from vesting upon each division the power to adjudicate
cases filed before the Commission, said Act further provides that the divisions of the
Commission shall have exclusive appellate jurisdiction over cases within their respective
territorial jurisdiction. Section 5 of RA 6715 provides as follows: xxxx The Commission may sit
en banc or in five (5) divisions, each composed of three (3) members. The Commission shall sit
en banc only for purposes of promulgating rules and regulations governing the hearing and
disposition of cases before any of its divisions and regional branches and formulating policies
affecting its administration and operations. The Commission shall exercise its adjudicatory and
all other powers, functions and duties through its divisions. xxxx In view of the enactment of
Republic Act 6715, the aforementioned rules requiring the Commission en banc to decide or
resolve a certified dispute have accordingly been repealed. Confirmed in Administrative Order
No. 36 (Series of 1989) promulgated by the Secretary under his delegated rule-making power.
Moreover, it is to be emphasized and it is a matter of judicial notice that since the effectivity of
R.A. 6715, many cases have already been decided by the 5 divisions of the NLRC. We find no
legal justification in entertaining petitioners claim considering that the clear intent of the
amendatory provision is to expedite the disposition of labor cases filed before the Commission.
To rule otherwise would not be congruous to the proper administration of justice.
ACCORDINGLY, PREMISES CONSIDERED, the petition is DISMISSED. The Resolutions of
the NLRC, dated June 5, 1989 and August 8, 1989 are AFFIRMED, except insofar as the ruling
absolving the private respondent of unfair labor practice which is declared SET ASIDE.:- From
Atty. Bayani^^
Posted October 23, 2012 by vbdiaz in LABOR LAW

PHIMCO vs.BRILLANTES and PHIMCO INDUSTRIES


LABOR ASSOCIATION Leave a comment
PHIMCO INDUSTRIES, INC., vs.HONORABLE ACTING SECRETARY OF LABOR
JOSE BRILLANTES and PHIMCO INDUSTRIES LABOR ASSOCIATION
G.R. No. 120751 March 17, 1999

FACTS: On March 9, 1995, the private respondent, Phimco Industries Labor Association
(PILA), duly certified collective bargaining representative of the daily paid workers of the
petitioner PHIMCO filed a notice of strike with the NCMB against PHIMCO, a corporation
engaged in the production of matches, after a deadlock in the collective bargaining and
negotiation. Parties failed to resolve their differences PILA (during the conciliation conferences),
composed of 352 members, staged a strike.
PHIMCO sent notice of termination to some 47 workers including several union officers.
Secretary Brillantes assumed jurisdiction over the labor dispute; issued a return-to-work order.
Hence, petitioner files this petition.
ISSUE: whether or not the public respondent acted with grave abuse of discretion amounting to
lack or excess of jurisdiction in assuming jurisdiction over subject labor dispute.
HELD: YES, the petition is impressed with merit.
Art. 263, paragraph (g) of the Labor Code, provides:
(g) When, in his opinion, there exist a labor dispute causing or likely to cause a strike or lockout
in an industry indispensable to the national interest, the Secretary of Labor and Employment may
assume jurisdiction over the dispute and decide it or certify the same to the Commission for
compulsory arbitration . . .
The Labor Code vests in the Secretary of Labor the discretion to determine what industries are
indispensable to the national interest. Accordingly, upon the determination by the Secretary of
Labor that such industry is indispensable to the national interest, he will assume jurisdiction over
the labor dispute in the said industry. 8 This power, however, is not without any limitation.
It stressed in the case of Free telephone Workers Union vs. Honorable Minister of Labor and
Employment, et al., 10 the limitation set by the legislature on the power of the Secretary of Labor
to assume jurisdiction over a labor dispute, thus:
xxx cannot be any clearer, the coverage being limited to strikes or lockouts adversely affecting
the national interest. 11
In this case at bar, however, the very admission by the public respondent draws the labor dispute
in question out of the ambit of the Secretarys prerogative, to wit.
While the case at bar appears on its face not to fall within the strict categorization of cases
imbued with national interest, this office believes that the obtaining circumstances warrant the
exercise of the powers under Article 263 (g) of the Labor Code, as amended. 12
The private respondent did not even make any effort to touch on the indispensability of the
match factory to the national interest. It must have been aware that a match factory,

though of value, can scarcely be considered as an industry indispensable to the national


interest as it cannot be in the same category as generation and distribution of energy, or those
undertaken by banks, hospitals, and export-oriented industries. 13 Yet, the public respondent
assumed jurisdiction thereover.
To uphold the action of the public respondent under the premises would be stretching too far the
power of the Secretary of Labor as every case of a strike or lockout where there are
inconveniences in the community, or work disruptions in an industry though not indispensable to
the national interest, would then come within the Secretarys power. It would be practically
allowing the Secretary of Labor to intervene in any Labor dispute at his pleasure.
This is precisely why the law sets and defines the standard: even in the exercise of his power of
compulsory arbitration under Article 263 (g) of the Labor Code, the Secretary must follow the
law.
PETITION GRANTED.
From Atty. Bayani^^
Posted October 23, 2012 by vbdiaz in LABOR LAW

Ymbong vs. ABS-CBN Leave a comment


Ymbong vs. ABS-CBN
G.R. No. 184885
Facts:
Petitioner Ernesto G. Ymbong started working for ABS-CBN Broadcasting Corporation (ABSCBN) in 1993 at its regional station in Cebu as a television talent, co-anchoring Hoy Gising and
TV Patrol Cebu. His stint in ABS-CBN later extended to radio when ABS-CBN Cebu launched
its AM station DYAB in 1995 where he worked as drama and voice talent, spinner, scriptwriter
and public affairs program anchor.
Like Ymbong, Leandro Patalinghug also worked for ABS-CBN Cebu. Starting 1995, he worked
as talent, director and scriptwriter for various radio programs aired over DYAB.
On January 1, 1996, the ABS-CBN Head Office in Manila issued Policy No. HR-ER-016 or the
Policy on Employees Seeking Public Office. The pertinent portions read:
1.
Any employee who intends to run for any public office position, must file his/her
letter of resignation, at least thirty (30) days prior to the official filing of the certificate of
candidacy either for national or local election.
xxxx

3.
Further, any employee who intends to join a political group/party or even with no
political affiliation but who intends to openly and aggressively campaign for a candidate or
group of candidates (e.g. publicly speaking/endorsing candidate, recruiting campaign workers,
etc.) must file a request for leave of absence subject to managements approval. For this
particular reason, the employee should file the leave request at least thirty (30) days prior to the
start of the planned leave period.
x x x x [Emphasis and underscoring supplied.]
Because of the impending May 1998 elections and based on his immediate recollection of the
policy at that time, Dante Luzon, Assistant Station Manager of DYAB issued the following
memorandum:
TO

ALL CONCERNED

FROM

DANTE LUZON

DATE

MARCH 25, 1998

SUBJECT

AS STATED

Please be informed that per company policy, any employee/talent who wants to run for any
position in the coming election will have to file a leave of absence the moment he/she files
his/her certificate of candidacy.
The services rendered by the concerned employee/talent to this company will then be
temporarily suspended for the entire campaign/election period.
For strict compliance.
After the issuance of the March 25, 1998 Memorandum, Ymbong got in touch
with Luzon. Luzon claims that Ymbong approached him and told him that he would leave radio
for a couple of months because he will campaign for the administration ticket. It was only after
the elections that they found out that Ymbong actually ran for public office himself at the
eleventh hour. Ymbong, on the other hand, claims that in accordance with the March 25, 1998
Memorandum, he informed Luzon through a letter that he would take a few months leave of
absence from March 8, 1998 to May 18, 1998 since he was running for councilor of Lapu-Lapu
City.
As regards Patalinghug, Patalinghug approached Luzon and advised him that he will run as
councilor for Naga, Cebu. According to Luzon, he clarified to Patalinghug that he will be
considered resigned and not just on leave once he files a certificate of candidacy.
Later, Ymbong and Patalinghug both tried to come back to ABS-CBN Cebu. According
to Luzon, he informed them that they cannot work there anymore because of company
policy. This was stressed even in subsequent meetings and they were told that the company was

not allowing any exceptions. ABS-CBN, however, agreed out of pure liberality to give them a
chance to wind up their participation in the radio drama, Nagbabagang Langit, since it was
rating well and to avoid an abrupt ending. The agreed winding-up, however, dragged on for so
long prompting Luzon to issue to Ymbong the memorandum dated September 14, 1998
automatically terminating them.
Issue:
1. whether Policy No. HR-ER-016 is valid
2. whether the March 25, 1998 Memorandum issued by Luzonsuperseded Policy No. HR-ER016
3. whether Ymbong, by seeking an elective post, is deemed to have resigned and not dismissed
by ABS-CBN.
Held:
1.
ABS-CBN had a valid justification for Policy No. HR-ER-016. Its rationale is
embodied in the policy itself, to wit:
Rationale:
ABS-CBN BROADCASTING CORPORATION strongly believes that it is to the best interest
of the company to continuously remain apolitical. While it encourages and supports its
employees to have greater political awareness and for them to exercise their right to
suffrage, the company, however, prefers to remain politically independent and unattached
to any political individual or entity.
Therefore, employees who [intend] to run for public office or accept political appointment
should resign from their positions, in order to protect the company from any public
misconceptions. To preserve its objectivity, neutrality and credibility, the company reiterates
the following policy guidelines for strict implementation.
We have consistently held that so long as a companys management prerogatives are exercised in
good faith for the advancement of the employers interest and not for the purpose of defeating or
circumventing the rights of the employees under special laws or under valid agreements, this
Court will uphold them. In the instant case, ABS-CBN validly justified the implementation of
Policy No. HR-ER-016. It is well within its rights to ensure that it maintains its objectivity and
credibility and freeing itself from any appearance of impartiality so that the confidence of the
viewing and listening public in it will not be in any way eroded. Even as the law is solicitous of
the welfare of the employees, it must also protect the right of an employer to exercise what are
clearly management prerogatives. The free will of management to conduct its own business
affairs to achieve its purpose cannot be denied.

It is worth noting that such exercise of management prerogative has earned a stamp of approval
from no less than our Congress itself when on February 12, 2001, it enacted Republic Act No.
9006, otherwise known as the Fair Election Act. Section 6.6 thereof reads:
6.6. Any mass media columnist, commentator, announcer, reporter, on-air correspondent or
personality who is a candidate for any elective public office or is a campaign volunteer for
or employed or retained in any capacity by any candidate or political party shall be deemed
resigned, if so required by their employer, or shall take a leave of absence from his/her work
as such during the campaign period: Provided, That any media practitioner who is an official of a
political party or a member of the campaign staff of a candidate or political party shall not use
his/her time or space to favor any candidate or political party. [Emphasis and underscoring
supplied.]
2.
The CA correctly ruled that though Luzon, as Assistant Station Manager for Radio of
ABS-CBN, has policy-making powers in relation to his principal task of administering the
networks radio station in the Cebu region, the exercise of such power should be in accord with
the general rules and regulations imposed by the ABS-CBN Head Office to its employees.
Clearly, the March 25, 1998 Memorandum issued by Luzon which only requires employees to go
on leave if they intend to run for any elective position is in absolute contradiction with Policy
No. HR-ER-016 issued by the ABS-CBN Head Office in Manila which requires the resignation,
not only the filing of a leave of absence, of any employee who intends to run for public
office. Having been issued beyond the scope of his authority, the March 25, 1998 Memorandum
is therefore void and did not supersede Policy No. HR-ER-016.
Also worth noting is that Luzon in his Sworn Statement admitted the inaccuracy of his
recollection of the company policy when he issued the March 25, 1998 Memorandum and stated
therein that upon double-checking of the exact text of the policy statement and subsequent
confirmation with the ABS-CBN Head Office in Manila, he learned that the policy required
resignation for those who will actually run in elections because the company wanted to maintain
its independence. Since the officer who himself issued the subject memorandum acknowledged
that it is not in harmony with the Policy issued by the upper management, there is no reason for it
to be a source of right for Ymbong.
3.
As Policy No. HR-ER-016 is the subsisting company policy and not Luzons March
25, 1998 Memorandum, Ymbong is deemed resigned when he ran for councilor.
We find no merit in Ymbongs argument that [his] automatic termination x x x was a blatant
[disregard] of [his] right to due process as he was never asked to explain why he did not tender
his resignation before he ran for public office as mandated by [the subject company
policy]. Ymbongs overt act of running for councilor of Lapu-Lapu City is tantamount to
resignation on his part. He was separated from ABS-CBN not because he was dismissed but
because he resigned. Since there was no termination to speak of, the requirement of due process
in dismissal cases cannot be applied to Ymbong. Thus, ABS-CBN is not duty-bound to ask him
to explain why he did not tender his resignation before he ran for public office as mandated by
the subject company policy.

Petition denied
From Atty. Alba^^
Posted October 23, 2012 by vbdiaz in LABOR LAW

EDI-Staffbuilders vs. NLRC Leave a comment


EDI-Staffbuilders vs. NLRC
G.R. No. 145587

October 26, 2007

Facts:
Petitioner EDI is a corporation engaged in recruitment and placement of OFWs. ESI is another
recruitment agency which collaborated with EDI to process the documentation and deployment
of private respondent to Saudi Arabia.
Private respondent Gran was an OFW recruited by EDI, and deployed by ESI to work for OAB,
in Riyadh, Kingdom of Saudi Arabia.
It appears that OAB asked EDI through its October 3, 1993 letter for curricula vitae of qualified
applicants for the position of Computer Specialist.7 In a facsimile transmission dated
November 29, 1993, OAB informed EDI that, from the applicants curricula vitae submitted to it
for evaluation, it selected Gran for the position of Computer Specialist. The faxed letter also
stated that if Gran agrees to the terms and conditions of employment contained in it, one of
which was a monthly salary of SR (Saudi Riyal) 2,250.00 (USD 600.00), EDI may arrange for
Grans immediate dispatch.
After accepting OABs offer of employment, Gran signed an employment contract that granted
him a monthly salary of USD 850.00 for a period of two years. Gran was then deployed to
Riyadh, Kingdom of Saudi Arabia on February 7, 1994.
Upon arrival in Riyadh, Gran questioned the discrepancy in his monthly salaryhis employment
contract stated USD 850.00; while his POEA Information Sheet indicated USD 600.00 only.
However, through the assistance of the EDI office in Riyadh, OAB agreed to pay Gran USD
850.00 a month.
After Gran had been working for about five months for OAB, his employment was terminated
through OABs July 9, 1994 letter, on the following grounds:
1. Non-compliance to contract requirements by the recruitment agency primarily on your salary
and contract duration.
2. Non-compliance to pre-qualification requirements by the recruitment agency[,] vide OAB
letter ref. F-5751-93, dated October 3, 1993.

3. Insubordination or disobedience to Top Management Order and/or instructions (non-submittal


of daily activity reports despite several instructions).
On July 11, 1994, Gran received from OAB the total amount of SR 2,948.00 representing his
final pay, and on the same day, he executed a Declaration releasing OAB from any financial
obligation or otherwise, towards him.
After his arrival in the Philippines, Gran instituted a complaint, on July 21, 1994, against
ESI/EDI, OAB, Country Bankers Insurance Corporation, and Western Guaranty Corporation
with the NLRC, National Capital Region, Quezon City, which was docketed as POEA ADJ (L)
94-06-2194 for underpayment of wages/salaries and illegal dismissal.
Issue:
1. WON Grans dismissal is justifiable by reason of incompetence, insubordination, and
disobedience
2. WON GRAN WAS AFFORDED DUE PROCESS PRIOR TO TERMINATION.
3. WON GRAN IS ENTITLED TO BACKWAGES FOR THE UNEXPIRED PORTION OF HIS
CONTRACT.
Held:
1.
In cases involving OFWs, the rights and obligations among and between the OFW, the
local recruiter/agent, and the foreign employer/principal are governed by the employment
contract. A contract freely entered into is considered law between the parties; and hence, should
be respected. In formulating the contract, the parties may establish such stipulations, clauses,
terms and conditions as they may deem convenient, provided they are not contrary to law,
morals, good customs, public order, or public policy.
In the present case, the employment contract signed by Gran specifically states that Saudi Labor
Laws will govern matters not provided for in the contract (e.g. specific causes for termination,
termination procedures, etc.). Being the law intended by the parties (lex loci intentiones) to apply
to the contract, Saudi Labor Laws should govern all matters relating to the termination of the
employment of Gran.
In international law, the party who wants to have a foreign law applied to a dispute or case has
the burden of proving the foreign law. The foreign law is treated as a question of fact to be
properly pleaded and proved as the judge or labor arbiter cannot take judicial notice of a foreign
law. He is presumed to know only domestic or forum law.
Unfortunately for petitioner, it did not prove the pertinent Saudi laws on the matter; thus, the
International Law doctrine ofpresumed-identity approach or processual presumption comes into
play. Where a foreign law is not pleaded or, even if pleaded, is not proved, the presumption is

that foreign law is the same as ours. Thus, we apply Philippine labor laws in determining the
issues presented before us.
In illegal dismissal cases, it has been established by Philippine law and jurisprudence that the
employer should prove that the dismissal of employees or personnel is legal and just.
Section 33 of Article 277 of the Labor Code states that:
ART. 277. MISCELLANEOUS PROVISIONS
(b) Subject to the constitutional right of workers to security of tenure and their right to be
protected against dismissal except for a just and authorized cause and without prejudice to the
requirement of notice under Article 283 of this Code, the employer shall furnish the worker
whose employment is sought to be terminated a written notice containing a statement of the
causes for termination and shall afford the latter ample opportunity to be heard and to defend
himself with the assistance of his representative if he so desires in accordance with company
rules and regulations promulgated pursuant to guidelines set by the Department of Labor and
Employment. Any decision taken by the employer shall be without prejudice to the right of the
workers to contest the validity or legality of his dismissal by filing a complaint with the regional
branch of the National Labor Relations Commission. The burden of proving that the
termination was for a valid or authorized cause shall rest on the employer.
Petitioners imputation of incompetence on private respondent due to his insufficient knowledge
in programming and zero knowledge of the ACAD system based only on the above mentioned
letters, without any other evidence, cannot be given credence.
An allegation of incompetence should have a factual foundation. Incompetence may be shown by
weighing it against a standard, benchmark, or criterion. However, EDI failed to establish any
such bases to show how petitioner found Gran incompetent.
In addition, the elements that must concur for the charge of insubordination or willful
disobedience to prosper were not present.
In Micro Sales Operation Network v. NLRC, we held that:
For willful disobedience to be a valid cause for dismissal, the following twin elements must
concur: (1) the employees assailed conduct must have been willful, that is, characterized by a
wrongful and perverse attitude; and (2) the order violated must have been reasonable, lawful,
made known to the employee and must pertain to the duties which he had been engaged to
discharge.
EDI failed to discharge the burden of proving Grans insubordination or willful disobedience. As
indicated by the second requirement provided for in Micro Sales Operation Network, in order to
justify willful disobedience, we must determine whether the order violated by the employee is
reasonable, lawful, made known to the employee, and pertains to the duties which he had been
engaged to discharge. In the case at bar, petitioner failed to show that the order of the company

which was violatedthe submission of Daily Activity Reportswas part of Grans duties as a
Computer Specialist. Before the Labor Arbiter, EDI should have provided a copy of the company
policy, Grans job description, or any other document that would show that the Daily Activity
Reports were required for submission by the employees, more particularly by a Computer
Specialist.
Even though EDI and/or ESI were merely the local employment or recruitment agencies and not
the foreign employer, they should have adduced additional evidence to convincingly show that
Grans employment was validly and legally terminated. The burden devolves not only upon the
foreign-based employer but also on the employment or recruitment agency for the latter is not
only an agent of the former, but is also solidarily liable with the foreign principal for any claims
or liabilities arising from the dismissal of the worker.
Thus, petitioner failed to prove that Gran was justifiably dismissed due to incompetence,
insubordination, or willful disobedience.
2.
Under the twin notice requirement, the employees must be given 2 notices before their
employment could be terminated: (1) a first notice to apprise the employees of their fault, and (2)
a second notice to communicate to the employees that their employment is being terminated. In
between the first and second notice, the employees should be given a hearing or opportunity to
defend themselves personally or by counsel of their choice.
A careful examination of the records revealed that, indeed, OABs manner of dismissing Gran
fell short of the two notice requirement. While it furnished Gran the written notice informing him
of his dismissal, it failed to furnish Gran the written notice apprising him of the charges against
him, as prescribed by the Labor Code. Consequently, he was denied the opportunity to respond to
said notice. In addition, OAB did not schedule a hearing or conference with Gran to defend
himself and adduce evidence in support of his defenses. Moreover, the July 9, 1994 termination
letter was effective on the same day. This shows that OAB had already condemned Gran to
dismissal, even before Gran was furnished the termination letter. It should also be pointed out
that OAB failed to give Gran the chance to be heard and to defend himself with the assistance of
a representative in accordance with Article 277 of the Labor Code. Clearly, there was no
intention to provide Gran with due process. Summing up, Gran was notified and his employment
arbitrarily terminated on the same day, through the same letter, and for unjustified grounds.
Obviously, Gran was not afforded due process.
Pursuant to the doctrine laid down in Agabon, an employer is liable to pay nominal damages as
indemnity for violating the employees right to statutory due process. Since OAB was in breach
of the due process requirements under the Labor Code and its regulations, OAB, ESI, and EDI,
jointly and solidarily, are liable to Gran in the amount of PhP 30,000.00 as indemnity.
3.
We reiterate the rule that with regard to employees hired for a fixed period of
employment, in cases arising before the effectivity of R.A. No. 8042 (Migrant Workers and
Overseas Filipinos Act) on August 25, 1995, that when the contract is for a fixed term and the
employees are dismissed without just cause, they are entitled to the payment of their salaries
corresponding to the unexpired portion of their contract.59 On the other hand, for cases arising

after the effectivity of R.A. No. 8042, when the termination of employment is without just, valid
or authorized cause as defined by law or contract, the worker shall be entitled to the full
reimbursement of his placement fee with interest of 12% per annum, plus his salaries for the
unexpired portion of his employment contract or for 3 months for every year of the unexpired
term whichever is less.
In the present case, the employment contract provides that the employment contract shall be
valid for a period of 2 years from the date the employee starts to work with the employer. Gran
arrived in Riyadh, Saudi Arabia and started to work on February 7, 1994; hence, his employment
contract is until February 7, 1996. Since he was illegally dismissed on July 9, 1994, before the
effectivity of R.A. No. 8042, he is therefore entitled to backwages corresponding to the
unexpired portion of his contract, which was equivalent to USD 16,150.
From Atty. Alba^^
Posted October 23, 2012 by vbdiaz in LABOR LAW

AUJERO VS. PHILCOMSAT Leave a comment


HYPTE R. AUJERO, PETITIONER, VS. PHILIPPINE COMMUNICATIONS
SATELLITE CORPORATION, RESPONDENT.
G.R. No. 193484, January 18, 2012
Facts:
It was in 1967 that the petitioner started working for respondent Philippine Communications
Satellite Corporation (Philcomsat) as an accountant in the latters Finance Department. On
August 15, 2001 or after 34 years of service, the petitioner applied for early retirement. His
application for retirement was approved, effective September 15, 2001, entitling him to receive
retirement benefits at a rate equivalent to one and a half of his monthly salary for every year of
service. At that time, the petitioner was Philcomsats Senior Vice-President with a monthly salary
of P274,805.00.
On September 12, 2001, the petitioner executed a Deed of Release and Quitclaim in Philcomsats
favor, following his receipt from the latter of a check in the amount of P9,439,327.91.
Almost three (3) years thereafter, the petitioner filed a complaint for unpaid retirement benefits,
claiming that the actual amount of his retirement pay is P14,015,055.00 and the P9,439,327.91
he received from Philcomsat as supposed settlement for all his claims is unconscionable, which
is more than enough reason to declare his quitclaim as null and void. According to the petitioner,
he had no choice but to accept a lesser amount as he was in dire need thereof and was all set to
return to his hometown and he signed the quitclaim despite the considerable deficiency as no
single centavo would be released to him if he did not execute a release and waiver in
Philcomsats favor.

The petitioner claims that his right to receive the full amount of his retirement benefits, which is
equivalent to one and a half of his monthly salary for every year of service, is provided under the
Retirement Plan that Philcomsat created on January 1, 1977 for the benefit of its employees.
Issue: Whether the quitclaim executed by the petitioner in Philcomsats favor is valid, thereby
foreclosing his right to institute any claim against Philcomsat.
Held:
While the law looks with disfavor upon releases and quitclaims by employees who are inveigled
or pressured into signing them by unscrupulous employers seeking to evade their legal
responsibilities, a legitimate waiver representing a voluntary settlement of a laborers claims
should be respected by the courts as the law between the parties.[29]Considering the petitioners
claim of fraud and bad faith against Philcomsat to be unsubstantiated, this Court finds the
quitclaim in dispute to be legitimate waiver.
While the petitioner bewailed as having been coerced or pressured into signing the release and
waiver, his failure to present evidence renders his allegation self-serving and inutile to invalidate
the same. That no portion of his retirement pay will be released to him or his urgent need for
funds does not constitute the pressure or coercion contemplated by law.
From Atty. Alba^^
Posted October 23, 2012 by vbdiaz in LABOR LAW

San Miguel Corporation vs. NLRC Leave a comment


San Miguel Corporation vs. NLRC
G.R. Nos. 146121-22, April 16, 2008
Facts:
Ibias (respondent) was employed by petitioner SMC on 24 December 1978 initially as a CRO
operator in its Metal Closure and Lithography Plant. Respondent continuously worked therein
until he advanced as Zamatic operator. He was also an active and militant member of a labor
organization called Ilaw Buklod Manggagawa (IBM)-SMC Chapter.
According to SMCs Policy on Employee Conduct, absences without permission or AWOPs,
which are absences not covered either by a certification of the plant doctor that the employee was
absent due to sickness or by a duly approved application for leave of absence filed at least 6 days
prior to the intended leave, are subject to disciplinary action characterized by progressively
increasing weight. The same Policy on Employee Conduct also punishes falsification of
company records or documents with discharge or termination for the first offense if the offender
himself or somebody else benefits from falsification or would have benefited if falsification is
not found on time.

It appears that per company records, respondent was AWOP on the following dates in 1997: 2, 4
and 11 January; 26, 28 and 29 April; and 5, 7, 8, 13, 21, 22, 28 and 29 May. For his absences on
2, 4 and 11 January and 28 and 29 April, he was given a written warning dated 9 May 1997 that
he had already incurred five (5) AWOPs and that further absences would be subject to
disciplinary action. For his absences on 28 and 29 April and 7 and 8 May, respondent was
alleged to have falsified his medical consultation card by stating therein that he was granted sick
leave by the plant clinic on said dates when in truth he was not.
After the completion of the investigation, SMC concluded that respondent committed the
offenses of excessive AWOPs and falsification of company records or documents, and
accordingly dismissed him.
On 30 March 1998, respondent filed a complaint for illegal dismissal against SMC. The labor
arbiter believed that respondent had committed the absences pointed out by SMC but found the
imposition of termination of employment based on his AWOPs to be disproportionate since SMC
failed to show by clear and convincing evidence that it had strictly implemented its company
policy on absences. It also noted that termination based on the alleged falsification of company
records was unwarranted in view of SMCs failure to establish respondents guilt.
The appellate court also held that respondents AWOPs did not warrant his dismissal in view of
SMCs inconsistent implementation of its company policies. It could not understand why
respondent was given a mere warning for his absences on 28 and 29 April which constituted his
5th and 6th AWOPs, respectively, when these should have merited suspension under SMCs
policy. According to the appellate court, since respondent was merely warned, logically said
absences were deemed committed for the first time; thus, it follows that the subject AWOPs did
not justify his dismissal because under SMCs policy, the 4th to 9th AWOPs are meted the
corresponding penalty only when committed for the second time.
Issue: WON the Court of Appeals erred in sustaining the findings of the labor arbiter and the
NLRC and in dismissing SMCs claims that respondent was terminated from service with just
cause.
Held:
Proof beyond reasonable doubt is not required as a basis for judgment on the legality of an
employers dismissal of an employee, nor even preponderance of evidence for that matter,
substantial evidence being sufficient. In the instant case, while there may be no denying that
respondents medical card had falsified entries in it, SMC was unable to prove, by substantial
evidence, that it was respondent who made the unauthorized entries. Besides, SMCs (Your)
Guide on Employee Conduct punishes the act of falsification of company records or documents;
it does not punish mere possession of a falsified document.
Respondent cannot feign surprise nor ignorance of the earlier AWOPs he had incurred. He was
given a warning for his 2, 4, and 11 January and 26, 28, and 29 April 1997 AWOPs. In the same
warning, he was informed that he already had six AWOPs for 1997. He admitted that he was
absent on 7 and 8 May 1997. He was also given notices to explain his AWOPs for the period 26

May to 2 June 1997, which he received but refused to acknowledge. It does not take a genius to
figure out that as early as June 1997, he had more than nine AWOPs.
In any case, when SMC imposed the penalty of dismissal for the 12th and 13th AWOPs, it was
acting well within its rights as an employer. An employer has the prerogative to prescribe
reasonable rules and regulations necessary for the proper conduct of its business, to provide
certain disciplinary measures in order to implement said rules and to assure that the same would
be complied with. An employer enjoys a wide latitude of discretion in the promulgation of
policies, rules and regulations on work-related activities of the employees.
It is axiomatic that appropriate disciplinary sanction is within the purview of management
imposition. Thus, in the implementation of its rules and policies, the employer has the choice to
do so strictly or not, since this is inherent in its right to control and manage its business
effectively. Consequently, management has the prerogative to impose sanctions lighter than those
specifically prescribed by its rules, or to condone completely the violations of its erring
employees. Of course, this prerogative must be exercised free of grave abuse of discretion,
bearing in mind the requirements of justice and fair play.
All told, we find that SMC acted well within its rights when it dismissed respondent for his
numerous absences. Respondent was afforded due process and was validly dismissed for cause.
Petition granted
From Atty. Alba^^
Posted October 23, 2012 by vbdiaz in LABOR LAW

PLDT vs. NLRC Leave a comment


PLDT vs. NLRC
G.R. No. 80609 August 23, 1988
Facts:
Abucay, a traffic operator of the PLDT, was accused by two complainants of having demanded
and received from them the total amount of P3,800.00 in consideration of her promise to
facilitate approval of their applications for telephone installation. Investigated and heard, she
was found guilty as charged and accordingly separated from the service. She went to the
Ministry of Labor and Employment claiming she had been illegally removed. After consideration
of the evidence and arguments of the parties, the company was sustained and the complaint was
dismissed for lack of merit. Nevertheless, the dispositive portion of labor arbiters decision
declared:
WHEREFORE, the instant complaint is dismissed for lack of merit.

Considering that Dr. Bangayan and Mrs. Martinez are not totally blameless in the light of the fact
that the deal happened outhide the premises of respondent company and that their act of giving
P3,800.00 without any receipt is tantamount to corruption of public officers, complainant must
be given one month pay for every year of service as financial assistance.
Both the petitioner and the private respondent appealed to the National Labor Relations Board,
which upheld the said decision in toto and dismissed the appeals. The private respondent took no
further action, thereby impliedly accepting the validity of her dismissal. The petitioner, however,
is now before us to question the affirmance of the above- quoted award as having been made
with grave abuse of discretion.
The position of the petitioner is simply stated: It is conceded that an employee illegally
dismissed is entitled to reinstatement and backwages as required by the labor laws. However, an
employee dismissed for cause is entitled to neither reinstatement nor backwages and is not
allowed any relief at all because his dismissal is in accordance with law. In the case of the private
respondent, she has been awarded financial assistance equivalent to ten months pay
corresponding to her 10 year service in the company despite her removal for cause. She is,
therefore, in effect rewarded rather than punished for her dishonesty, and without any legal
authorization or justification. The award is made on the ground of equity and compassion, which
cannot be a substitute for law. Moreover, such award puts a premium on dishonesty and
encourages instead of deterring corruption.
For its part, the public respondent claims that the employee is sufficiently punished with her
dismissal. The grant of financial assistance is not intended as a reward for her offense but merely
to help her for the loss of her employment after working faithfully with the company for ten
years. In support of this position, the Solicitor General cites the cases of Firestone Tire and
Rubber Company of the Philippines v. Lariosa and Soco v. Mercantile Corporation of Davao,
where the employees were dismissed for cause but were nevertheless allowed separation pay on
grounds of social and compassionate justice.
Issue: WON Separation pay is proper.
Held:
We hold that henceforth separation pay shall be allowed as a measure of social justice only in
those instances where the employee is validly dismissed for causes other than serious misconduct
or those reflecting on his moral character. Where the reason for the valid dismissal is, for
example, habitual intoxication or an offense involving moral turpitude, like theft or illicit sexual
relations with a fellow worker, the employer may not be required to give the dismissed employee
separation pay, or financial assistance, or whatever other name it is called, on the ground of
social justice.
A contrary rule would, as the petitioner correctly argues, have the effect, of rewarding rather than
punishing the erring employee for his offense. And we do not agree that the punishment is his
dismissal only and that the separation pay has nothing to do with the wrong he has committed.
Of course it has. Indeed, if the employee who steals from the company is granted separation pay

even as he is validly dismissed, it is not unlikely that he will commit a similar offense in his next
employment because he thinks he can expect a like leniency if he is again found out. This kind of
misplaced compassion is not going to do labor in general any good as it will encourage the
infiltration of its ranks by those who do not deserve the protection and concern of the
Constitution.
The policy of social justice is not intended to countenance wrongdoing simply because it is
committed by the underprivileged. At best it may mitigate the penalty but it certainly will not
condone the offense. Compassion for the poor is an imperative of every humane society but only
when the recipient is not a rascal claiming an undeserved privilege. Social justice cannot be
permitted to be refuge of scoundrels any more than can equity be an impediment to the
punishment of the guilty. Those who invoke social justice may do so only if their hands are clean
and their motives blameless and not simply because they happen to be poor. This great policy of
our Constitution is not meant for the protection of those who have proved they are not worthy of
it, like the workers who have tainted the cause of labor with the blemishes of their own character.
Applying the above considerations, we hold that the grant of separation pay in the case at bar is
unjustified. The private respondent has been dismissed for dishonesty, as found by the labor
arbiter and affirmed by the NLRC and as she herself has impliedly admitted. The fact that she
has worked with the PLDT for more than a decade, if it is to be considered at all, should be taken
against her as it reflects a regrettable lack of loyalty that she should have strengthened instead of
betraying during all of her 10 years of service with the company. If regarded as a justification for
moderating the penalty of dismissal, it will actually become a prize for disloyalty, perverting the
meaning of social justice and undermining the efforts of labor to cleanse its ranks of all
undesirables.
Petition granted
From Atty. Alba
Posted October 23, 2012 by vbdiaz in LABOR LAW

Areno vs. Skycable Leave a comment


Areno vs. Skycable
G.R. No. 180302
Facts:
On January 17, 1995, petitioner was employed as a cable technician by respondent Skycable
PCC-Baguio. On January 17, 2002, an accounting clerk of respondent, Soriano, sent to the
human resource manager a letter-complaint against petitioner alleging that on two separate
occasions, the latter spread false rumors about her (the first in the middle of 2001 and the second
on December 22, 2001). On January 27, 2002, she was again insulted by petitioner when the
latter approached her and said that she was seen going out with Aldrin Estrada, their field service

supervisor, at Central Park, Baguio City. During that incident, petitioner uttered, Ikaw lang ang
nakakaalam ng totoo with malicious intent and in a provocative manner. Soriano averred that
petitioners unscrupulous behavior constituted serious and grave offense in violation of the
companys Code of Discipline.
On the same day, respondent issued a Memorandum requiring petitioner to submit an explanation
within 76 hours from notice thereof. Petitioner submitted his written explanation dated January
23, 2002 denying all the allegations in Sorianos letter-complaint and further denying having
uttered the statement imputed on him.
An administrative investigation was accordingly conducted on January 31, 2002. In a Memo
dated February 6, 2002, the investigating committee found petitioner guilty of having made
malicious statements against Soriano during the January 7, 2002 conversation, which is
categorized as an offense under the Company Code of Discipline. Consequently, petitioner was
suspended for three days without pay effective February 13-15, 2002. The Memo was allegedly
served on February 7, 2002 but petitioner refused to sign it.
Notwithstanding the suspension order, however, petitioner still reported for work on February 13,
2002. By reason thereof, respondent sent petitioner a letter denominated as 1st Notice of
Termination requiring him to explain in writing why he should not be terminated for
insubordination. On February 18, 2002, petitioner inquired from respondent whether he is
already dismissed or merely suspended since he was refused entry into the company premises on
February 14, 2002. Respondent replied that petitioner was merely suspended and gave him
additional time to tender his written explanation to the 1st Notice of Termination.
On March 2, 2002, petitioner again wrote to respondent, this time requesting for further
investigation on his alleged act of spreading rumors against Soriano in order for him to confront
his accuser and present his witnesses with the assistance of counsel. Respondent denied the
request reiterating that there has been substantial compliance with due process and that a
reinvestigation is moot because the suspension was already served.
Anent the new charge of insubordination, petitioner submitted to respondent his written
explanation averring that he still reported for work on the first day of his suspension because the
accusation of Soriano is baseless and her testimony is hearsay. Besides, according to petitioner,
he did not defy any order related to his duties, no representative of the management prevented
him from working and that reporting to work without being paid for the service he rendered on
that day did not in any way affect the companys productivity.
On March 15, 2002, an investigation on the insubordination case was conducted which was
attended by the parties and their respective counsels. Through a Final Notice of Termination
dated April 1, 2002, petitioner was dismissed from service on the ground of insubordination or
willful disobedience in complying with the suspension order.
Issue: WON petitioner was validly dismissed from work due to insubordination.
Held:

As a just cause for dismissal of an employee under Article 282 of the Labor Code, willful
disobedience of the employers lawful orders requires the concurrence of two elements: (1) the
employees assailed conduct must have been willful, i.e., characterized by a wrongful and
perverse attitude; and (2) the order violated must have been reasonable, lawful, made known to
the employee, and must pertain to the duties which he had been engaged to discharge. Both
requisites are present in the instant case. It is noteworthy that upon receipt of the notice of
suspension, petitioner did not question such order at the first instance. He immediately defied
the order by reporting on the first day of his suspension. Deliberate disregard or disobedience of
rules by the employee cannot be countenanced. It may encourage him to do even worse and will
render a mockery of the rules of discipline that employees are required to observe.
Petitioner was served the first notice of termination and was given time to submit his written
explanation. A hearing was conducted wherein both parties with their respective counsels were
present. After finding cause for petitioners termination, a final notice apprising him of the
decision to terminate his employment was served. All things considered, respondent validly
dismissed petitioner for cause after complying with the procedural requirements of the law.
Petition denied
From Atty. Alba^^
Posted October 23, 2012 by vbdiaz in LABOR LAW

Arco Metal vs. Samahan ng Manggagawa sa Arco Leave a


comment
Arco Metal vs. Samahan ng Manggagawa sa Arco
G.R. No. 170734
Facts:
Petitioner is a company engaged in the manufacture of metal products, whereas respondent is the
labor union of petitioners rank and file employees. Sometime in December 2003, petitioner
paid the 13th month pay, bonus, and leave encashment of three union members in amounts
proportional to the service they actually rendered in a year, which is less than a full 12 months.
Respondent protested the prorated scheme, claiming that on several occasions petitioner did not
prorate the payment of the same benefits to 7 employees who had not served for the full 12
months. The payments were made in 1992, 1993, 1994, 1996, 1999, 2003, and 2004. According
to respondent, the prorated payment violates the rule against diminution of benefits under Article
100 of the Labor Code. Thus, they filed a complaint before the NCMB. The parties submitted
the case for voluntary arbitration.
The voluntary arbitrator, Mangabat, ruled in favor of petitioner and found that the giving of the
contested benefits in full, irrespective of the actual service rendered within one year has not

ripened into a practice. He noted the affidavit of Baingan, manufacturing group head of
petitioner, which states that the giving in full of the benefit was a mere error. He also interpreted
the phrase for each year of service found in the pertinent CBA provisions to mean that an
employee must have rendered one year of service in order to be entitled to the full benefits
provided in the CBA.
The CA ruled that the CBA did not intend to foreclose the application of prorated payments of
leave benefits to covered employees. The appellate court found that petitioner, however, had an
existing voluntary practice of paying the aforesaid benefits in full to its employees, thereby
rejecting the claim that petitioner erred in paying full benefits to its seven employees. The
appellate court noted that aside from the affidavit of petitioners officer, it has not presented any
evidence in support of its position that it has no voluntary practice of granting the contested
benefits in full and without regard to the service actually rendered within the year. It also
questioned why it took petitioner 11 years before it was able to discover the alleged error.
Issue: WON the grant of 13th month pay, bonus, and leave encashment in full regardless of
actual service rendered constitutes voluntary employer practice and, consequently, the prorated
payment of the said benefits does not constitute diminution of benefits under Article 100 of the
Labor Code.
Held:
Petitioner claims that its full payment of benefits regardless of the length of service to the
company does not constitute voluntary employer practice. It points out that the payments had
been erroneously made and they occurred in isolated cases in the years 1992, 1993, 1994, 1999,
2002 and 2003. According to petitioner, it was only in 2003 that the accounting department
discovered the error when there were already 3 employees involved with prolonged absences
and the error was corrected by implementing the pro-rata payment of benefits pursuant to law
and their existing CBA. It adds that the seven earlier cases of full payment of benefits went
unnoticed considering the proportion of one employee concerned (per year) vis vis the 170
employees of the company. Petitioner describes the situation as a clear oversight which
should not be taken against it. To further bolster its case, petitioner argues that for a grant of a
benefit to be considered a practice, it should have been practiced over a long period of time and
must be shown to be consistent, deliberate and intentional, which is not what happened in this
case. Petitioner tries to make a case out of the fact that the CBA has not been modified to
incorporate the giving of full benefits regardless of the length of service, proof that the grant has
not ripened into company practice.
Any benefit and supplement being enjoyed by employees cannot be reduced, diminished,
discontinued or eliminated by the employer. The principle of non-diminution of benefits is
founded on the Constitutional mandate to protect the rights of workers and promote their
welfare, and to afford labor full protection. Said mandate in turn is the basis of Article 4 of
the Labor Code which states that all doubts in the implementation and interpretation of this
Code, including its implementing rules and regulations shall be rendered in favor of labor.
Jurisprudence is replete with cases which recognize the right of employees to benefits which
were voluntarily given by the employer and which ripened into company practice. Thus in

Davao Fruits Corporation v. Associated Labor Unions, et al. where an employer had freely and
continuously included in the computation of the 13th month pay those items that were expressly
excluded by the law, we held that the act which was favorable to the employees though not
conforming to law had thus ripened into a practice and could not be withdrawn, reduced,
diminished, discontinued or eliminated.
In the years 1992, 1993, 1994, 1999, 2002 and 2003, petitioner had adopted a policy of freely,
voluntarily and consistently granting full benefits to its employees regardless of the length of
service rendered. True, there were only a total of seven employees who benefited from such a
practice, but it was an established practice nonetheless. Jurisprudence has not laid down any rule
specifying a minimum number of years within which a company practice must be exercised in
order to constitute voluntary company practice. Thus, it can be 6 years, 3 years, or even as short
as 2 years. Petitioner cannot shirk away from its responsibility by merely claiming that it was a
mistake or an error, supported only by an affidavit of its manufacturing group head.
In cases involving money claims of employees, the employer has the burden of proving that
the employees did receive the wages and benefits and that the same were paid in
accordance with law.
Petition denied
From Atty. Alba^^
Posted October 23, 2012 by vbdiaz in LABOR LAW

FEU-NRMF and LUNA versus FEU-NRMFEA-AFW et al


Leave a comment
FAR EASTERN UNIVERSITY DR. NICANOR REYES MEDICAL FOUNDATION
(FEU-NRMF) and LILIA P. LUNA. M.D., Petitioners, versus FEU-NRMF EMPLOYEES
ASSOCIATION-ALLIANCE OF FILIPINO WORKERS (FEU-NRMFEA-AFW), et al
Respondents., G.R. No. 168362; October 12, 2006
FACTS:
In 1994, petitioner FEU-NRMF (a medical institution organized and existing under the
Philippine laws), and respondent union (a legitimate labor organization and is the duly
recognized representative of the rank and file employees of petitioner), entered into a CBA that
will expire on 30 April 1996. In view of the forthcoming expiry, respondent union sent a letterproposal to petitioner FEU-NRMF stating their economic and non-economic proposals for the
negotiation of the new CBA.
Petitioner FEU-NRMF rejected respondent unions demands and proposed to maintain
the same provisions of the old CBA reasoning that due to financial constraints, it cannot afford
to accede to a number of their demands. In an effort to arrive at a compromise, subsequent
conciliation proceedings were conducted before the NCMB, but the negotiation failed.
Respondent union filed a Notice of Strike before NCMB on the ground of bargaining deadlock.

Union staged a strike.


Petitioner FEU-NRMF filed a Petition for the Assumption of Jurisdiction or for
Certification of Labor Dispute with the NLRC, underscoring the fact that it is a medical
institution engaged in the business of providing health care for its patients. Secretary of Labor
granted the petition and an Order assuming jurisdiction over the labor dispute was issued,
thereby prohibiting any strike or lockout and enjoining the parties from committing any acts
which may exacerbate the situation.
September 6, 1996, Francisco Escuadra, the NLRC process server, certified that, on
September 5, 1996 at around 4:00 P.M., he attempted to serve a copy of the Assumption of
Jurisdiction Order (AJO) to the union officers but since no one was around at the strike area, he
just posted copies of the said Order at several conspicuous places within the premises of the
hospital.
Striking employees continued holding a strike until 12 September 1996, claiming that
they had no knowledge that the Secretary of Labor already assumed jurisdiction over the pending
labor dispute as they were not able to receive a copy of the AJO.
Secretary of Labor issued another Order directing all the striking employees to return to
work and the petitioner FEU-NRMF to accept them under the same terms and conditions
prevailing before the strike. A Return to Work Agreement was executed by the disputing parties.
Subsequently, petitioner FEU-NRMF filed a case before the NLRC, contending that
respondent union staged the strike in defiance of the AJO, hence, it was illegal. LA declared the
strike illegal and allowed dismissal of union officers for conducting the strike in defiance of the
AJO. Respondent union filed an Appeal before the NLRC. NLRC affirmed in toto the Decision
of the LA. Respondent union filed MR, it was denied. Respondent union brought a Petition for
Certiorari before CA. CA granted the Petition and reversed the Resolutions of NLRC. Petitioner
filed MR but it was denied. Hence this petition.
ISSUE: Whether the service of the AJO was validly effected by the process server so as to bind
the respondent union and hold them liable for the acts committed subsequent to the issuance of
the said Order.
RULING:
The process server resorted to posting the Order when personal service was rendered
impossible since the striking employees were not present at the strike area. This mode of
service, however, is not sanctioned by either the NLRC Revised Rules of Procedure or the
Revised Rules of Court.
The pertinent provisions of the NLRC Revised Rules of Procedure read:
Section 6. Service of Notices and Resolutions.
(a) Notices or summons and copies of orders, shall be served on the parties to the case
personally by the Bailiff or duly authorized public officer within 3 days from receipt thereof or
by registered mail; Provided that in special circumstances, service of summons may be effected
in accordance with the pertinent provisions of the Rules of Court; Provided further, that in cases
of decisions and final awards, copies thereof shall be served on both parties and their counsel or
representative by registered mail; Provided further, that in cases where a party to a case or his
counsel on record personally seeks service of the decision upon inquiry thereon, service to said
party shall be deemed effected upon actual receipt thereof; Provided finally, that where parties
are so numerous, service shall be made on counsel and upon such number of complainants, as

may be practicable, which shall be considered substantial compliance with Article 224(a) of the
Labor Code, as amended.
An Order issued by the Secretary of Labor assuming jurisdiction over the labor dispute is
not a final judgment for it does not dispose of the labor dispute with finality. Consequently,
the rule on service of summons and orders, and not the proviso on service of decisions and final
awards, governs the service of the Assumption of Jurisdiction Order.
Under the NLRC Revised Rules of Procedure, service of copies of orders should be made
by the process server either personally or through registered mail. However, due to the urgent
nature of the AJO and the public policy underlying the injunction carried by the issuance of the
said Order, service of copies of the same should be made in the most expeditious and effective
manner, without any delay, ensuring its immediate receipt by the intended parties as may be
warranted under the circumstances. Thus, personal service is the proper mode of serving the
AJO.
Personal service effectively ensures that the notice desired under the constitutional
requirement of due process is accomplished. If, however, efforts to find the party concerned
personally would make prompt service impossible, service may be completed by substituted
service, that is, by leaving a copy, between the hours of eight in the morning and six in the
evening, at the partys or counsels residence, if known, with a person of sufficient age and
discretion then residing therein (RULE 12 of Rev Rules of Court).
Substituted service derogates the regular method of personal service. It is therefore
required that statutory restrictions for effecting substituted service must be strictly, faithfully and
fully observed. Failure to comply with this rule renders absolutely void the substituted service
along with the proceedings taken thereafter. The underlying principle of this rigid requirement is
that the person, to whom the orders, notices or summons are addressed, is made to answer for the
consequences of the suit even though notice of such action is made, not upon the party
concerned, but upon another whom the law could only presume would notify such party of the
pending proceedings.
In the case at bar, presumption of receipt of the copies of the Assumption of Jurisdiction
Order AJO could not be taken for granted considering the adverse effect in case the parties failed
to heed to the injunction directed by such Order. Defiance of the assumption and return-to-work
orders of the Secretary of Labor after he has assumed jurisdiction is a valid ground for the loss of
employment status of any striking union officer or member. Employment is a property right of
which one cannot be deprived of without due process. Due process here would demand that the
respondent union be properly notified of the Assumption of Jurisdiction Order of the Secretary
of Labor enjoining the strike and requiring its members to return to work. Thus, there must be a
clear and unmistakable proof that the requirements prescribed by the Rules in the manner of
effecting personal or substituted service had been faithfully complied with.
Merely posting copies of the AJO does not satisfy the rigid requirement for proper
service outlined by the above stated rules. Needless to say, the manner of service made by the
process server was invalid and irregular. Respondent union could not therefore be adjudged to
have defied the said Order since it was not properly apprised thereof. Accordingly, the strike
conducted by the respondent union was valid under the circumstances.
WHEREFORE, premises considered, the instant Petition is DENIED. Costs against the
petitioner

from Atty. Daan^^


Posted October 23, 2012 by vbdiaz in LABOR LAW

OWWA,represented by Administrator Roque vs


CHAVEZ, etal. Leave a comment
OVERSEAS WORKERS WELFARE ADMINISTRATION,represented by Administrator
Marianito D. Roque, Petitioner vs ATTY. CESAR L. CHAVEZ, etal. Respondents.
G.R. No. 169802; June 8, 2007
FACTS:
OWWA traces its beginnings to 1 May 1977, when the Welfare and Training Fund for
Overseas
main objective, inter alia, of providing social and welfare services to OFW, including insurance
coverage, social work, legal and placement assistance, cultural and remittances services,
and the like. On 1 May 1980, PD 1694 was signed into law, formalizing the operations of a
comprehensive Welfare Fund (Welfund), as authorized and created under Letter of Instructions
No. 537. On 16 January 1981, PD 1809 was promulgated, amending certain provisions of PD
1694. Subsequently, EO 126 was passed renaming the Welfare Fund as the OWWA.
January 9, 2004, as there was yet no formal OWWA structure duly approved by the DBM
and CSC, the OWWA Board of Trustees passed Resolution No. 001, Series of 2004, bearing the
title
the organizational structure and staffing pattern of the OWWA.
March 24, 2004, DBM Secretary Emilia T. Boncodin approved the organizational
structure and staffing pattern of the OWWA. In her approval thereof, she stated that the total
funding requirements for the revised organizational structure shall be P107,546,379 for 400
positions.
May
01, advising the officials and employees of the OWWA that the DBM had recently approved
OWWAs
also
placement
staffing pattern.
June 3, 2004, DOLE Secretary Sto. Tomas issued Administrative Order No. 171 creating
a Placement Committee to evaluate qualifications of employees, and to recommend their
appropriate placement in the new organizational chart, functional statements and staffing pattern
of the OWWA.
June 18, 2004, DOLE Acting Secretary Imson issued Administrative Order No. 186,
Series
pattern of the OWWA.
June
Organizational Structure of the OWWA, as approved by OWWA Board Resolution No. 001,
Series

petitioner OWWA and its Board of Trustees to prevent the implementation of the said Board
Resolution. According to the respondents, the resulting decrease in the number of employees
due to Organizational Structure will result in the constructive dismissal of at least 110
employees. Meanwhile, the deployment of the regular central office personnel to the regional
offices
validity of the new organizational structure of the OWWA. In fine, they contended that the same
is null and void; hence, its implementation should be prohibited.
RTC granted respondents prayer for a writ of preliminary injunction. Petitioner, thru
OSG
and
the RTC to restrain, for the meantime, the implementation of OWWAs reorganization to
prevent injury until after the main case is heard and decided. Petitioner, thru OSG filed the
instant petition.
ISSUE:
Whether CA erred in affirming the RTC in its grant of the assailed writ of preliminary
injunction.
RULING:
Section 1, Rule 58 of the Rules of Court, defines a preliminary injunction as an order
granted at any stage of an action prior to the judgment or final order requiring a party or a court,
an agency or a person to refrain from a particular act or acts. It persists until it is dissolved or
until the termination of the action without the court issuing a final injunction. To be entitled to an
injunctive writ,
and an urgent and paramount necessity for the writ to prevent serious damage. A writ
of
evidence required to justify the issuance of a writ of preliminary injunction in the hearing
thereon need not be conclusive or complete.
Preliminary injunction is merely a provisional remedy, an adjunct to the main case
subject to the latters outcome, the sole objective of which is to preserve the status quo until the
trial court hears fully the merits of the case. The status quo usually preserved by a preliminary
injunction is the last actual, peaceable and uncontested status which preceded the actual
controversy. The status quo ante litem is the state of affairs which is existing at the time of the
filing of the case. The trial court must not make use of its injunctive power to alter such status.
In
of jurisdiction because it did not maintain the status quo when it issued the writ of preliminary
injunction. Rather, it effectively restored the situation prior to the status quo, in effect, disposing
the issue of the main case without trial on the merits. What was preserved by the RTC was the
state
OWWA, and the subsequent administrative orders pursuant to its passing. The RTC forgot that
what is imperative in preliminary injunction cases is that the writ cannot be effectuated to
establish new relations between the parties.
Courts should avoid issuing a writ of preliminary injunction which would in effect
dispose of the main case without trial. In this case, the RTC also did not maintain the status

quo but restored the landscape before the implementation of OWWAs reorganization. In thus
issuing
by the trial court. What was done by the RTC was quite simply a disposition of the case without
trial.
the validity of the issuances released by the OWWA Board of Trustees, as well as the other
governmental bodies
staffing pattern of the OWWA.
This Court is not convinced that respondents were able to show a clear and unmistakable
legal right to warrant their entitlement to the writ. A mere blanket allegation that they are all
officers and employees of the OWWA without a showing of how they stand to be directly
injured
a right in esse. There was no showing that Respondents are the employees who are in grave
danger of being displaced due to the reorganization. Injunction is not a remedy to protect or
enforce contingent, abstract, or future rights; it will not issue to protect aright not in esse and
which may never arise, or to restrain an action which did not give rise to a cause of action.
The
main case pending before the trial court. Its annulment is outside the realm of the instant
Petition.
WHEREFORE, the Petition is GRANTED. The Writ of Preliminary Injunction issued by
the RTC is LIFTED and SET ASIDE.
from Atty. Daan^^
Posted October 23, 2012 by vbdiaz in LABOR LAW

CAPITOL MEDICAL CENTER, INC., v. HON.


CRESENCIANO B. TRAJANO, in his capacity as Secretary
of the Department of Labor and Employment, and
CAPITOL MEDICAL CENTER
EMPLOYEES ASSOCIATION-AFW Leave a comment
CAPITOL MEDICAL CENTER, INC., v. HON. CRESENCIANO B. TRAJANO, in his
capacity
as Secretary of the Department of Labor and Employment, and CAPITOL MEDICAL
CENTER EMPLOYEES ASSOCIATION-AFW
G.R. No. 155690/ June 30, 2005
FACTS:
Petitioner is a hospital with address at Panay Avenue corner Scout Magbanua Street,
Quezon City. Upon the other hand, Respondent is a duly registered labor union acting as the
certified collective bargaining agent of the rank-and-file employees of petitioner hospital.

Respondent sent petitioner a letter requesting a negotiation of their Collective Bargaining


Agreement (CBA).
Petitioner, however, challenged the unions legitimacy and refused to bargain with
respondent. Subsequently petitioner filed with the (BLR), Department of Labor and
Employment, a petition for cancellation of respondents certificate of registration.
For its part, respondent filed with the (NCMB), National Capital Region, a notice of strike.
Respondent alleged that petitioners refusal to bargain constitutes unfair labor practice.
Despite several conferences and efforts of the designated conciliator-mediator, the parties
failed to reach an amicable settlement.
Respondent staged a strike.
Former Labor Secretary Leonardo A. Quisumbing, now Associate Justice of this Court,
issued an Order assuming jurisdiction over the labor dispute and ordering all striking
workers to return to work and the management to resume normal operations, thus:
xxx all striking workers are directed to return to work within twenty-four (24) hours from
the receipt of this Order and the management to resume normal operations and accept
back all striking workers under the same terms and conditions prevailing before the
strike. Further, parties are directed to cease and desist from committing any act that may
exacerbate the situation.
Moreover, parties are hereby directed to submit within 10 days from receipt of this Order
proposals and counter-proposals leading to the conclusion of the collective bargaining
agreement in compliance with aforementioned Resolution of the Office as affirmed by the
Supreme Court. xxx
ISSUE:
Whether or not Secretary of Labor cannot exercise his powers under Article 263 (g) of the Labor
Code without observing the requirements of due process.
RULING:
The discretion to assume jurisdiction may be exercised by the Secretary of
Labor and Employment without the necessity of prior notice or hearing given to any of the
parties. The rationale for his primary assumption of jurisdiction can justifiably rest
on his own consideration of the exigency of the situation in relation to the national
interests.
xxx In labor disputes adversely affecting the continued operation of such hospitals, clinics
or medical institutions, it shall be the duty of the striking union or locking-out employer
to provide and maintain an effective skeletal workforce of medical and other health
personnel, whose movement and services shall be unhampered and unrestricted, as are
necessary to insure the proper and adequate protection of the life and health of its patients,
most especially emergency cases, for the duration of the strike or lockout. In such cases,

therefore, the Secretary of Labor and Employment is mandated to immediately


assume, within twenty-four (24) hours from knowledge of the occurrence of such
a strike or lockout, jurisdiction over the same or certify it to the Commission for
compulsory arbitration. For this purpose, the contending parties are strictly enjoined to
comply with such orders, prohibitions and/or injunctions as are issued by the Secretary of
Labor and Employment or the Commission, under pain of immediate disciplinary action,
including dismissal or loss of employment status or payment by the locking-out employer
of backwages, damages and other affirmative relief, even criminal prosecution against
either or both of them.
The foregoing notwithstanding, the President of the Philippines shall not be precluded
from determining the industries that, in his opinion, are indispensable to the national
interest, and from intervening at any time and assuming jurisdiction over any such labor
dispute in order to settle or terminate the same. xxx
from Atty. Soriano^^
Posted October 23, 2012 by vbdiaz in LABOR LAW

PLDT CO. INC., v. MANGGAGAWA NG


KOMUNIKASYON SA PILIPINAS and CA Leave a
comment
PHILIPPINE LONG DISTANCE TELEPHONE CO. INC., v. MANGGAGAWA NG
KOMUNIKASYON SA PILIPINAS and the COURT OF APPEALS,
G.R. No. 162783/ July 14, 2005
FACTS:
Petitioner Philippine Long Distance Telephone Co., Inc. (PLDT) is a domestic
corporation engaged in the telecommunications business.
Private respondent
Manggagawa ng Komunikasyon sa Pilipinas (MKP) is a labor union of rank and file
employees in PLDT.
The members of respondent union learned that a redundancy program would
be implemented by the petitioner. Thereupon it filed a Notice of Strike with the
National Conciliation and Mediation Board (NCMB) on 04 November 2002. The Notice
fundamentally contained the following:
UNFAIR LABOR PRACTICES, to wit:
1.
PLDTs abolition of the Provisioning Support Division, in violation of the duty to
bargain collectively with MKP in good faith.

2.
PLDTs unreasonable refusal to honor its commitment before this Honorable
Office that it will provide MKP its comprehensive plan/s with respect to personnel
downsizing/reorganization and closure of exchanges. Such refusal violates its duty to
bargain collectively with MKP in good faith.
3.
PLDTs continued hiring of contractual, temporary, project and casual
employees for regular jobs performed by union members, resulting in the decimation of
the union membership and in the denial of the right to self-organization to the concerned
employees.
4.
PLDTs gross violation of the legal and CBA provisions on overtime work and
compensation.
5.
PLDTs gross violation of the CBA provisions on promotions and job grade reevaluation or reclassification.
On 11 November 2002, another Notice of Strike was filed by the private respondent,
which contained the following: UNFAIR LABOR PRACTICES, to wit: PLDTs alleged
restructuring of its GMM Operation Services.
A number of conciliation meetings, conducted by the NCMB, National Capital Region,
were held between the parties. However, these efforts proved futile.
On 23 December 2002, the private respondent staged a strike. On 31 December 2002,
three hundred eighty three (383) union members were terminated from service pursuant
to PLDTs redundancy program.
On 02 January 2003, the Secretary, Patricia Sto. Tomas, issued an Order[4] in NCMBNCR-NS-11-405-02 and NCMB-NCR-NS-11-412-02. Portions of the Order are reproduced
hereunder:
xxx Accordingly, the strike staged by the Union is hereby enjoined. All striking workers
are hereby directed to return to work within twenty four (24) hours from receipt of this
Order, except those who were terminated due to redundancy. The employer is hereby
enjoined to accept the striking workers under the same terms and conditions prevailing
prior to the strike. The parties are likewise directed to cease and desist from committing
any act that might worsen the situation. xxx
ISSUE:
WHETHER THE SUBJECT ORDERS OF THE SECRETARY OF THE DOLE EXCLUDING
FROM THE RETURN-TO-WORK

ORDER THE WORKERS DISMISSED DUE TO THE REDUNDANCY PROGRAM


OF PETITIONER, ARE VALID OR NOT.
RULING:
. . . Assumption of jurisdiction over a labor dispute, or as in this case the
certification of the same to the NLRC for compulsory arbitration, always
co-exists with an order for workers to return to work immediately and for
employers to readmit all workers under the same terms and conditions
prevailing before the strike or lockout.
Time and again, this Court has held that when an official bypasses the law on the
asserted ground of attaining a laudable objective, the same will not be maintained if the
intendment or purpose of the law would be defeated.[30]
One last piece. Records would show that the strike occurred on 23 December
2002. Article 263(g) directs that the employer must readmit all workers under the
same terms and conditions prevailing before the strike. Since the strike was held on the
aforementioned date, then the condition prevailing before it, which was the condition
present on 22 December 2002, must be maintained.
Undoubtedly, on 22 December 2002, the members of the private respondent who
were dismissed due to alleged redundancy were still employed by the petitioner and
holding their respective positions. This is the status quo that must be maintained.
Valid.
from Atty. Soriano^^
Posted October 23, 2012 by vbdiaz in LABOR LAW

National Union of Workers in Hotels V. NLRC Leave a


comment
National Union of Workers in Hotels V. NLRC
This is a Petition for Review on Certiorari under Rule 45, assailing the Decision1 rendered
by the Court of Appeals, which affirmed the Resolution2of the National Labor Relations
Commission, dismissing for lack of merit the complaint for unfair labor practice filed by
petitioner National Union of Workers in Hotels, Restaurants and Allied Industries-Manila
Pavilion Hotel (NUWHRAIN) against Manila Pavilion Hotel (the Hotel).
Petitioner NUWHRAIN is a legitimate labor organization composed of rank-and-file
employees of the Hotel,3 while respondent Acesite Philippines Hotel Corporation is the
owner and operator of said Hotel.4

The Hotel entered into a Collective Bargaining Agreement with HI-MANILA PAVILION
HOTEL LABOR UNION (HIMPHLU), the exclusive bargaining agent of the rank-and-file
employees of the Hotel. Both parties consented that the representation aspect and other
non-economic provisions of the Collective Bargaining Agreement were to be effective
for five years or until 30 June 2005; and the economic provisions of the same were to be
effective for three years or until 30 June 2003. The parties subsequently re-negotiated the
economic provisions of the Collective Bargaining Agreement and extended the term of their
effectivity for another two years or until 30 June 2005.5
During the 60-day freedom period which preceded the expiration of the Collective
Bargaining Agreement, starting on 1 May 2005 and ending on 30 June 2005, the Hotel and
HIMPHLU negotiated the extension of the provisions of the existing Collective Bargaining
Agreement for two years, effective 1 July 2005 to 30 June 2007. The parties signed the
Memorandum of Agreement on 20 May 2005 and the employees ratified it on 27 May
2005.6
On 21 June 2005, NUWHRAIN was accorded by the Labor Relations Division of
the Department of Labor and Employment (DOLE) the status of a legitimate labor
organization.7 Thereafter, NUWHRAIN exercised the right to challenge the majority
status of the incumbent union, HIMPHLU, by filing a Petition for Certification Election
on 28 June 2005.8
On 5 July 2007, the Industrial Relations Division of the DOLE allowed the registration of
the Memorandum of Agreement executed between HIMPHLU and the Hotel, extending the
effectivity of the existing Collective Bargaining Agreement for another two years.9
After the lapse of the 60-day freedom period, but pending the disposition of the
Petition for Certification Election filed by NUWHRAIN, HIMPHLU served the Hotel
with a written demand dated 28 July 200510 for the dismissal of 36 employees
following their expulsion from HIMPHLU for alleged acts of disloyalty and violation
of its Constitution and by-laws. An Investigation Report11 was attached to the said
written demand, stating that the 36 employees, who were members of HIMPHLU,
joined NUWHRAIN, in violation of Section 2, Article IV of the Collective Bargaining
Agreement, which provided for a union security clause.12
On 1 August 2005, the Hotel issued Disciplinary Action Notices13 (Notices) to the 36
employees identified in the written demand of HIMPHLU. The Notices directed the 36
employees to submit a written explanation for their alleged acts of disloyalty and violation of
the union security clause for which HIMPHLU sought their dismissal.
NLRC decided that there was no unfair labor practice.NUWHRAIN filed a Motion for
Reconsideration of the foregoing NLRC Resolution. It was denied by the NLRC in another
Resolution dated 30 June 2006.20 Thus, NUWHRAIN filed a Petition for Certiorari before
the Court of Appeals, docketed as C.A. G.R. SP No. 96171.

In the meantime, on 16 June 2006, the Certification Election for regular rank and file
employees of the Hotel was held, which HIMPHLU won. It was accordingly certified
as the exclusive bargaining agent for rank and file employees of the Hotel.21
On 30 May 2007, the Court of Appeals promulgated its Decision2 upholding the Resolution
of the NLRC. It declared that the Hotel had acted prudently when it issued the Notices to
the 36 employees after HIMPHLU demanded their dismissal. It clarified that these Notices
did not amount to the termination of the employees concerned but merely sought their
explanation on why the union security clause should not be applied to them. The appellate
court also gave credence to the denial by the officers of the respondent and the Hotel
that they made statements favoring HIMPHLU over NUWHRAIN during the reconciliatory
conferences. The Court of Appeals further noted that the unhampered organization and
registration of NUWHRAIN negated its allegation that the Hotel required its employees not
to join a labor organization as a condition for their employment.
NUWHRAINs Motion for Reconsideration of the aforementioned Decision of the Court of
Appeals was denied by the same court in a Resolution.23
Hence, the present Petition, in which NUWHRAIN makes the following assignment of
errors:
ISSUE: Whether the dismissal of the subject employees in accordance with CBAs Union
Security Clause deemed unfair labor practice.
The instant Petition lacks merit.
NUWHRAIN maintains that the respondent committed unfair labor practice when (1) the
Hotel issued the Notices to the 36 employees, former members of HIMPHLU, who switched
allegiance to NUWHRAIN; and (2) the officers of the respondent and the Hotel allegedly
uttered statements during the reconciliatory conferences indicating their preference for
HIMPHLU and their disapproval of NUWHRAIN. This argument is specious.
Union security is a generic term which is applied to and comprehends closed
shop, union shop, maintenance of membership or any other form of agreement
which imposes upon employees the obligation to acquire or retain union membership
as a condition affecting employment.25 Article 248(e) of the Labor Code recognizes the
effectivity of a union shop clause:
The law allows stipulations for union shop and closed shop as a means of encouraging
workers to join and support the union of their choice in the protection of their rights and
interests vis--vis the employer. By thus promoting unionism, workers are able to negotiate
with management on an even playing field and with more persuasiveness than if they
were to individually and separately bargain with the employer.26 In Villar v. Inciong,27 this
Court held that employees have the right to disaffiliate from their union and form a new
organization of their own; however, they must suffer the consequences of their separation
from the union under the security clause of the Collective Bargaining Agreement.

In the present case, the Collective Bargaining Agreement includes a union security
provision.28 To avoid the clear possibility of liability for breaching the union security clause
of the Collective Bargaining Agreement and to protect its own interests, the only sensible
option left to the Hotel, upon its receipt of the demand of HIMPHLU for the dismissal of the
36 employees, was to conduct its own inquiry so as to make its own findings on whether
there was sufficient ground to dismiss the said employees who defected from HIMPHLU.
The issuance by the respondent of the Notices requiring the 36 employees to submit their
explanations to the charges against them was the reasonable and logical first step in a fair
investigation. It is important to note that the Hotel did not take further steps to terminate the
36 employees. Instead, it arranged for reconciliatory conferences between the contending
unions in order to avert the possibility of dismissing the 36 employees for violation of the
union security clause of the Collective Bargaining Agreement.
This Court, in Malayang Samahan ng Manggagawa sa M. Greenfield v. Ramos29 clearly
stated the general rule: the dismissal of an employee by the company pursuant to a labor
unions demand in accordance with a union security agreement does not constitute unfair
labor practice. An employer is not considered guilty of unfair labor practice if it merely
complied in good faith with the request of the certified union for the dismissal of employees
expelled from the union pursuant to the union security clause in the Collective Bargaining
Agreement.30 In the case at bar, there is even less possibility of sustaining a finding of guilt
for unfair labor practice where respondent did not dismiss the 36 employees, despite the
insistence of HIMPHLU, the sole bargaining agent for the rank and file employees of the
Hotel, on the basis of the union security clause of the Collective Bargaining Agreement. The
only act attributed to the respondent is its issuance of the Notices which, contrary to being
an unfair labor practice, even afforded the employees involved a chance to be heard.
In all, respondent had not committed any act which would constitute unfair labor practice.
IN VIEW OF THE FOREGOING, the instant Petition is DENIED. The assailed Decision
dated 30 May 2007 of the Court of Appeals in CA-G.R. SP No. 96171 is hereby AFFIRMED.
Costs against petitioner NUWHRAIN.
from Atty. Manalundong^^
Posted October 23, 2012 by vbdiaz in LABOR LAW

University of the Immaculate Conception vs Sec of Labor


Leave a comment
University of the Immaculate Conception vs Sec of Labor
GR 151379
Facts:

This case stemmed from the collective bargaining negotiations between petitioner University
of Immaculate Concepcion, Inc. (UNIVERSITY) and respondent The UIC Teaching and NonTeaching Personnel and Employees Union (UNION). The UNION, as the certified bargaining
agent of all rank and file employees of the UNIVERSITY, submitted its collective bargaining
proposals to the latter on February 16, 1994. However, one item was left unresolved and this was
the inclusion or exclusion of some positions in the scope of the bargaining unit.
The UNION it filed a notice of strike on the grounds of bargaining deadlock and unfair labor
practice. During the thirty (30) day cooling-off period, two union members were dismissed by
petitioner. Consequently, the UNION went on strike.
On January 23, 1995, the then Secretary of Labor, Ma. Nieves R. Confessor, issued an Order
assuming jurisdiction over the labor dispute.
On March 10, 1995, the UNION filed another notice of strike, this time citing as a reason
the UNIVERSITYs termination of the individual respondents. The UNION alleged that the
UNIVERSITYs act of terminating the individual respondents is in violation of the Order of the
Secretary of Labor.
On March 28, 1995, the Secretary of Labor issued another Order reiterating the directives
contained in the January 23, 1995 Order. Hence, the UNIVERSITY was directed to reinstate the
individual respondents under the same terms and conditions prevailing prior to the labor dispute.
The UNIVERSITY filed a MR. In the Order dated August 18, 1995, then Acting Secretary Jose
S. Brilliantes denied the MR, but modified the two previous Orders by adding:
Anent the Unions Motion, we find that superseding circumstances would not
warrant the physical reinstatement of the twelve (12) terminated employees.
Hence, they are hereby ordered placed under payroll reinstatement until the
validity of their termination is finally resolved.
Issue: WON payroll reinstatement, instead of actual reinstatement, is proper.
Held:
With respect to the Secretarys Order allowing payroll reinstatement instead of actual
reinstatement for the individual respondents herein, an amendment to the previous Orders
issued by her office, the same is usually not allowed. Article 263(g) of the Labor Code
aforementioned states that all workers must immediately return to work and all employers
must readmit all of them under the same terms and conditions prevailing before the strike or
lockout. The phrase under the same terms and conditions makes it clear that the norm is actual
reinstatement. This is consistent with the idea that any work stoppage or slowdown in that
particular industry can be detrimental to the national interest.

In ordering payroll reinstatement in lieu of actual reinstatement, then Acting Secretary of Labor
Jose S. Brillantes said:
Anent the Unions Motion, we find that superseding circumstances would not warrant the
physical
reinstatement of the twelve (12) terminated employees. Hence, they are hereby ordered placed
under payroll reinstatement until the validity of their termination is finally resolved.
As an exception to the rule, payroll reinstatement must rest on special circumstances that render
actual reinstatement impracticable or otherwise not conducive to attaining the purposes of the
law.
The superseding circumstances mentioned by the Acting Secretary of Labor no doubt refer
to the final decision of the panel of arbitrators as to the confidential nature of the positions
of the twelve private respondents, thereby rendering their actual and physical reinstatement
impracticable and more likely to exacerbate the situation. The payroll reinstatement in lieu of
actual reinstatement ordered in these cases, therefore, appears justified as an exception to the
rule until the validity of their termination is finally resolved. This Court sees no grave abuse of
discretion on the part of the Acting Secretary of Labor in ordering the same. Furthermore, the
issue has not been raised by any party in this case.
Petition denied.
from Atty. Alba^^
Posted October 23, 2012 by vbdiaz in LABOR LAW

Santa Rosa Coca Cola Plant Employee Union vs Coca Cola


Bottlers Phil Leave a comment
Santa Rosa Coca Cola Plant Employee Union vs Coca Cola Bottlers Phil
GR 164302-03
Facts:
The Sta. Rosa Coca-Cola Plant Employees Union (Union) is the sole and exclusive bargaining
representative of the regular daily paid workers and the monthly paid non-commission-earning
employees of the Coca-Cola Bottlers Philippines, Inc. (Company) in its Sta. Rosa, Laguna plant.
Upon the expiration of the CBA, the Union informed the Company of its desire to renegotiate
its terms. The CBA meetings commenced on July 26, 1999, where the Union and the Company
discussed the ground rules of the negotiations. The Union insisted that representatives
from the Alyansa ng mga Unyon sa Coca-Cola be allowed to sit down as observers in the
CBA meetings. The Union officers and members also insisted that their wages be based
on their work shift rates. For its part, the Company was of the view that the members of

the Alyansa were not members of the bargaining unit. The Alyansa was a mere aggregate of
employees of the Company in its various plants; and is not a registered labor organization. Thus,
an impasse ensued.
On August 30, 1999, the Union, its officers, directors and six shop stewards filed a Notice of
Strike with the NCMB.
The Union decided to participate in a mass action organized by the Alyansa in front of the
Companys premises. Thus, the Union officers and members held a picket along the front
perimeter of the plant on September 21, 1999. As a result, all of the 14 personnel of the
Engineering Section of the Company did not report for work, and 71 production personnel were
also absent. As a result, only one of the three bottling lines operated during the day shift. All the
three lines were operated during the night shift with cumulative downtime of five (5) hours due
to lack of manning, complement and skills requirement. The volume of production for the day
was short by 60,000 physical cases versus budget.
On October 13, 1999, the Company filed a Petition to Declare Strike Illegal
Issue: WON the strike, dubbed by petitioner as picketing, is illegal.
Held:
Article 212(o) of the Labor Code defines strike as a temporary stoppage of work by the
concerted action of employees as a result of an industrial or labor dispute. In Bangalisan v.
CA, the Court ruled that the fact that the conventional term strike was not used by the striking
employees to describe their common course of action is inconsequential, since the substance of
the situation, and not its appearance, will be deemed to be controlling.
Picketing involves merely the marching to and fro at the premises of the employer, usually
accompanied by the display of placards and other signs making known the facts involved in
a labor dispute. As applied to a labor dispute, to picket means the stationing of one or more
persons to observe and attempt to observe. The purpose of pickets is said to be a means of
peaceable persuasion.
The basic elements of a strike are present in this case. They marched to and fro in front of the
companys premises during working hours. Thus, petitioners engaged in a concerted activity
which already affected the companys operations. The mass concerted activity constituted a
strike.
For a strike to be valid, the following procedural requisites provided by Art 263 of the Labor
Code must be observed: (a) a notice of strike filed with the DOLE 30 days before the intended
date thereof, or 15 days in case of unfair labor practice; (b) strike vote approved by a majority
of the total union membership in the bargaining unit concerned obtained by secret ballot in a
meeting called for that purpose, (c) notice given to the DOLE of the results of the voting at least
seven days before the intended strike. These requirements are mandatory and the failure of a

union to comply therewith renders the strike illegal. It is clear in this case that petitioners totally
ignored the statutory requirements and embarked on their illegal strike.
Petition denied.
from Atty. Alba^^
Posted October 23, 2012 by vbdiaz in LABOR LAW

NUWHRAIN vs CA Leave a comment


NUWHRAIN vs CA
GR 163942 and 166295
Facts:
Because of the collective bargaining deadlock, petitioner Union staged a strike against the Hotel,
herein private respondent. This strike was declared illegal by the SC.
Issue: The effects of an illegal strike on employees.
Held:
Regarding the Union officers and members liabilities for their participation in the illegal picket
and strike, Article 264(a), paragraph 3 of the Labor Code provides that any union officer who
knowingly participates in an illegal strike and any worker or union officer who knowingly
participates in the commission of illegal acts during a strike may be declared to have lost his
employment status x x x. The law makes a distinction between union officers and mere union
members. Union officers may be validly terminated from employment for their participation in
an illegal strike, while union members have to participate in and commit illegal acts for them
to lose their employment status. Thus, it is necessary for the company to adduce proof of the
participation of the striking employees in the commission of illegal acts during the strikes.
Clearly, the 29 Union officers may be dismissed pursuant to Art. 264(a), par. 3 of the Labor
Code which imposes the penalty of dismissal on any union officer who knowingly participates
in an illegal strike. We, however, are of the opinion that there is room for leniency with respect
to the Union members. It is pertinent to note that the Hotel was able to prove before the NLRC
that the strikers blocked the ingress to and egress from the Hotel. But it is quite apparent that
the Hotel failed to specifically point out the participation of each of the Union members in
the commission of illegal acts during the picket and the strike. For this lapse in judgment or
diligence, we are constrained to reinstate the 61 Union members.
Further, we held in one case that union members who participated in an illegal strike but were
not identified to have committed illegal acts are entitled to be reinstated to their former positions
but without backwages.

from Atty. Alba^^


Posted October 23, 2012 by vbdiaz in LABOR LAW

MSF Tire and Rubber vs CA Leave a comment


MSF Tire and Rubber vs CA
GR 128632
Facts:
Respondent Union filed a notice of strike in the NCMB charging (Phildtread) with unfair labor
practice. Thereafter, they picketed and assembled outside the gate of Philtreads plant. Philtread,
on the other hand, filed a notice of lockout. Subsequently, the Secretary of Labor assumed
jurisdiction over the labor dispute and certified it for compulsory arbitration.
During the pendency of the labor dispute, Philtread entered into a Memorandum of Agreement
with Siam Tyre whereby its plant and equipment would be sold to a new company, herein
petitioner, 80% of which would be owned by Siam Tyre and 20% by Philtread, while the land on
which the plant was located would be sold to another company, 60% of which would be owned
by Philtread and 40% by Siam Tyre.
Petitioner then asked respondent Union to desist from picketing outside its plant. As the
respondent Union refused petitioners request, petitioner filed a complaint for injunction with
damages before the RTC. Respondent Union moved to dismiss the complaint alleging lack of
jurisdiction on the part of the trial court.
Petitioner asserts that its status as an innocent bystander with respect to the labor dispute
between Philtread and the Union entitles it to a writ of injunction from the civil courts.
Issue: WON petitioner has shown a clear legal right to the issuance of a writ of injunction under
the innocent bystander rule.
Held:
In Philippine Association of Free Labor Unions (PAFLU) v. Cloribel, this Court, through Justice
J.B.L. Reyes, stated the innocent bystander rule as follows:
The right to picket as a means of communicating the facts of a labor dispute is
a phase of the freedom of speech guaranteed by the constitution. If peacefully
carried out, it cannot be curtailed even in the absence of employer-employee
relationship.
The right is, however, not an absolute one. While peaceful picketing is entitled to protection as
an exercise of free speech, we believe the courts are not without power to confine or localize the
sphere of communication or the demonstration to the parties to the labor dispute, including those

with related interest, and to insulate establishments or persons with no industrial connection or
having interest totally foreign to the context of the dispute. Thus the right may be regulated at
the instance of third parties or innocent bystanders if it appears that the inevitable result of its
exercise is to create an impression that a labor dispute with which they have no connection or
interest exists between them and the picketing union or constitute an invasion of their rights.
Thus, an innocent bystander, who seeks to enjoin a labor strike, must satisfy the court it is
entirely different from, without any connection whatsoever to, either party to the dispute and,
therefore, its interests are totally foreign to the context thereof.
In the case at bar, petitioner cannot be said not to have such connection to the dispute. We
find that the negotiation, contract of sale, and the post transaction between Philtread, as
vendor, and Siam Tyre, as vendee, reveals a legal relation between them which, in the interest
of petitioner, we cannot ignore. To be sure, the transaction between Philtread and Siam Tyre,
was not a simple sale whereby Philtread ceased to have any proprietary rights over its sold
assets. On the contrary, Philtread remains as 20% owner of private respondent and 60% owner
of Sucat Land Corporation which was likewise incorporated in accordance with the terms of the
Memorandum of Agreement with Siam Tyre, and which now owns the land were subject plant
is located. This, together with the fact that private respondent uses the same plant or factory;
similar or substantially the same working conditions; same machinery, tools, and equipment; and
manufacture the same products as Philtread, lead us to safely conclude that private respondents
personality is so closely linked to Philtread as to bar its entitlement to an injunctive writ.
Petition denied.
from Atty. Alba^^
Posted October 23, 2012 by vbdiaz in LABOR LAW

Manila Diamond Hotel Employee Union vs CA Leave a


comment
Manila Diamond Hotel Ee Union vs CA
GR 140518
Facts:
The Union filed a petition for a certification election, which was dismissed by the DOLE.
Despite the dismissal of their petition, the Union sent a letter to the Hotel informing the latter
of its desire to negotiate for a collective bargaining agreement. The Hotel, however, refused to
negotiate with the Union, citing the earlier dismissal of the Unions petition for certification by
DOLE.
Failing to settle the issue, the Union staged a strike against the Hotel. Numerous confrontations
followed, further straining the relationship between the Union and the Hotel. The Hotel claims

that the strike was illegal and dismissed some employees for their participation in the allegedly
illegal concerted activity. The Union, on the other hand, accused the Hotel of illegally dismissing
the workers.
A Petition for Assumption of Jurisdiction under Article 263(g) of the Labor Code was later
filed by the Union before the Secretary of Labor. Thereafter, Secretary of Labor Trajano issued
an Order directing the striking officers and members of the Union to return to work within
twenty-four (24) hours and the Hotel to accept them back under the same terms and conditions
prevailing prior to the strike.
After receiving the above order the members of the Union reported for work, but the Hotel
refused to accept them and instead filed a Motion for Reconsideration of the Secretarys Order.
Acting on the motion for reconsideration, then Acting Secretary of Labor Espaol modified the
one earlier issued by Secretary Trajano and instead directed that the strikers be reinstated only in
the payroll.
Issue: WON payroll reinstatement is proper in lieu of actual reinstatement under Article 263(g)
of the Labor Code.
Held:
Payroll reinstatement in lieu of actual reinstatement is not sanctioned under the provision of the
said article.
The Court noted the difference between UST vs. NLRC and the instant case. In UST case the
teachers could not be given back their academic assignments since the order of the Secretary
for them to return to work was given in the middle of the first semester of the academic year.
The NLRC was, therefore, faced with a situation where the striking teachers were entitled to a
return to work order, but the university could not immediately reinstate them since it would be
impracticable and detrimental to the students to change teachers at that point in time.
In the present case, there is no similar compelling reason that called for payroll reinstatement as
an alternative remedy. A strained relationship between the striking employees and management
is no reason for payroll reinstatement in lieu of actual reinstatement.
Under Article 263(g), all workers must immediately return to work and all employers must
readmit all of them under the same terms and conditions prevailing before the strike or lockout.
The Court pointed out that the law uses the precise phrase of under the same terms and
conditions, revealing that it contemplates only actual reinstatement. This is in keeping with the
rationale that any work stoppage or slowdown in that particular industry can be inimical to the
national economy.
The Court reiterates that Article 263(g) was not written to protect labor from the excesses of
management, nor was it written to ease management from expenses, which it normally incurs
during a work stoppage or slowdown. This law was written as a means to be used by the State to
protect itself from an emergency or crisis. It is not for labor, nor is it for management.

Petition granted.
from Atty. Alba^^
Posted October 23, 2012 by vbdiaz in LABOR LAW

Interphil Laboratories Employee Union vs


Interphil Laboratories Leave a comment
Interphil Laboratories Ee Union vs Interphil Laboratories
GR 142824
Facts:
Petitioner is the sole and exclusive bargaining agent of the rank-and-file employees of
Respondent. They had a CBA.
Prior to the expiration of the CBA, respondent company was approached by the petitioner,
through its officers. The Union inquired about the stand of the company regarding the duration
of the CBA which was set to expire in a few months. Salazar told the union officers that the
matter could be best discussed during the formal negotiations which would start soon.
All the rank-and-file employees of the company refused to follow their regular two-shift
work schedule. The employees stopped working and left their workplace without sealing the
containers and securing the raw materials they were working on.
To minimize the damage the overtime boycott was causing the company, Salazar immediately
asked for a meeting with the union officers. In the meeting, Enrico Gonzales, a union director,
told Salazar that the employees would only return to their normal work schedule if the company
would agree to their demands as to the effectivity and duration of the new CBA. Salazar again
told the union officers that the matter could be better discussed during the formal renegotiations
of the CBA. Since the union was apparently unsatisfied with the answer of the company, the
overtime boycott continued. In addition, the employees started to engage in a work slowdown
campaign during the time they were working, thus substantially delaying the production of the
company.
Respondent company filed with the National NLRC a petition to declare illegal petitioner
unions overtime boycott and work slowdown which, according to respondent company,
amounted to illegal strike. It also filed with Office Secretary of Labor a petition for assumption
of jurisdiction. Secretary of Labor Nieves Confesor issued an assumption order over the labor
dispute.
Labor Arbiter Caday submitted his recommendation to the then Secretary of Labor Leonardo A.
Quisumbing. Then Secretary Quisumbing approved and adopted the report in his Order, finding
illegal strike on the part of petitioner Union.

Issue: WON the Labor Secretary has jurisdiction to rule over an illegal strike.
Held:
On the matter of the authority and jurisdiction of the Secretary of Labor and Employment to
rule on the illegal strike committed by petitioner union, it cannot be denied that the issues
of overtime boycott and work slowdown amounting to illegal strike before Labor Arbiter
Caday are intertwined with the labor dispute before the Labor Secretary.
The appellate court also correctly held that the question of the Secretary of Labor and
Employments jurisdiction over labor-related disputes was already settled in International
Pharmaceutical, Inc. vs. Hon. Secretary of Labor and Associated Labor Union (ALU) where the
Court declared:
In the present case, the Secretary was explicitly granted by Article 263(g) of the
Labor Code the authority to assume jurisdiction over a labor dispute causing or
likely to cause a strike or lockout in an industry indispensable to the national
interest, and decide the same accordingly. Necessarily, this authority to assume
jurisdiction over the said labor dispute must include and extend to all questions
and controversies arising therefrom, including cases over which the labor arbiter
has exclusive jurisdiction.
Moreover, Article 217 of the Labor Code is not without, but contemplates, exceptions
thereto. This is evident from the opening proviso therein reading (e)xcept as otherwise
provided under this Code x x x. Plainly, Article 263(g) of the Labor Code was meant to make
both the Secretary (or the various regional directors) and the labor arbiters share jurisdiction,
subject to certain conditions. Otherwise, the Secretary would not be able to effectively and
efficiently dispose of the primary dispute. To hold the contrary may even lead to the absurd and
undesirable result wherein the Secretary and the labor arbiter concerned may have diametrically
opposed rulings. As we have said, it is fundamental that a statute is to be read in a manner that
would breathe life into it, rather than defeat it.
In fine, the issuance of the assailed orders is within the province of the Secretary as authorized
by Article 263(g) of the Labor Code and Article 217(a) and (5) of the same Code, taken
conjointly and rationally construed to subserve the objective of the jurisdiction vested in the
Secretary.
Petition denied.
from Atty. Alba^^
Posted October 23, 2012 by vbdiaz in LABOR LAW

GLOBAL INC. vs. ATIENZA ET AL Leave a comment


GLOBAL INC. vs. ATIENZA ET AL

G.R. No.L-51612-13
JULY 22, 1986

FACTS: Rosal, herein private respondent, commenced her employment with petitioner Global
Incorporated in February, 1970, as a Sales Clerk. In November 1976 Global Inc. filed with the
Department of Labor Regional Office, an application for clearance to terminate the services of
Clarita Rosal, for having violated company rules and regulations by incurring repeated absences
and tardiness. The subject employee was placed under preventive suspension on November
16, 1976 pending resolution of the application for clearance.c
Clarita Rosal filed her opposition to the clearance application as well as a counter-complaint
against Global Inc., for illegal dismissal, overtime pay and premium pay.
The officer-in-charge of Regional Office, Ministry of labor Leogardo, Jr. lifted the preventive
suspension of Clarita Rosal, finding her suspension not warranted, and reinstated her to her
former position without loss of rights and with full backwages from the time of preventive
suspension up to the date of her actual reinstatement.
The Labor Arbiter rendered his decision dismissing the complaint for illegal dismissal, overtime
compensation and premium pay, and the clearance for the complainants termination is granted.
Rosal appealed the aforesaid decision to the NLRC.Respondents Commissioners Atienza and
Quadra modified the appealed decision, whereby:
(a) respondent is ordered to pay complainant overtime pay for the period Nov. 1, 1974 to Nov.
16, 1976 when she was suspended;
(b) respondent is likewise ordered to pay complainant backwages from Dec. 2, 1976 to May 31,
1978;
(c) the decision of the Labor Arbiter granting clearance to terminate the services of the
complainant is affirmed.
Respondent Commissioner Villatuya voted to affirm the Labor Arbiters decision. Hence, the
instant petition.
ISSUE: WON

1. Rosal is entitled to overtime pay


2. Rosal is entitled to backwages

HELD: The assailed decision of the NLRC is modified, where the order to pay overtime pay to
Rosal is set aside, the order to pay Rosal backwages affirmed, and the decision granting
clearance to terminate the services of Rosal likewise affirmed
1. NO. We agree with the conclusion of the Labor Arbiter that the same should be denied for
want of sufficient factual and legal basis. No employee is authorized to work after office hours,
during Sundays and Holidays unless required by a written memorandum from the General
Manager. During the period from Nov. 1, 1974 to Nov. 16, 1976, no employee of the company
was never required to work after 5:00 in the afternoon. There is nothing in the record except her
bare allegations which would show that she truly and actually rendered said overtime work
2. YES. the NLRC ordered petitioner to pay Rosal backwages from Dec. 2, 1976 to May 31,
1978, the date when Asst. Secretary Leogardo, Jr., rendered his decision lifting the preventive
suspension of Rosal and ordering petitioner to reinstate her to her former position without loss of
rights and with full backwages from the time of preventive suspension up to the date of her
actual reinstatement.c
We agree. We note that this decision of the Labor Arbiter ordering reinstatement had not been
complied with. Neither was it appealed by petitioner, therefore, the decision had become final
and executory. To exempt petitioner from the payment of backwages would be to give premium
to the blant disregard of orders of the Ministry of Labor. Moreover, it would be in consonance
with compassionate justice that Rosal be paid backwages during the period that she was
supposed to be reinstated
Note that the only ground for the imposition of preventive suspension is provided for under Sec.
4, Rule XIV of the Implementing Regulations of the Ministry of Labor which readsSEC. 4. Preventive suspension. The employer may place the employee concerned under
preventive suspension only if the continued employment of the employee poses a serious and
imminent threat to the life or property of the employer or of the co-employees. Any preventive
suspension before the filing of the application shall be considered worked days, and shall be duly
paid as such if the continued presence of the employee concerned does not pose a serious threat
to the life and property of the employer or of the co-employees.
As aptly held by Asst. Secretary Leogardo Jr., the continued presence of Clarita Rosal never
posed a serious and imminent threat to the life or property of the employer or co-employees as
would warrant her preventive suspension
Posted March 25, 2011 by vbdiaz in LABOR LAW

SALAZAR VS. NLRC Leave a comment


SALAZAR VS. NLRC

G.R. No 109210
APRIL 17, 1996
FACTS: On 17 April 1990, private respondent Carlos Construction, at a monthly salary of
P4,500.00, employed Salazar as construction/project engineer for the construction of a building
in Cubao. Allegedly, by virtue of an oral contract, petitioner would also receive a share in the
profits after completion of the project and that petitioners services in excess of 8 ours on regular
days and services rendered on weekends and legal holidays shall be compensable overtime.
On 16 April 1991, petitioner received a memorandum issued by private respondents project
manager informing him of the termination of his services effective on 30 April 1991.
On 13 September 1991, Salazar filed a complaint against private respondent for illegal dismissal,
unfair labor practice, illegal deduction, non-payment of wages, overtime rendered, service
incentive leave pay, commission, allowances, profit-sharing and separation pay with the NLRCNCR Arbitration Branch, Manila.
The Labor Arbiter rendered a decision dismissing the instant case for lack of merit. Petitioner
appealed to the NLRC, where it affirmed in toto the decision of the Labor Arbiter. His MR was
likewise dismissed. Hence the instant petition.
ISSUE:
1) WON petitioner is entitled to overtime pay, premium pay for services rendered on rest days
and holidays and service incentive leave pay
2) WON petitioner is entitled to a share in the profits of the construction project;.
3) WON petitioner rendered services from 1 May to 15 May 1991 and is, therefore, entitled to
unpaid wages;
4) WON private respondent is liable to reimburse petitioners legal expenses and;
5) WON petitioner is entitled to separation pay.
HELD: The assailed decision is modified.
1. NO. Although petitioner cannot strictly be classified as a managerial employee, nonetheless
he is still not entitled to payment of the aforestated benefits because he falls squarely under
another exempt category officers or members of a managerial staff as defined under sec.
2(c) of the abovementioned implementing rules:
Sec. 2. Exemption. The provisions of this Rule shall not apply to the following persons if they
qualify for exemption under the condition set forth herein:xxx

(c) Officers or members of a managerial staff xxx

That petitioner was paid overtime benefits does not automatically and necessarily denote that
petitioner is entitled to such benefits

1. NO. petitioner insists that private respondent promised him a share in the profits after
completion of the construction project. It is because of this oral agreement, petitioner elucidates,
that he agreed to a monthly salary of P4,500.00, an amount which he claims is too low for a
professional civil engineer like him with the rank of project engineer.

We cannot accede to petitioners demand. Nowhere in the disbursement vouchers can we find
even the remotest hint of a profit-sharing agreement between petitioner and private respondent.
Petitioners rationalization stretches the imagination way too far.
Also, as said by the Labor Arbiter:

As to the issue of profit sharing, we simply cannot grant the same on the mere basis of
complainants allegation that respondent verbally promised him that he is entitled to a share in
the profits derive(d) from the projects. Benefits or privileges of this nature (are) usually in
writing, besides complainant failed to (establish) that said benefits or privileges (have) been
given to any of respondent(s) employees as a matter of practice or policy.

3. YES. On April 30, he was advised by the Manager to continue supervising the finishing
touches to the building until May 15, the date appearing in the Certificate of Service as the date
of the termination of the contract between Salazar and the Company. But the Manager insists that
Salazars services terminated at April 30 according to the Memorandum given the petitioner.
The purpose for which the said certificate was issued becomes irrelevant. The fact remains that
private respondent knowingly and voluntarily issued the certificate. Mere denials and selfserving statements to the effect that petitioner allegedly promised not to use the certificate
against private respondent are not sufficient to overturn the same. Hence, private respondent is
estopped from assailing the contents of its own certificate of service.

4. YES. During the construction of the building, a criminal complaint for unjust vexation was
filed against the officers of the owner of the building. Petitioner avers that he was implicated in
the complaint for the sole reason that he was the construction engineer of the project.

Although not directly implicated in the criminal complaint, Carlos Construction is nonetheless
obligated to defray petitioners legal expenses. Petitioner was included in the complaint not in his
personal capacity but in his capacity as project engineer of private respondent and the case arose
in connection with his work as such. At the construction site, petitioner is the representative of
private respondent being its employee and he acts for and in behalf of private respondent. Hence,
the inclusion of petitioner in the complaint for unjust vexation, which was work-related, is
equivalent to inclusion of private respondent itself.

5. NO. On the last issue, we rule that petitioner is a project employee and, therefore, not entitled
to separation pay.
The applicable provision is Article 280 of the Labor Code which defines the term project
employee, thus:
Art. 280. Regular and Casual Employment. The provisions of written agreement to the
contrary notwithstanding and regardless of the oral agreement of the parties, an employment
shall be deemed to be regular where the employee has been engaged to perform activities which
are usually necessary or desirable in the usual business or trade of the employer, except where
the employment has been fixed for a specific period or undertaking the completion or
termination of which has been determined at the time of the engagement of the employee or
where the work or services to be performed is seasonal in nature and the employment is for the
duration of the season. (Emphasis ours.)
In the case at bench, it was duly established that private respondent hired petitioner as project or
construction engineer specifically for its Monte de Piedad building project. Accordingly, as
project employee, petitioners services are deemed coterminous with the project, that is,
petitioners services may be terminated as soon as the project for which he was hired is
completed. There can be no dispute that petitioners dismissal was due to the completion of the
construction of the building.
NOTES:
1. Although we agree with private respondent that appeals to the SC from decisions of the NLRC
should be in the form of a special civil action for certiorari under Rule 65 of the Revised Rules of
Court, this rule is not inflexible. In a number of cases this Court has resolved to treat as special
civil actions for certiorari petitions erroneously captioned as petitions for review on certiorari in
the interest of justice.

2. Policy Instruction No. 20 entitled Stabilizing Employer-Employee Relations in the


Construction Industry explicitly mandates that:
xxx xxx xxx
Project employees are not entitled to termination pay if they are terminated as a result of the
completion of the project or any phase thereof in which they are employed, regardless of the
number of projects in which they have been employed by a particular construction company.
Moreover, the company is not required to obtain a clearance from the Secretary of Labor in
connection with such termination. What is required of the company is a report to the nearest
Public Employment Office for statistical purposes.
Posted March 25, 2011 by vbdiaz in LABOR LAW

PNB V PNB EMPLOYEES ASSOCIATION Leave a


comment
PNB V PNB EMPLOYEES ASSOCIATION
115 SCRA 507
July 30, 1982
NATURE
Appeal from decision of the Court of Industrial Relations (CIR)
FACTS
- PNB and PNB Employees Association (PEMA) had a dispute regarding the proper computation
of overtime pay. PEMA wanted the cost of living allowance (granted in 1958) and longevity pay
(granted in 1961) to be included in the computation. PNB disagreed and the 2 parties later went
before the CIR to resolve the dispute.
- CIR decided in favor of PEMA and held that PNB should compute the overtime pay of its
employees on the basis of the sum total of the employees basic salary or wage plus cost of living
allowance and longevity pay. The CIR relied on the ruling in NAWASA v NAWASA
Consolidated Unions, which held that for purposes of computing overtime compensation,
regular wage includes all payments which the parties have agreed shall be received during the
work week, including differentiated payments for working at undesirable times, such as at night
and the board and lodging customarily furnished the employee. This prompted PNB to appeal,
hence this case.
ISSUE

WON the cost of living allowance and longevity pay should be


included in the computation of overtime pay as held by the CIR

HELD
NO
Ratio Overtime pay is for extra effort beyond that
contemplated in the employment contract; additional pay given for any other purpose cannot be
included in the basis for the computation of overtime pay.
- Absent a specific provision in the CBA, the bases for the
computation of overtime pay are 2 computations, namely:

1. WON the additional pay is for extra work done or service


rendered
2. WON the same is intended to be permanent and regular, not contingent nor temporary as a
given only to remedy a situation which can change any time.
Reasoning
- Longevity pay cannot be included in the computation of
overtime pay for the very simple reason that the contrary is expressly stipulated in the CBA,
which constitutes the law between the parties.
- As regards cost of living allowance, there is nothing in Commonwealth Act 444 [or the 8-hour
Labor Law, now Art. 87 Labor Code] that could justify PEMAs posture that it should be added
to the regular wage in computing overtime pay. C.A. 444 prescribes that overtime work shall be
paid at the same rate as their regular wages or salary, plus at least 25% additional. The law did
not define what is a regular wage or salary. What the law emphasized is that in addition to
regular wage, there must be paid an additional 25% of that regular wage to constitute
overtime rate of pay. Parties were thus allowed to agree on what shall be mutually considered
regular pay from or upon which a 25% premium shall be based and added to makeup overtime
compensation.

- No rule of universal application to other cases may be justifiably extracted from the NAWASA
case. CIR relies on the part of the NAWASA decision where the SC cited American decisions
whose legislation on overtime is at variance with the law in this jurisdiction. The US legislation
considers work in excess of forty hours a week as overtime; whereas, what is
generally considered overtime in the Philippines is work in
excess of the regular 8 hours a day. It is understandably
material to refer to precedents in the US for purposes of computing weekly wages under a 40hour week rule, since the particular issue involved in NAWASA is the conversion of prior weekly
regular earnings into daily rates without allowing diminution or addition.
- To apply the NAWASA computation would require a different formula for each and every
employee. It would require reference to and continued use of individual earnings in the past, thus
multiplying the administrative difficulties of the Company. It would be cumbersome and tedious
a process to compute overtime pay and this may again cause delays in payments, which in turn
could lead to serious disputes. To apply this mode of computation would retard and stifle the
growth of unions themselves as Companies would be irresistibly drawn into denying, new and
additional fringe benefits, if not those already existing, for fear of bloating their overhead
expenses through overtime which, by reason of being unfixed, becomes instead a veritable
source of irritant in labor relations.
**Overtime Pay Rationale Why is a laborer or employee who works beyond the regular hours of
work entitled to extra compensation called, in this enlightened time, overtime pay?
Verily, there can be no other reason than that he is made to work longer than what is
commensurate with his agreed compensation for the statutorily fixed or voluntarily agreed hours
of labor he is supposed to do. When he thus spends additional time to his work, the effect upon
him is multi- faceted; he puts in more effort, physical and/or mental; he is delayed in going home
to his family to enjoy the comforts thereof; he might have no time for relaxation, amusement or
sports; he might miss important pre-arranged engagements; etc. It is thus the additional work,
labor or service employed and the adverse effects just mentioned of his longer stay in his place of
work that justify and are the real reasons for the extra compensation that is called overtime pay.
**Overtime Pay Definition The additional pay for service or
work rendered or performed in excess of 8 hours a day by employees or laborers in employment
covered by the 8 hour Labor Law [C.A. 444, now Art. 87 Labor Code] and not exempt from its
requirements. It is computed by multiplying the overtime hourly rate by the number of hours
worked in excess of eight.
Disposition decision appealed from is REVERSED

Posted March 25, 2011 by vbdiaz in LABOR LAW

PHILIPPINE AIRLINES vs. NLRC et al Leave a comment


PHILIPPINE AIRLINES vs. NLRC et al
G.R. No. 132805
Feb. 2, 1999

FACTS: Private respondent Dr. Fabros was employed as flight surgeon at petitioner company.
He was assigned at the PAL Medical Clinic and was on duty from 4:00 in the afternoon until
12:00 midnight.
On Feb.17, 1994, at around 7:00 in the evening, Dr. FAbros left the clinic to have his dinner at
his residence, which was abou t5-minute drive away. A few minutes later, the clinic received an
emergency call from the PAL Cargo Services. One of its employeeshad suffered a heart attack.
The nurse on duty, Mr. Eusebio, called private respondent at home to inform him of the
emergency. The patient arrived at the clinic at 7:50 in the evening and Mr. Eusebio immediately
rushed him to the hospital. When Dr. Fabros reached the clinic at around 7:51 in the evening, Mr.
Eusebio had already left with the patient to the hospital. The patient died the following day.
Upon learning about the incident, PAL Medical Director ordered the Chief Flight Surgeon to
conduct an investigation. In his explanation, Dr. Fabros asserted that he was entitled to a thirtyminute meal break; that he immediately left his residence upon being informed by Mr. Eusebio
about the emergency and he arrived at the clinic a few minutes later; that Mr. Eusebio panicked
and brought the patient to the hospital without waiting for him.
Finding private respondents explanation unacceptable, the management charged private
respondent with abandonment of post while on duty. He denied that he abandoned his post on
February 17, 1994. He said that he only left the clinic to have his dinner at home. In fact, he
returned to the clinic at 7:51 in the evening upon being informed of the emergency.
After evaluating the charge as well as the answer of private respondent, he was given a
suspension for three months effective December 16, 1994.
Private respondent filed a complaint for illegal suspension against petitioner.
On July 16, 1996, the Labor Arbiter rendered a decision declaring the suspension of private
respondent illegal. It also ordered petitioner to pay private respondent the amount equivalent to
all the benefits he should have received during his period of suspension plus P500,000.00 moral
damages.
Petitioner appealed to the NLRC.

The NLRC, however, dismissed the appeal after finding that the decision of the Labor Arbiter is
supported by the facts on record and the law on the matter. The NLRC likewise denied
petitioners motion for reconsideration.
Hence, this petition.
ISSUE:
1. WON the nullifying of the 3-month suspension by the NLRC erroneous.
2. WON the awarding of moral damages is proper.
HELD: The petition is PARTIALLY GRANTED. The portion of the assailed decision awarding
moral damages to private respondent is DELETED. All other aspects of the decision are
AFFIRMED
1. The legality of private respondents suspension: Dr. Fabros left the clinic that night only to
have his dinner at his house, which was only a few minutes drive away from the clinic. His
whereabouts were known to the nurse on duty so that he could be easily reached in case of
emergency. Upon being informed of Mr. Acostas condition, private respondent immediately left
his home and returned to the clinic. These facts belie petitioners claim of abandonment.
Petitioner argues that being a full-time employee, private respondent is obliged to stay in the
company premises for not less than eight (8) hours. Hence, he may not leave the company
premises during such time, even to take his meals. We are not impressed. Art. 83 and 85 of the
Labor Code read: Art. 83. Normal hours of work. The normal hours of work of any employee
shall not exceed eight (8) hours a day. Health personnel in cities and municipalities with a
population of at least one million (1,000,000) or in hospitals and clinics with a bed capacity of at
least one hundred (100) shall hold regular office hours for eight (8) hours a day, for five (5) days
a week, exclusive of time for meals, except where the exigencies of the service require that such
personnel work for six (6) days or forty-eight (48) hours, in which case they shall be entitled to
an additional compensation of at least thirty per cent (30%) of their regular wage for work on the
sixth day. For purposes of this Article, health personnel shall include: resident physicians,
nurses, nutritionists, dieticians, pharmacists, social workers, laboratory technicians, paramedical
technicians, psychologists, midwives, attendants and all other hospital or clinic personnel.
(emphasis supplied) Art. 85. Meal periods. Subject to such regulations as the Secretary of
Labor may prescribe, it shall be the duty of every employer to give his employees not less than
sixty (60) minutes time-off for their regular meals. Sec. 7, Rule I, Book III of the Omnibus Rules
Implementing the Labor Code further states: Sec. 7. Meal and Rest Periods. Every employer
shall give his employees, regardless of sex, not less than one (1) hour time-off for regular meals,
except in the following cases when a meal period of not less than twenty (20) minutes may be
given by the employer provided that such shorter meal period is credited as compensable hours
worked of the employee; (a) Where the work is non-manual work in nature or does not involve
strenuous physical exertion; (b) Where the establishment regularly operates not less than sixteen
hours a day; (c) In cases of actual or impending emergencies or there is urgent work to be
performed on machineries, equipment or installations to avoid serious loss which the employer
would otherwise suffer; and (d) Where the work is necessary to prevent serious loss of perishable

goods. Rest periods or coffee breaks running from five (5) to twenty (20) minutes shall be
considered as compensable working time. Thus, the eight-hour work period does not include the
meal break. Nowhere in the law may it be inferred that employees must take their meals within
the company premises. Employees are not prohibited from going out of the premises as long as
they return to their posts on time. Private respondents act, therefore, of going home to take his
dinner does not constitute abandonment. 2. The award of moral damages: Not every employee
who is illegally dismissed or suspended is entitled to damages. As a rule, moral damages are
recoverable only where the dismissal or suspension of the employee was attended by bad faith or
fraud, or constituted an act oppressive to labor, or was done in a manner contrary to morals, good
customs or public policy In the case at bar, there is no showing that the management of petitioner
company was moved by some evil motive in suspending private respondent. It suspended private
respondent on an honest, albeit erroneous, belief that private respondents act of leaving the
company premises to take his meal at home constituted abandonment of post which warrants the
penalty of suspension. Under the circumstances, we hold that private respondent is not entitled to
moral damages.

Вам также может понравиться